Mathematics - Phase 1
Mathematics - Phase 1
BASIC CALCULUS
CONTENTS
Introduction 1
Functions 1
Differential Calculus 3
Exercise 1 6
Exercise 2 8
Behaviour of Functions 9
Exercise 3 11
Integral Calculus 11
Exercise 4 14
Definite Integral 15
Differential Equations 16
Exercise 5 17
Answers to Exercises 18
Solved Problems 19
Assignment Problems 32
Answers to Chapter 44
Practice Problems
Answers to Assignment 45
Problems
Introduction:
Calculus is the study of the concepts of functions and their behavior. These concepts are based
on the theory of real numbers. Broadly, it is classified into two parts: Differential calculus and
Integral Calculus. Differential Calculus deals with finding the rate of change of one physical
quantity with respect to (written as w.r.t. from now onwards) another when the two are connected
by a functional relation.
Integral Calculus, on the other hand, deals with finding one physical quantity in terms of the other
(i.e., the functional dependence) given the rate of change of the first with respect to the second.
A variable is a quantity that takes on various numerical values. A constant is a quantity whose
numerical value remains fixed e.g. when a body falls towards the surface of the earth, its velocity
increases but the acceleration (due to gravity) remains constant.
It should be noted that when considering specific physical phenomena, it may happen that one
and the same quantity in one phenomena is a constant while in another it is a variable. For
example, the velocity in a uniform motion is constant, whereas the velocity in a uniformly
accelerated motion is variable.
Quantities that have the same value under all circumstances are called absolute constants. For
example, the ratio of the circumference of a circle to its diameter (an absolute constant ), period
of rotation of earth about its axis or around the sun, universal gravitational constant, permeability
of free space, etc.
Functions
In the study of natural phenomena and the solution of technical and mathematical problems, it is
necessary to consider the variation of one quantity as dependent on the variation of another. For
example, in studies of motion, the path traversed is regarded as a variable, which varies with
time. Here we say that the distance traversed is a function of time. The area of a circle, in terms
2
of its radius R, is R . If R takes on various numerical values, the area assumes different
numerical values. So the variation of one variable brings about a variation in the other. Hence
area of the circle is a function of its radius R.
If to each value of variable x (within a certain range) there corresponds a unique value of another
variable y, then we say that y is a function of x, or, in functional notation y = f(x). The variable x is
called the independent variables or argument and the variable y is called the dependent variable.
The relation between the variable x and y is called a functional relation. The letter 'f' in the
functional notation y = f(x) indicates that some kind of operation must be performed on the values
of x in order to obtain the corresponding values of y.
P-2022-CBSE-P1-MATHEMATICS-BASIC CALCULUS
2
f(x) L
y = f(x)
C f(x, y) =0
y3
y2 B
y2
y1 y1
A
x2
x1 x3 x x0 x
Fig. (a) Fig . (b)
These figures show the graph of two arbitrary curves. In fig.(a) any line drawn parallel to the y-axis
would meet the curve at only one point. That means each element of x would have one and only one
image. Thus fig (a) would represent the graph of a function.
In fig.(b) certain line (e.g. line L) would meet the curve in more than one points (A, B and C). Thus
element xo of x would have three distinct images. Thus this curve will not represent a function.
The set of all possible values which the independent variable (here 'x') is permitted to take for a
given functional dependence to be defined is called the domain of definition or simply the
domain of the function.
e.g. The function y = sin x is defined for all values of x. Therefore its domain of definition is the
infinite interval < x < .
1
The function y = is defined for all x 1 its domain is (1, ).
x 1
Range of a Variable
The set of all numerical values of a dependent variable quantity is called the range of the
variable. e.g. The range of values of the variable y = cos x for all possible values of x is the
interval [1, 1] i.e. –1 y 1.
P-2022-CBSE-P1-MATHEMATICS-BASIC CALCULUS
3
vi) Algebraic functions:
(a) Polynomial function: y = a0xn + a1xn-1 + … + an, where a0, a1….an are real
constants (a0 0) and n is a non-negative integer, is called a polynomial of degree n.
e.g: y = ax + b, a 0 (a linear function),
2
y = ax + bx +c, a0 (a quadratic function),
A polynomial function is defined for all real values of x.
a0 xn a1xn 1 ..... an
(b) Rational function: y = ; n, m I+
m m1
b0 x b1x .... bm
e.g: y = a/x (inverse variation).
The rational function is defined for all values of x except for those where the denominator
becomes zero.
(c) Irrational function:
2x 2 x
e.g. y= .
1 5x 2
Differential Calculus
Let y = f(x) be a function of x. Putting the values of 'x' in this relation, we obtain the corresponding
values of 'y'. Suppose we start putting some values of 'x' in increasing order. The respective
values of 'y' that we obtain may turn out to be in increasing order, or in decreasing order, or they
may remain constant, or they may even have a mixed trend, depending upon the type of function.
Let us take two values of x: x1 and x2 (x1 < x2), so that, y1 = f(x1) and y2 = f(x2).
y y1
Then, the quantity 2 will tell us the average rate of change of y w.r.t. x in the
x 2 x1
interval [x1, x2].
y2 y1
Let y2 > y1 is positive Function is increasing on an average.
x2 x1
y2 y1
If y2 < y1 is negative Function is decreasing on an average.
x2 x1
y2 y1
If y2 = y1 is zero Function is constant on an average.
x2 x1
y y1
As you can see, if x1 and x2 are sufficiently far apart, the quantity 2 can not give the
x 2 x1
exact idea of the variation of y w.r.t. x in the interval [x1,x2]. It just provides an overall information.
For example, if y2 = y1, it does not necessarily mean that y is the same for all x in the interval
[x1,x2]. Thus, in order to obtain a "sufficiently accurate information", we have to choose x1 and x2
" sufficiently close" to each other. This 'sufficiently close' is the key word here. To know the rate of
change of y w.r.t. x at x = x1, we take x2 very near to x1 ( as much as possible), i.e, 'x2 tends to x1'
y y1
and then calculate 2 . In the limiting case,
x2 x1
P-2022-CBSE-P1-MATHEMATICS-BASIC CALCULUS
5
Theorem 1
dy df(x)
If a function is of the form y = k f(x), where k is a constant, then =k .
dx dx
Theorem 2
The derivative of the sum or difference of a finite number of differentiable functions is equal to the
sum or difference of the derivatives of these functions,
i.e. if y = u (x) + v(x) + w(x), then y ' = u'(x) + v'(x) + w'(x).
1
Illustration 1. Find the derivative of y = 3x (x)1/3 .
x
Theorem 3
The derivative of the product of two differentiable functions is equal to the product of the
derivative of the first function with the second function plus the product of the first function with
the derivative of the second function: i.e. if y = uv , then y' = u'v + uv'.
This formula can be extended for the derivatives of the product of any (finite) number of functions.
Theorem 4
u(x) dy u ' v uv '
If y = , then y' = .
v(x) dx v2
ax
Illustration 3. Find the derivative of y = w.r.t. x.
ax
dy
Solution: (i) y = xx x
x.x x 1.1 x x ln x.1 = x (1+lnx).
dx
x2 dy 2 d 2 dx2
(ii) y = sin x x 2 (sin x )x 1 (sin x) (sin x) x l n sinx
dx dx dx
2 2
= x 2 (sin x )x -1
cos x (sin x)x (ln sinx).2x .
Exercise 1.
i) Find the derivative of the following functions w.r.t. x :
7/2 5/2
(a) y = 6x + 4x – x, (b) y = tan (ln x),
2 x
(c) y = ln (x +1), (d) y = (sin x) .
2x dy dy
ii) (a) For y = 2 , find at x = 0. (b) If y = x 2 1 , find at x = 1.
x 1 dx dx
iii) Find the derivative of
x
(a) y = ax 2 2bx c , (b) y = ( e ) x
(c) y = cos(cotx) (d) (sinx)cosx
P-2022-CBSE-P1-MATHEMATICS-BASIC CALCULUS
7
Parametric Representation of a Function and it’s Derivative
Y
We find the trajectory of a load dropped from an aeroplane moving
horizontally with uniform velocity v0 at an altitude y0. vO
We take the co-ordinate system as shown and assume that the
load is dropped at the instant the aeroplane cuts the y-axis. Since (x,y)
yO
the horizontal translation is uniform, the position of the load at any
time t, is given y
gt 2 X
x = v0t, y = y0 - .
2
These two equations are called the parametric equations of the trajectory because the two
variables x and y have been expressed in terms of the third variable ‘t’ (parameter)
i.e. two equations x = (t), y = (t)
where t assumes values that lie in a given interval (t1, t2).
dy dy / dt (d / dt) '(t)
Then .
dx dx / dt (d / dt) '(t)
Illustration 6. The function y of x is given by, x = a cos t, y = a sin t. Find the derivative of y
w.r. t. x.
dy (a sint)' cos t
Solution: cot t .
dx (a cost)' sin t
dy
If we want to compute at a particular t, say t = , then
dx 4
dy
dx = cot 1 .
x / 4 4
Illustration 7. Find the slope of the tangent to the cycloid x = a(t-sint), y = a(1-cost), 0 t 2.
The acceleration 'a' of a particle is the second derivative of the distance 's' (given as a function of
time).
ds d2 s dv
i.e., if s = f(t) then v = f '(t) and a = 2 f ''(t) .
dt dt dt
d3y
Illustration 8. Find at x = /4, where y = sin2x.
dx 3
dy d2 y
Solution: 2 sin x cos x = sin2x 2 cos 2x .
dx dx2
d3 y
4 sin2x
dx3
d3 y
3 4
dx x / 4
Exercise 2.
dy
i) Find if:
dx
(a) x = a cos3t, y = b sin3t (b) x = at2 , y= 2at.
d2y
II) Find if (a) y = enx, (b) y = ln x2
dx 2
dy
iii) Find if (a) x = y ln (xy), (b) y = cos(x + y).
dx
dy x ....
iv) Find if y = xx .
dx
P-2022-CBSE-P1-MATHEMATICS-BASIC CALCULUS
9
Behaviour of Functions
The study of the flight of a shell in empty space yields a formula that gives the dependence of the
v 20 sin 2
range R upon the angle of elevation and the initial velocity v0 i.e. R= , where g is the
g
acceleration due to gravity. We can now determine at what angle , the range R will be greatest
or least, and what must be the condition for the range to increase as the angle of elevation is
increased.
In the oscillations of a load on a spring (of a railway car or automobile),the deviation y of the load
from the position of equilibrium is given by y = e-kt(A cos wt + B sin wt) where k, A, B, w (having
definite meaning) are considered constant. This formula gives the information as to what values
of the time t yield the maximum or minimum deviation and for what values of t, y will increase with
increasing t.
Illustration 9. Find the intervals of increase and decrease of the function y =x4.
To Find the Points of Maxima and Minima, We Resort to Either of the Following Tests:
A) First Derivative Test:
Suppose x = x0 is a critical point i.e. f (x0) = 0.
If f(x) changes sign from positive to negative in the neighborhood of x = x0
Local maximum at 'x0'.
If f(x) changes sign from negative to positive in the neighborhood of x = x0
Local minimum at 'x0'.
P-2022-CBSE-P1-MATHEMATICS-BASIC CALCULUS
11
3
Illustration 12. Determine the maximum and minimum values of the function y = x 3x + 3 on the
interval [ 3, 3/2].
2
Solution: For the given function, y = 3x 3.
2
For the critical points, 3x 3 = 0 x = 1.
Then y = 6x > 0 at x = 1,
<0 at x= 1.
Hence there is a local maximum at x = 1 at which y = 1 + 3 + 3 = 5.
Also there is a local minimum at x = 1, y = 1 3 + 3 = 1.
Now at x = 3, y = 27 + 9 + 3 = 15
and at x = 3/2, y = 15/8.
Hence the minimum value of the given function is 15 at x = 3 and the
maximum value is 5 at x = 1.
Exercise 3.
i) Find the intervals of increase and decrease of the following functions
(a) y = 4x3 - 21x2 + 18x +20, (b) y = sinx , 0 x .
ii) Find the points of local maxima and minima of the following functions:
x3
(a) y = 2x2 3x 1 , (b) y = (x 2) (x 3).
3
iii) Find the maximum and minimum values of x + sin 2x in [0, ].
iv) Find the maximum and minimum values of sinx + cosx in [0, 2].
INTEGRAL CALCULUS
The Anti derivative of a Function:
A function F(x) is called the antiderivative of the function f(x) if f(x) = F'(x).
x3 x3
2
For example, the antiderivative of the function f(x) = x is , as = x2. The functions
3 3
x3 x3 2 x3
2 and 1 are also antiderivatives of f(x) = x . Infact, C , where C is an arbitrary
3 3 3
constant, is the antiderivative of x2 . So if a function f(x) possesses an anti-derivative F(x), then it
possesses infinitely many antiderivatives, all of them being contained in the expression F(x) + C,
where C is a constant.
If the function F(x) is an antiderivative of f(x), then the expression F(x) + C is called the indefinite
integral of the function f(x) and is denoted by the symbol f(x)dx. Thus, by definition,
f(x) dx = F(x) + C, if F'(x) = f(x). The process of finding the antiderivative of a function f(x) is
called integration. Two different integrals of a given function differ by a constant.
sin x dx = cosx + c
dx
sin1
x x
c or cos1 c
a2 x 2 a a
cos x dx = sinx + c dx
2
sin1 x c or cos1 x c
sec x dx = tanx + c 1 x 2
2 dx
cos ec x dx = cotx + c 2
sec 1 x c or cos ec 1x c
x x 1
1
x1/ 21 2
x2 + c = . x3/2 + c,
e.g (i)
dx =
1
1
3
2
1 x 2 1 1
(ii)
2 dx = +c= x2 dx =
+ c.
x 2 1 x
The following points are to be noted:
1
x
dx = log x + c if x is positive
The only legitimate conclusion is that they differ by some constant. In fact
sin-1x – (-cos –1 x ) = sin –1x + cos –1 x = /2 .
If a is a constant, then
a f(x) dx = a
f(x) dx.
[ f(x) g(x)] dx, = f(x) dx g(x) dx.
2 2
Solution: (i) (a0 + a1x +a2x )dx = a0 dx + a1 x dx + a2 x dx
x2 3
= a0 x + a1 + a2 x + c.
2 3
2 x 1 x
(ii) cos x e dx = cos x dx + 2
x x dx
e dx
P-2022-CBSE-P1-MATHEMATICS-BASIC CALCULUS
13
2 2
x x 1 1 1
(iii) 1 x 2
dx = 1 x2
dx = 1 1 x 2 dx
dx
= dx 1 x 2
= x tan-1 x + c.
x4 x4 1 1 2 1
(iv) x 2
1
dx = 1 x 2
dx = x 1 dx
1 x 2
dx x3
= x2 dx
dx + 1 x 2
= x + tan –1 x + c.
3
Methods of Integration:
1. Integration by substitution
This method consists of expressing the integral f(x) dx, where x is the independent variable,
in terms of another integral where some other variable, say 't', is the independent variable; x and t
being connected by the relation x = (t). i.e , f(x)dx = f[φ(t)] φ '(t) dt.
This method is useful only when a relation x = (t) can be so selected that the new integrand
dx
f(x) is of a form whose integral is known.
dt
dx 1 1 bx k
Note: a2 bx k
2
b
sin
a
+c
2. Integration by parts
d du dv
If u, v be two function of x, we have uv v u .
dx dx dx
Integrating both sides, we get
dv du dv du
uv = u dx
dx v
dx
dx
u dx .dx = uv v. dx .dx .
dv
Let u = f(x) and (x)
dx
df
f(x)φ(x)dx f(x) φ(x)dx dx (x)dx dx .
Integral of the product of two functions = first function integral of second integral of
(derivative of first integral of second).
= x sin1 x + 1 x2 c .
Exercise 4.
Integrate the following functions w.r.t x :
1/3 1/2 3/5 2
i) (a) x +x +x (b) 5 cos x 3 sin x + ,
cos 2 x
3x2
ii) (a) (b) sin4x cos x
1 x3
2 1
iii) (a) x sinx (b)
x log x
iv) (a) x2 . ex (b) lnx
P-2022-CBSE-P1-MATHEMATICS-BASIC CALCULUS
15
Definite Integral
The difference in the values of an integral of a function f(x) for two assigned values of the
independent variable x, say a, b, is called the definite integral of f(x) over the interval (a,b) and is
b b
denoted by
a
f(x) dx. Thus f(x) dx = F(b) – F(a), where F(x) is the anti derivative of f(x).
a
Or,
b
we write f(x) dx= F(x) ba = F(b) F(a). a is called the lower limit and b, the upper limit of
a
integration.
Note:
b a
a
f(x) dx = f(x) dx
b
b c b
a
f(x) dx = a
f(x) dx + f(x) dx, where c is any point inside or outside the interval (a, b).
c
b
Geometrically, the definite integral f(x) dx denotes the area bounded by the curve
a
i.e., W = F .dx ; where F is the force applied to the body.
a
Illustration 17. Find the area of the triangle bounded by the sides y = x, y = 0 and x = 2.
/2
cos x
Illustration 18. Evaluate 1 sin
0
2
x
dx .
The order of a differential equation is the order of the highest derivative occurring in it. A solution
or integral of a differential equation is a relationship between the dependent and the independent
variables, not involving the differential coefficients such that this relation and the derivatives
obtained from it satisfy the given differential equation. This also implies that a differential equation
can be derived from its solution by the process of differentiation and elimination.
Illustration 19. Show that y = A cosx + B sinx is a solution of the differential equation
d2y
y 0.
dx 2
dy d2 y
Solution: We have = A sinx + B cosx and A cos x B sin x y
dx dx2
d2 y
y 0.
dx2
1 1
log(x 2 1) log(1 y 2 ) c
2 2
or (x2 + 1) (y2 + 1) = e2c = a, is the required solution containing arbitrary
constant 'a'.
P-2022-CBSE-P1-MATHEMATICS-BASIC CALCULUS
17
Exercise 5.
i) Evaluate the following:
/2 1 4
2 1 x
(a)
0
sin x cos x dx , (b) sin
0
x dx , (c) e
1
dx .
x
ii) Find the area enclosed by the curve y = e , the x-axis and the two ordinates x= 1
and x = 1.
dy dy
iii) Solve (a) = cot x .coty, (b) ln = x+ y.
dx dx
iv) Solve differential equation, sec2x tany dx + sec2y tanx dy = 0.
Exercise 1:
(i). (a) 21x5/2 + 10 x3/2 1 (b) (1/x). sec2 (ln(x)
(c) 2x/[(x2 +1) ] (d) (sinx)x [ln (sinx) + x cotx]
1
(ii) (a) 2 (b)
2
ax b xx x
(iii) (a) (b) e . x . (1 + log x)
2
ax 2bx c
2 cosx
(c) sin(cotx) cosec x (d) (sinx) {cotx cosx sinx ln(sinx)}
Exercise 2:
b 1
(i). (a) tan t (b)
a t
2
(ii). (a) n2 enx (b)
x2
xy sin x y
(iii). (a) (b)
x 1 ln xy 1 sin x y
y2
(iv)
x 1 ln y
Exercise 3:
1 1
(i). (a) Decreasing in ,3 , Increasing in , (3,)
2 2
π
(b) Increasing in [0, /2], decreasing in ,
2
(ii). Maxima Minima
(a) x = 1 x=3
(b) x = 5/2
(iii) 0, (iv) 2, 1
Exercise 4:
3 4/3 2 3/2 5 8/5
(i) (a) x x x c (b) 5 sin x + 3 cos x + 2 tan x + c
4 3 8
3 sin5 x
(ii). (a) log |(1+x )| + c (b) +c
5
(iii). (a) x2 cosx + 2x sinx + 2 cosx + c (b) ln |ln x| + c
(iv). (a) (x2 2x + 2)ex + c (b) x(lnx 1) + c
Exercise 5:
1 2
(i). (a) (b) 1 (c) 2e
3 2
1
(ii). e
e
(iii). (a) sin x = c sec y (b) ex + e–y = c
(iv) tanx tany = c
P-2022-CBSE-P1-MATHEMATICS-BASIC CALCULUS
19
SOLVED PROBLEMS
Subjective:
Level – 0
dy
Problem 1. If y x sin x cos x. , find .
dx
1
= x 1/ 2 sinxcosx x cos x. cos x x sinx -sinx
2
1 1 1
= x 1/ 2
2 2
sin 2x x cos2 x sin2 x =
4 x
sin2x x.cos2x .
x3
Problem 2. Differentiate w.r.t. x .
cos x
dy
Problem 3. If x = 2 ln cot t and y = tan t + cot t, find .
dx
dy dy/dt
Solution: We have . . . (1)
dx dx/dt
dx cosec 2 t 2 4
where 2
dt cot t cost sin t sin 2t
dy sin2 t cos2 t cos2t
and sec 2 t cosec 2 t 2 2
4.
dt sin t cos t sin2 2t
dy 4cos2t sin2t cos2t
Hence cot. 2t .
dx sin2 2t 4 sin2t
1/2 3/2
Problem 4. Evaluate 3x (1 + x ) dx.
dy
Solution: For critical points =0
dx
1.(x 1) 1(x 1)
0
(x 1)2
2
0
(x 1)2
2
No such value of x exists. Infact > 0 in the given interval.
x 12
That means y is entirely increasing in [0, 4].
Hence there is no critical point in the interval 0 x 4. At x = 0, y = 1, and at
x = 4, y = 3/5.
Hence the function is minimum at x= 0, with y = 1 and is maximum at x = 4 with
y = 3/5.
Problem 6. Find the maximum and the minimum values of f (x) = 2x3 –24x + 107 in the
interval [1, 3].
dy
Problem 7. Solve e x y x 2 e y .
dx
dy e x x 2 y 2 x
Solution: We write as e dy = (x + e )dx.
dx e y e y
x3
Integrating both sides, we get ey = e x c which is the solution of the above
3
differential equation, where c is an arbitrary constant.
2
Problem 8. Evaluate x dx .
1
2 0 2
Solution: Let I = x dx = x dx x dx .
1 1 0
b c b
By property, f x dx f x dx f x dx , so that
a a c
P-2022-CBSE-P1-MATHEMATICS-BASIC CALCULUS
21
0 2
I= x dx x dx (since |x| = −x for x < 0 and |x| = x for x 0)
1 0
0 2
x2 x2 1 4 5
=−
2 = − 0 0 .
1 2 0 2 2 2
dy
Problem 9. Find , when x = a ( + sin ), y = a (1 + sin )
dx
dx dy
Solution: We have = a (1 + cos ) and = a cos
d d
dy
dy d a cos cos
.
dx dx a(1 cos ) 1 cos
d
dy
Problem 10. Find , when x = et(sin t + cot t), y = et log t.
dx
dx
Solution: Here = et (cos t sin t) + et (sin t + cos t) = 2et. cos t,
dt
dy 1 1
and et log t et et log t .
dt t t
Level – II
sin 3 x cos 3 x
Problem 11. Evaluate sin 2 x cos 2 x
dx.
=
tan x sec x
dx + cot x cos ecx dx = sec x – cosec x + c.
x
cos e x dx.
Problem 12. Evaluate e
dx
Solution: Let ex = t ex = 1 dx = e– x. dt
dt
x
cos e x dx = x x x
e e .cos e .e dt = cos t dt
x
= sin t + c = sin e +c.
1
2 2x
Problem 13. Evaluate x e
0
dx .
dy sin 2 ( a y )
Problem 15. If sin y = x sin (a + y), prove that .
dx sin a
sin y
Solution: The given equation can be written as x = so that
sin(a y)
dx sin(a y)cos y sin y cos(a y)
dy sin2 (a y)
sin(a y y) sin a
= 2
2
sin (a y) sin (a y)
2
dy sin (a y)
.
dx sin a
Alternate: Differentiate the given equation with respect to x. We get
dy dy
cos y = sin (a + y) + x cos (a + y)
dx dx
dy sin(a y)
dx cos y x cos(a y)
sin2 (a y) sin2 (a y)
= .
cos y sin(a y) sin y cos(a y) sin a
dx
Problem 16. Evaluate I sin 2
x.cos2 x
.
sec
2
Hence I = x cos ec 2 x dx = tanx – cotx + c.
dy et log t 1
dy t t log t 1
Hence dt .
dx dx 2e t cos t 2t cos t
dt
P-2022-CBSE-P1-MATHEMATICS-BASIC CALCULUS
23
Objective:
Level – 0
True / False
1 t2 2at dy a(t 2 1)
Problem 1. If x and y , then
1 t2 1 t2 dx 2t
Solution: True
1 t2 2at
x 2
and y
1 t 1 t2
Differentiating with respect to t, we get
dx (1 t 2 )(0 2t) (1 t 2 )(0 2t) 4t
2 2
dt (1 t ) (1 t 2 )2
dy (1 t 2 )2a 2at(2t) 2a(1 t 2 )
and
dt (1 t 2 )2 (1 t 2 )2
dy dy / dt a(1 t 2 ) dy a(t 2 1)
; .
dx dx / dt 2t dx 2t
t dy
Problem 2. If x a cos t logtan ,y a sin t, then tan t
2 dx
Solution: True
t
Given that x a cos t logtan and y a sin t . Differentiating with respect to t,
2
dy
we get a cos t
dt
.....(i)
dx t 1 t
and a sin t cot sec 2
dt 2 2 2
1 cos2 t
a sin t a a cos t cot t .....(ii)
sin t sin t
dy
From (ii) and (i), we get tan t .
dx
1 4 2 dy 2 2
Problem 3. If y u ,u x 3 5 , then x (2x3 15)3
4 3 dx 27
Solution: True
dy dy du
. u3 .2x 2
dx du dx
3
2 2 2
x3 5 .2x 2 x (2x3 15)3 .
3 27
Solution: True
f(x) log x ; f[log x] loglog x
1 d 1
f '[log x] . log x .
log x dx xlog x
5x 1 dy 5x 1 dy 5x 1
Problem 5. If y f 2 and f '(x) cos x , then dx cos
10x 3 10x2 3 dx 10x2 3
Solution: True
5x 1
Suppose that t , so y f(t)
10x 2 3
dy dt
f (t). [Since f '(x) cos x ]
dx dx
dy 5x 1 d 5x 1
cos .
dx 10x 2 3 dx 10x 2 3
dy
Problem 6. If x 2 cos t cos 2t , y 2sin t sin 2t , then at t , _______
4 dx
Solution: 2 1
dx dy
2sin t 2 sin2t and 2cos t 2cos 2t
dt dt
dy cos t cos 2t
dx sin 2t sin t
dy
Put t , we have
4 dx t /4
cos / 4 cos / 2
2 1.
sin / 2 sin / 4
dy
Problem 7. If x 3 8xy y3 64 ,then _______
dx
3x 2 8y
Solution:
8x 3y 2
dy dy
x 3 8xy y3 64 3x2 8 y x 3y2 0
dx dx
dy 3x 2 8y
.
dx 8x 3y 2
dy
Problem 8. If ax 2 2hxy by 2 2gx 2fy c 0 , then _______
dx
P-2022-CBSE-P1-MATHEMATICS-BASIC CALCULUS
25
ax hy g
Solution:
hx by f
ax 2 2hxy by 2 2gx 2fy c 0
Differentiating w.r.t. x of y, we get
dy dy dy
2ax 2h y x 2by 2g 2f 0
dx dx dx
dy
(2hx 2by 2f ) (2ax 2hy 2g)
dx
dy (ax hy g)
or .
dx (hx by f )
Level – I
dy
Problem 9. If cos(x y) y sin x, then
dx
sin(x y) y cos x sin(x y) y cos x
(A) (B)
sin x sin x y) sin x sin(x y)
sin(x y) y cos x
(C) (D) None of these
sin x sin x y)
Solution: A
cos(x y) (y sin x)
dy dy
sin(x y) 1 y cos x sin x dx
dx
dy y cos x sin(x y)
.
dx sin(x y) sin x
dy
Problem 10. x 1 y y 1 x 0 , then
dx
(A) 1 x (B) (1 x)2
(C) (1 x)1 (D) (1 x)2
Solution: D
x 1 y y 1 x 0 x 2 (1 y) y 2 (1 x)
d2 s
2 8 x y xy 0, x y
dt t 1/2
dy 1
.
dx (1 x)2
Problem 12. Let f(x) e x , g(x) sin1 x and h(x) f(g(x)), then h'(x) / h(x)
1
(A) esin x (B) 1/ 1 x 2
(C) sin1 x (D) 1/(1 x 2 )
Solution: B
f(x) e x and g(x) sin1 x and h(x) f(g(x))
1
h(x) = f(sin1 x) esin x
1
h(x) esin x
1 1 h(x) 1
h(x) esin x
. .
1 x 2 h(x) 1 x2
1
Problem 13. If f(x) , then the derivative of the composite function f[f {f(x)}] is equal to
1 x
1
(A) 0 (B)
2
(C) 1 (D) 2
Solution: C
1 1 x
f(x) f {f(x)}
1 x x
x
f[f {f(x)}] x
x 1 x
Derivative of f[f {f(x)}] 1.
Level – II
1 1 dy
Problem 14. If x 2 y 2 t , x 4 y 4 t 2 2 , then x 3 y
t t dx
(A) 1 (B) 2
(C) 3 (D) 4
Solution: A
2
1
x 4 y 4 t 2 (x2 y 2 )2 2
t
1
x 2 y 2 1 y 2 2
x
dy 2 dy
Differentiating, we get 2y 3 or x 3 y 1.
dx x dx
2t 2t dy
Problem 15. If tan y 2
and sin x 2
, then
1 t 1 t dx
2 1
(A) (B)
1 t2 1 t2
(C) 1 (D) 2
P-2022-CBSE-P1-MATHEMATICS-BASIC CALCULUS
27
Solution: C
2t
tan y .....(i)
1 t2
2t
and sin x .....(ii)
1 t2
From (i), differentiating w.r.t. t of y, we get,
dy 1 cos( sin x cos x ) dy 2(1 t 2 ) 1
(cos x sin x) and .
dx 2 sin x cos x dt (1 t ) (1 tan2 y)
2 2
dy 2(1 t 2 ) 1 2
or . .....(iii)
dt (1 t 2 )2 2t 2 1 t 2
1 2
1 t
and from (ii), differentiating w.r.t. t of x, we get
dx 2(1 t 2 )
cos x
dt (1 t 2 )2
dx 2(1 t 2 ) 1 2
or .....(iv)
dt (1 t 2 )2 (2t) 2 1 t2
1
(1 t 2 )2
dy
Hence 1.
dx
1 t2 2t dy
Problem 16. If x 2
and y 2
, then
1 t 1 t dx
y y
(A) (B)
x x
x x
(C) (D)
y y
Solution: C
1 t2 2t
x and y
1 t2 1 t2
Put t tan in both the equations, we get
1 tan2 2 tan
x 2
cos 2 and y sin2 .
1 tan 1 tan2
Differentiating both the equations, we get
dx dy
2sin2 and 2cos 2.
d d
dy cos2 x
Therefore .
dx sin 2 y
d2 y
Problem 17. If x at 2 ,y 2at , then
dx2
1 1
(A) (B)
t2 2at 3
1 1
(C) 3 (D)
t 2at 3
Solution: D
dy
Problem 18. If sin y x sin(a y), then
dx
sin2 (a y) sin2 (a y)
(A) (B)
sin(a 2y) sin(a 2y)
sin2 (a y) sin2 (a y)
(C) (D)
sina cosa
Solution: C
sin y
sin y x sin(a y) x
sin(a y)
dy dy
cos y. .sin(a y) sin y cos(a y)
1 dx dx
sin2 (a y)
dy
.sin(a y y)
dy sin2 (a y)
dx 2 .
sin (a y) dx sina
dy
Problem 19. If tan(x y) tan(x y) 1, then
dx
sec 2 (x y) sec 2 (x y) sec 2 (x y) sec 2 (x y)
(A) (B)
sec 2 (x y) sec 2 (x y) sec 2 (x y) sec 2 (x y)
sec 2 (x y) sec 2 (x y)
(C) (D) None of these
sec 2 (x y) sec 2 (x y)
Solution: B
tan(x y) tan(x y) 1
Differentiating w.r.t. x of y, we get
dy dy
sec 2 (x y) 1 sec 2 (x y) 1 0
dx dx
dy sec 2 (x y) sec 2 (x y)
.
dx sec 2 (x y) sec 2 (x y)
dy
Problem 20. If y sec x tan x x2 y 0 , then =
dx
2xy sec 2 x y sec x tan x 2xy sec 2 x sec x tan x
(A) (B)
x 2 sec x x 2 sec x
2xy sec 2 x y sec x tan x
(C) (D) None of these
x 2 sec x
Solution: C
P-2022-CBSE-P1-MATHEMATICS-BASIC CALCULUS
29
2
y sec x tan x x y 0
dy dy
sec x y sec x tan x sec 2 x 2xy x 2 0
dx dx
dy 2xy sec 2 x y sec x tan x
.
dx x2 sec x
x dy
Problem 21. If sin(xy) x2 y, then
y dx
y[2xy y 2 cos(xy) 1] [2xy y 2 cos(xy) 1]
(A) (B)
xy 2 cos(xy) y 2 x xy 2 cos(xy) y 2 x
y[2xy y 2 cos(xy) 1]
(C) (D) None of these
xy 2 cos(xy) y 2 x
Solution: A
x
sin(xy) x2 y
y
Differentiating both sides,
d 1 dy 1 dy
cos(xy) (xy) x 2 2x
dx y dx y dx
x dy 1
[x cos(xy) 2 1] 2x y cos(xy)
y dx y
dy 2xy 2 y y3 cos(xy)
.
dx xy 2 cos(xy) x y 2
dy
Problem 22. If sin2 x 2cos y xy 0 , then
dx
y 2sin x y sin 2x
(A) (B)
2 sin y x 2 sin y x
y 2sin x
(C) (D) None of these
sin y x
Solution: B
sin2 x 2cos y xy 0
dy dy
2sin x cos x 2sin y yx 0
dx dx
dy y sin2x
.
dx 2sin y x
Subjective:
Level – I
1 x2 dy
1. If y = tan1 2 , then find at x = 1.
1 x dx
Level – II
2t 2t dy
5. If tany = 2
and sinx = 2
, then find .
1 t 1 t dx
Objective:
1. The differential equation whose solution is y = c1sinx + c2cosx, where c1, c2 one arbitrary
is
d2 y dy
(A) 2
y0 (B) y2 0
dx dx
d2 y
(C) y0 (D) none of these
dx2
2. lim
sin cos2 x equals to
2
x 0 x
(A) (B)
(C) (D) 1
2
2 ab x
3. If y tan1 tan , where a b 0 then
ab 2
a2 b2
dy 1 d2 y b sin x
(A) (B) 2
dx a bcos x dx (a b cos x)2
dy 1 d2 y b sin x
(C) (D) .
dx a bcos x dx2 (a bcos x)2
P-2022-CBSE-P1-MATHEMATICS-BASIC CALCULUS
31
4. The derivative of f(tanx) w.r.t. g(secx) at x , where f(1) = 2, g( 2) 4 , is
4
1
(A) (B) 2
2
(C) 1 (D) none of these.
1 cos2 x 1
5. The value of lim
x 1 x 1
(A) exists and it equals 2
(B) exists and it equals 2
(C) does not exist because (x – 1) 0
(D) does not exist because left hand limit is not equal to right hand limit.
nk sin2 n!
6. The value of lim ,0 k 1 , is equal to
n n2
(A) (B) 1
(C) 0 (D) None of these
x tan2x 2x tan x
7. lim 2
is
x 0
1 cos2x
(A) 2 (B) 2
1 1
(C) (D)
2 2
Numerical Based
1
x sin , x0
1. If f(x) x , then lim f(x)
x 0
0, x0
x 3 cot x
2. If lim k then k =
x 0 1 cos x
x(e x 1)
3. If lim then =
x 0 1 cos x
x
4. If f(x) for x R, then f '(0)
1 | x |
Subjective:
Level – 0
1 3x 1 3x
1. Evaluate : lim
x 0 x
x2
2. Evaluate : lim
x 2 3x 2 x 2
1 x x2 1
3. Evaluate : lim
x 0 x
tan3x 2x
4. Evaluate : lim
x 0 3x sin2 x
sin3x 7x
5. Evaluate : lim
x 0 4x sin 2x
2 sin x sin2x
6. Evaluate : lim
x 0 x3
x sin x
7. Evaluate : lim
x x cos2 x
tan x x
8. Evaluate : lim 2
x 0 x tan x
9.
Differentiate w.r.t. x, y = e x x 3 x
10. Differentiate w.r.t. x, y = (tanx + secx) (cotx + cosecx)
x2 sin x
11. Differentiate w.r.t. x, y =
1 sin x
x tan x
12. Differentiate w.r.t. x, y =
sec x tan x
sin x x cos x
13. Differentiate w.r.t. x, y =
x sin x cos x
P-2022-CBSE-P1-MATHEMATICS-BASIC CALCULUS
33
15. Find the interval of increase and decrease of the function
f (x) = 6x4 8x3 12x2 + 24x 5 = 0.
1 1 dy
16. If x = t + , y = t , find in term of x and y.
t t dx
sin x
17. Differentiate with respect to x, y =
x 1
dy
19. Find if x = a cos2 t, y = a sin2 t
dx
dy
20. If x = a( - sin), y = a(1 cos), then find .
dx
dy
21. If y = xsinx, find .
dx
d2 y
22. If x = a(t + sint) and y = a(1 – cost), find .
dx 2
x2 5x 6
23. Evaluate : lim
x 1 x2 x 1
x3 8
25. Evaluate : lim
x 2 x 2
dy
2. Find , if
dx
2 3cos x 2 x
(i) y (ii) y = x e sinx
sin x
x
e sin x
3. Evaluate : dx
2 x
dy
4. Find
dx
if y = tan cos e
1 x 2
x4
5. If eln f x dx + c, then find f (x).
4
1 1 dy
6. If x2 + y2 = t + and x4 + y4 = t2 + 2 , then find .
t t dx
dy 1 y2
7. If 1 x2 1 y 2 a x y , then prove that .
dx 1 x2
2x 9
8. Evaluate x 2 9x 30 dx .
log x 1 log x
9. Evaluate dx .
x x 1
/4
2
11. Evaluate sin
0
x dx .
P-2022-CBSE-P1-MATHEMATICS-BASIC CALCULUS
35
Level – II
dy
1. The slope at any point of a curve y = f(x) is given by = 3x2 + x and it passes through
dx
(1, 0).Then find the equation of curve.
dy
2. Find , when x and y are given by (i) x = acos3t, y = b sin3t,
dx
(ii) x = (v0 cos) t, y = (v0 sin) t – gt2/2, where v 0, and g are constants.
x 2 5x 4
5. (i) (3x2 x 4) (x 1) (2x 3) > 0 (ii) 0
3 x 3 x 2 4
2
(iii)
x
x 1
(iv)
x 2
x 0
x 1 x 4 5
x 2 x 1
sec 2 (loge x) x 1 cos x
6. Evaluate: (i) dx (ii) e tan e x sec e x dx (iii) 3
dx
x x sin x
7. Integrate the following functions:
x x 1 x log x 2
(i) sin x cos x (ii) (iii)
(x 2 1)1/ 3 2x
1 1
(iv) log x (v)
x x x 1
8. Find the value of the following
/2 /2
1 2
(i)
0
1 cos x
dx (ii)
1 cos x
0
sin x dx
dy
11. Find (i) y = xx (ii) xy = y x
dx
a
ax
12. Evaluate dx , a > 0.
ax
0
Objective:
Level – I
/2
cos x
1.
0
1 sin x
dx is equal to
(A) 2 2 1 (B)
2 1
(C) 2 2 1 (D) none of these
x sin x 1
2. lim is equal to
x x cos x
(A) 1 (B) 2
(C) 3 (D) 4
2
2 ax
3
3. The value of the integral bx c dx depends on the
(A) value of b (B) value of c
(C) value of a (D) value of a and b
3
2
4. x sin x3 e x dx equals to
x3
3
1 x3 sin x cos x
3
(A) e sin x 3
cos x 3
(B)
3
e
2
3
(C) e x sin x3 (D) None of these
x x
a a d2 y
5. If y = e e a and = y, then a equals
2
dx 2
(A) 2 (B) 1
1 1
(C) (D)
2 2
x dy
6. If y = 25 , than equals to
dx
x x
(A) 25 ln8 (B) 25 5 x ln8
x
(C) 25 5 x ln5ln2 (D) None
2 dy
7. If y = sin (cox3x), then equals
dx
(A) 3sin(3x) sin(2cos3x) (B) -3sin3x sin(2cos3x)
(C) -3sincos(6x) (D) 3sin(cos6x)
dy
8. If y = f(e-x) and f(x) = lnx then at x = ln2 is
dx
ln2
(A) 2ln2 (B)
2
(C) ln2 (D) none of these
P-2022-CBSE-P1-MATHEMATICS-BASIC CALCULUS
37
dy
9. If y = (x a) (x b), then the value of x for which 0 is
dx
a b
(A) (B)
2 2
ab ab
(C) (D)
2 2
3x 3
10. a dx is equal to
a3x 3 a3x 3
(A) c (B) c
log a 3 log a
(C) a3x 3 log a c (D) 3a3x 3 log a c
/2
4 cos x
11. The value of dx is equal to
0
cos sin x
(A) 1 (B) 2
(C) 1 (D) 0
/ 3
1
12. If dx = k, then k is equal to
/4
sin x cos2 x
2
2
(A) (B) 3
3
(C) 2 (D) 5
dy
13. The Solution of the differential equation loge 3x 5y is
dx
5y 3x
(A) 5e = 3e + c (B) 3e5y – 5e3x = c
5y -3x
(C) 5e = - 3e + c (D) 3e5y – 5e-3x = c
14. The equation of a curve whose slope at any point is thrice its abscissa and which passes
through
(-1, -3) is
(A) y = x2 – 4 (B) 3y = 2(x2 – 4)
2
(C) 2y = 3(x – 3) (D) None of these
ex 1 x
16. cos (xe ) dx is equal to
2 x
(A) cot xe + c x
(B) tan xe x +c
(C) tan e + c x
(D) none of these
18. 0 4 x tan x2 dx is equal to
1
(A) log2 (B) log2
2
1
(C) log2 (D) None of these
4
19. A differentiable function f is defined for all x > 0 and satisfies f(x) = x4 for all x > 0, then f
(16) is equal to
(A) 64 (B) 16
(C) 8 (D) None
1 ln x
20. 4 x ln x 2 dx equals
1 1
(A) ln 4 xln x2 c (B) ln 4 xln x 2 c
2 4
1 1
(C) ln 2 x ln x2 c (D) ln 2 x ln x2 c
2 2
21. Let f(x) = ln|x 1|, x 1, then the value of f
2
(A) is 2 (B) is 2
(C) is 0 (D) does not exist
dx
22. If x 1 x 2 A ln x 1 Bln x 2 c , then
(A) A + B = 0 (B) AB = 1
A
(C) 2 (D) A B = 0
B
Level – II
x 1
1. 2x3 / 2 dx equals to
x2 1
(A) x1/ 2 x 1/ 2 c (B) c
x3 / 2 x1/ 2
2
(C)
x2 2x 1
c (D)
x 1/ 2
1 3
c
x3 / 2 x1/ 2 x1/ 2
P-2022-CBSE-P1-MATHEMATICS-BASIC CALCULUS
39
1 x 1
2. 2 x 1 2 x2 1 2 x2 1 dx = Alog (x 1) + Blog (x2 + 1) + C tan-1x + K then,
e e
1 1
(A) A (B) B
2 4
1 1
(C) C (D) C
2 2
log log, x
dy
If y x
log x
3. , then is
dx
y y
(A)
ln x x 1 2ln x ln ln x (B) log x loglog x 2log log x 1
x x
y 2 y log y
(C) ln x 2ln ln x (D) 2log log x 1 .
x ln x x log x
cos ec 2 x sin x
4. dx 2 dx equals
2
cot x 1 1 2 sin2 x
cos2x
(A) cos ec 2 x 2 dx (B) dx
2 sin x
cos2x
(C) dx (D) cos ec 2 x 1dx
sin x
2t 2t dy
7. If tany = 2
and sinx = 2
, then is equal to
1 t 1 t dx
(A) 1 (B) sin2 + cos2
2 2
(C) sec tan (D) none of these
dy
8. If y = (sin x)x , then is
dx
(A) (sin x)x (x cot x + ln sin x) (B) y(x cot x + ln sin x)
x1
(C) (sin x) (x cos x + sinx ln sin x) (D) none of these
x x dy
9. If y e e , then is equal to
dx
x x
e e e x
e x
(A) (B)
2 x 2x
1 1
(C) y2 4 (D) y2 4
2 x 2 x
13. f(x) = |x2 – 3|x| + 2|. Then which of the following is/are true?
(A) f(x) = 2x – 3 for x (0, 1) (2, )
(B) f(x) = 2x + 3 for x (–, –2) (–1, 0)
(C) f(x) = –2x – 3 for x (–2, –1)
(D) None of these.
x 4 x2 1 dy
14. If y 2
and ax b , then the value of a – b is
x 3x 1 dx
(A) cot (B) cot
8 12
5 5
(C) tan (D) tan .
12 8
x 2 x 1
15. Let f x x . Then
x 1 1
(A) f(10) = 1 (B) f(3/2) = – 1
(C) domain of f(x) is x 1 (D) range of f(x) is (–2, –1] (2, )
Assertion Reason
Questions (9 – 10) contain STATEMENT-1 (Assertion) and STATEMENT-2 (Reason).
Option (A) if both the statements are TRUE and STATEMENT-2 is the correct explanation of
STATEMENT-1
Option (B) if both the statements are TRUE but STATEMENT-2 is NOT the correct
explanation of STATEMENT- 1
Option (C) if STATEMENT-1 is TRUE and STATEMENT-2 is FALSE.
Option (D) if STATEMENT-1 is FALSE and STATEMENT-2 is TRUE.
P-2022-CBSE-P1-MATHEMATICS-BASIC CALCULUS
41
17. Let f(x) = sinx x [0, 2]
STATEMENT -1 : f(x) is invertible in the given domain.
because
STATEMENT -2 : f(x) is many one-onto.
COMPREHENSIONS
(I) Read the following write up carefully and answer the following questions:
x
Let f x
1 x
d3 y
Consider the differential equation 0
dx3
Matrix Match
23. Match the following:
Column I Column II
1 (p) tanx – cotx + c
(A) a2 cos2 x b2 sin2 x dx
3 1
(B) sin xdx b
(q) tan1 tan x c
ab a
(C)
dx sec 2 x
sin x cos3 x (r) log tanx +
2
c
2
(D) sec x cos ec 2 x dx 1 a
(s) cot 1 cot x c
ab b
cos3 x
(t) – cosx + c
3
P-2022-CBSE-P1-MATHEMATICS-BASIC CALCULUS
43
Numerical Based
/2
26.
/2
sin x dx is equal to ______
2
27. 2 x dx
1
is equal to ______
2 3
28. If the minimum value of x + x + 1 is , then k is equal to _______
k
3 2
29. The maximum value of the function f(x) = 2x 15x + 36x 48 on the set
A = {x|x2 + 20 9x} is ________
Subjective:
Level – I
1. –1
3(ln x)2
2. cos (ln x)3
x
2x
3.
1 x4
Level – II
x 2 cos(x 2 1)
4.
sin(x 2 1)
log sin x 2 1
5. 1
Objective:
1. A 2. B 3. A 4. A
5. C 6. C 7. C
1. A, C
Numerical Based
1. 0 2. 2 3. 2 4. 1
P-2022-CBSE-P1-MATHEMATICS-BASIC CALCULUS
45
ANSWERS TO ASSIGNMENT PROBLEMS
Subjective:
Level – 0
1
1. 3 2. 8 3.
2
1 5
4. 5. 6. 1
3 3
1 1
7. 1 8. 9. e x x 3 3x 2 x
3 2 x
10. (secx cosex) (secx + tanx) (cosecx + cotx)
x2 1 x cos x x sin x 2 x x sec x sec x tan x tan x
11. 12.
1 x
2
sec x tan x
x2
13. 2
14. tan x tan y = ec = k constant.
x sin x cos x
15. For increasing 1 < x < 1, x > 1, for decreasing x < 1.
2
xy
1
dy 2 2 (x 1)cos x sin x
16. 17.
dx x y 2 2(x 1)3/2
1
2
x 2 cos( x 2 1)
18. 2
sin( x 1)
log sin x 2 1 19. 1
sin x
20. cot /2 21. x sin x cos x log x
x
1 t 2
22. sec 4 23.
4a 2 3
24. 0 25. 12
Level – I
3
1. (i) (ii) n
2
2. (i) cosecx(3cosecx 2cotx) (ii) xex(xcosx + xsinx + 2sinx)
et
3. I= (sin t cos t) + c.
2
dy x
4.
dx
sec 2 cos e 1 x2
sin e e
1 x 2 1 x 2
1 x2
.
3 y
5. f (x) = x . 6.
x
Level – II
x2 1
1. y = x3
2 2
b gt
2. (i) tan t (ii) tan sec
a v0
1 a
3. (i) c (ii) loge bx n c K
1 sin x bn
(iii) 2loge(1+x) + c
1
4. (i) ln 2 (ii) 3 2 1
2
2 3/2 3 2 2/3
7. (i)
3
sin x c (ii)
4
x 1 c
1 3 log x 2
(iii) x log x c (iv) c
6 2
2 3/2 2
(v) x 1 x3 / 2 c
3 3
9. (i) 1 (ii) 2/3 (iii)
16
dy dy y x loge y y
11. (i) x x 1 loge x (ii)
dx dx x y loge x x
12. a 1
2
P-2022-CBSE-P1-MATHEMATICS-BASIC CALCULUS
47
Objective:
Level – I
1. A 2. A 3. B 4. B
5. B 6. C 7. B 8. C
9. D 10. B 11. A 12. A
13. B 14. C 15. D 16. B
17. B 18. C 19. D 20. A
21. A 22. A
Level – II
1. A, B, C 2. A, B, C 3. B, D 4. A, C
5. A, C, D 6. C, D 7. A, B, C 8. A, B, C
9. A, C 10. A, C, D 11. B, C, D 12. A, C
13. A, B, C 14. B, C 15. A, B, D 16. D
17. D 18. C 19. D 20. A
21. C 22. A
23. (A) (q), (s) (B) (t) (C) (r) (D) (p)
24. C
25. (I). B (II). C (III). A
26. 2 27. 5 28. 4 29. 7
LINEAR INEQUALITIES
CONTENTS SYLLABUS
Introduction 1
CBSE: Linear inequalities. Algebraic solutions of
Exercise 2 6
Answers to Exercises 7
Solved Problems 8
Assignment Problems 11
Answers to Assignments 18
Introduction
A relation of the type f(x, y) > 0, f(x, y) < 0, f(x, y) 0, or f(x, y) 0 is called an inequality or an
inequation. It is also called a constraint or a condition. If f(x, y) is linear in x and y, then the
inequality is called a linear inequality or a linear inequation. For example
3x + 2y – 4 > 0, ax + by c
are linear inequations in x and y. The solution set of a linear inequation is the set of all values of
(x, y) which satisfy the given inequation.
The above examples involve two variables, namely x and y and the inequations are called linear
inequations in two variables x and y.
The relations ax + b 0 or cx + d 0 are linear inequations in one variable x.
Note:
(i) Equal numbers may be added to (or subtracted from) both sides of an inequality without
affecting the sign of the inequality.
(ii) Both sides of an inequality can be multiplied (or divided) by the same positive number. But
when both sides are multiplied or divided by a negative number, then the sign of the inequality is
reversed.
Let us reconsider the inequation (i).
We have 19x < 200
19x 200
or
19 19
200
or x .
19
(i) When x is a natural number.
In this case the solutions are
P-2022-CBSE-P1-MATHEMATICS-LINEAR INEQUALITIES
2
1, 2, 3, ….10.
These solutions can be represented on the number line by ten points as shown
-3 -2 -1 0 1 2 3 4 5 6 7 8 9 10
–9 –8 –7 –6 –5 –4 –3 –2 –1 0 1
(ii) For real values, the solutions are all real numbers x which are less than –6.
A circle over –6 implies that –6 is not included in the solution set.
–8 –7 –6 –5 –4
2x 3 4x
Illustration 2. Solve 10 4 .
4 3
2x 3 4x 2x 3 4x
Solution: Here 10 4 or, 12 10 12 4
4 3 4 3
or, 6x 9 120 48 16x or, 6x 111 48 16x
10x 63 63
or, 10x 63 or, or, x
10 10 10
4 5 6 7
i.e. all real numbers which are less than or equal to 6.3 are solutions of the given
inequality.
P-2022-CBSE-P1-MATHEMATICS-LINEAR INEQUALITIES
3
Illustration 3. Solve for x, the following system of inequalities:
2x – 7 > 5 – x, 11 – 5x 1.
–3 –2 –1 –0 1 2 3 4 5 6
x2
Solution: Here 4x + 3 2x + 17 2x 14 or x 7.
Also, 3x – 5 –2 3x < 3 or x < 1.
x<1
–2 –1 0 1 2 3 4 5 6 7
Here x is less than one and at the same time greater than or equal to seven,
which is not possible. Hence the given system has no solution.
Exercise 1:
Solve the inequations:
i) 4x + 3 > 3x; –(x + 3) + 4 –2x + 5
4x 9 3 7x 1 7x 2
ii) x ; x
3 4 4 3 6
3x
iii) 5 x 7 3( x 3 ) ; 1 x4
2
2x 3 4x
iv) 2( 2 x 3 ) 10 6( x 2 ); 6 2 .
4 3
P-2022-CBSE-P1-MATHEMATICS-LINEAR INEQUALITIES
4
As such the values of y corresponding to x = 0, can be 0, 1, 2, 3, …., 33. Hence the
solutions of (i) are (0, 0), (0, 1), (0, 2) ,….., (0, 33).
Similarly, the solutions corresponding to x = 1, 2, 3, …, 12 are
(1, 0), (1, 1), …. , (1, 30),
(2, 0), (2, 1), …., (2, 28),
…………………………..
(11, 0), (11, 1), …, (11, 4),
(12, 0), (12, 1).
We note that the values of x and y cannot be more than 12 and 33 respectively. We also note that
some of the pairs namely (5, 20), (8, 12), and (11, 4) satisfy the equation
16x + 6y = 200 which is a part of the given inequation.
Note:
In order to identify the half plane represented by an inequation, it is sufficient to take any known
point (not lying on the line) and check whether it satisfies the given inequation or not. If it satisfies,
then the inequation represent that half plane, containing the known point, otherwise the
inequation represents the other half plane.
Illustration 5: Draw the graph of the solution region satisfied by the in-equation 3x 5y + 6 > 0.
P-2022-CBSE-P1-MATHEMATICS-LINEAR INEQUALITIES
5
For a set of linear inequations, in two variables, the solution region may be
(i) a closed region inside a polygon (bounded by straight lines),
(ii) an unbounded region (bounded partly by straight lines),
or (iii) empty.
Illustration 6. Draw the graph of the solution region satisfied by the inequalities
2x + y 4, y 2, x 0.
Illustration 7. Draw the graph of the solution region satisfied by the inequations
x + y 1, 2x + y 4, x 0, y 0.
lines
x+y = 1, (1)
2x + y = 4, (2)
(0, 1)
x = 0, (3) (2, 0)
(0, 0) (1, 0)
y = 0. (4)
The straight line (1) meets the coordinates
axes at (1, 0) and (0, 1). For (0, 0),
x + y 1 gives 0 1 which is not true.
Hence (0, 0) does not lie in the half plane represented by x + y 1. The line (2)
meets the coordinate axes in (2, 0) and (0, 4) and 0 4.Hence (0, 0) lies in the
half plane 2x + y 4. Hence the region bounded by x + y 1 and 2x + y 4
P-2022-CBSE-P1-MATHEMATICS-LINEAR INEQUALITIES
6
belongs to either the fourth quadrant, or the first quadrant or the second
quadrant. But the point belonging to x 0, y 0, all lie in the third quadrant.
x – y 1, x 0, y 3.
Y
Solution: The solution region is bounded
(0, 4)
by the straight lines y=3
(0, 3)
2x + y = 4, … (1)
3x + 3y = 1, … (2)
x – y = 1, … (3)
x = 0, … (4) (0, 1/3)
(0, 0) (1, 0) (2, 0) X
y = 3. … (5) (1/3,0)
(0, –1)
The points where the first three lines meet
the x-axis are (2, 0), (1/3, 0), (1, 0)
respectively.
The points where the first three lines meet the y-axis are (0, 4), (0, 1/3), (0, 1)
respectively. Moreover, (0, 0) belongs to the half planes 2x + y 4 and
x – y 1, y 3 but not to the half plane 3x + 3y 1.
The solution region is the shaded region. All the boundary lines are part of the
solution region and the closed region inside a polygon.
Exercise 2.
Draw the graphs of the region satisfied by the solutions of
i) 2x + 3y 3,
ii) 2x + y > 0, x + 2y 2,
iii) x – y 1, x + y 1, y > 1,
iv) 2x + 3y 6, x + y 1, 2x – y – 4 0, y 0.
P-2022-CBSE-P1-MATHEMATICS-LINEAR INEQUALITIES
7
ANSWERS TO EXERCISES
Exercise 1:
i) 3<x4
ii) 4<x<9
iii) 2x<8
iv) x
Exercise 2:
i)
(0, 1)
(3/2,0)
ii)
x+2y =2
2x+y =0
iv)
2x+3y =6
x+y =1
2xy =4
P-2022-CBSE-P1-MATHEMATICS-LINEAR INEQUALITIES
8
SOLVED PROBLEMS
The straight lines (1) and (2) meet the x=0 (4, 0)
(3, 0)
x-axis in (3, 0) and (4, 0) and for (0, 0),
x + 2y 3 0 3, which is true.
Hence (0, 0) lies in the half plane x + 2y 3. Also the lines (1) and (2) meet the
y-axis in
(0, 3/2) and (0, 3) and for (0, 0)
3x + 4y 12
0 12 which is not true. Hence (0, 0) doesn’t belong to the half plane
3x + 4y 12. Also x 0, y 1 that the solution set belongs to the first
quadrant. Moreover all the boundary lines are part of the solution. From the
shaded region, we find that there is no solution of the given system. Hence the
solution set is an empty set.
Problem 2. Find all the pairs of consecutive odd natural numbers, both of which are larger
than 10, such that their sum is less than 40.
Solution: Let x be one of the odd natural numbers so that the other one is x + 2 (being
consecutive odd numbers). Here x + 2 > x and x > 10.
Also x + x + 2 < 40 2x < 38 or x < 19.
Hence 10 < x < 19, so that the required pairs are (11, 13), (13, 15), (15, 17),
(17, 19).
1
Problem 3. Solve 3x 2 .
2
1
Solution: Since 3x 2 , we have
2
1 1 1 1
3x 2 or, 2 3x 2
2 2 2 2
1 3 1 5 1 5
or, x or, x .
3 2 3 2 2 6
Hence solution set consists of all real numbers lying between
1 5 1 5
and including and .
2 6 2 6
Problem 4. A man wants to cut three lengths from a single piece of wire of total length 91
cm. The second length is to be 3 cm longer than the shortest and the third length
is to be twice as long as the shortest. What are the possible lengths of the
shortest piece if the third length is to be at least 5 cm longer than the second.
P-2022-CBSE-P1-MATHEMATICS-LINEAR INEQUALITIES
9
Solution: Let the length of the shortest piece of wire be x so that the second and the third
lengths are x + 3 and 2x respectively. Now, it is given that third length is to be at
least 5 cm longer than the second 2x (x 3) 5
or x 8 . (1)
Also the total length is 91 cm so that
x + (x + 3) + (2x) 91 4x 88
or, x 22. (2)
From (1) and (2), we find that 8 x 22.
x 8 x 5
Problem 5. Solve, for x, the inequations (i) 1, ( ii ) 0.
x 2 x2
x8 6
Solution: (i) The given inequation is rearranged as 1 0 or 0
x2 x2
x + 2 > 0 or, x > –2.
x5
(ii) For 0 , we first consider that x + 2 > 0 or x > 2 so that x – 5 < 0 or
x2
x < 5. Combining the two results, we get that –2 < x < 5.
If, on the other hand, x + 2 < 0, then x – 5 > 0
x < –2 and x > 5 which is not possible.
Hence the only solution is –2 < x < 5.
Problem 6. Draw the graph for the solution region satisfied by the inequalities
5
x y 5, x 2y 1, x y 4, x 0, y 0.
2
P-2022-CBSE-P1-MATHEMATICS-LINEAR INEQUALITIES
10
Solution: We find that the solution set satisfies x 0, y 0, 5x + y = 5
Line (1) meets the coordinate axes in (5, 0) and (0, 5) and the lines x = 4 and
y = 4 in (4, 1) and (1, 4), and 0 < 5 is true. Hence (0, 0) belongs to the half plane
x + y 5. But (0, 0) does not belong to the half planes 5x + y 5 and x + 6y 6.
The line 5x + y = 5, meets the coordinate axes in (1, 0) and (0, 5), and meets the
line x = 4 in (4, 15), where as it meet the line y = 4 in (1/5, 4).
Similarly x + 6y = 6 meets x = 4 in (4, 1/3) and y = 4 in (–18, 4).
The solution is marked as the shaded region.
Problem 8. Draw the graph of the solution region of the inequations x 0, y 0, x – y 1,
x – y –1.
Solution: Here the region belongs to the first quadrant as Y
x 0, y 0.
Moreover, the region is bounded by the straight
lines x – y = –1 x–y=1
x – y = 1, (1)
x – y = –1. (2)
X
These are parallel lines.
Moreover, both 0 1 and 0 –1 are not true.
Hence the origin does not belong to the half planes
x – y 1 and x – y –1
Both the half planes are away from the origin and are disjointed.
Hence the solution set is empty.
2 3x 1 x 3 4 x
Problem 10. Solve for x: .
5 3 2
P-2022-CBSE-P1-MATHEMATICS-LINEAR INEQUALITIES
11
ASSIGNMENT PROBLEMS
4. Find all pairs of consecutive even positive integers, both of which are larger than 5 such
that their sum is less than 23.
5. In a drilling process, the temperature T (in degree Celsius) x km below the surface of the
earth, was found to be given by T = 30 + 25(x – 3), 3 < x < 15.
At what depth will the temperature be between 200C and 300C .
6. An aeroplane can carry a maximum of 360 passengers. The airlines reserves at least 20
seats for business class. But at least 8 times as many passengers prefer to travel by
economy class than by business class. Find the possible graphic distribution of
passengers in the two classes.
7. A company makes chairs and tables. The company has two different types of machines
on which the chairs and tables are manufactured. The time required in hours for
manufacturing each item and total available time in a week is given below:
Chairs Tables Total available time (in hours)
Machine 1 5 2 24
Machine 2 4 8 64
Show graphically the sets of chairs and tables that can possibly be manufactured in a
week.
8. Solve the systems:
2x 5 2x 5
(i) 2x 3 5, 3 Hint : Write 3 0
x7 x7
x2
(ii) 3, 2x 7 5 .
x2
x 3 2x 5 3x 5
9. Solve for x : .
4 5 7
10. A plumber can be paid either (i) Rs. 600 and Rs. 50 per hour or (ii) Rs. 170 per hour. If
the job takes n hour, for what value of n the first mode i.e. (i) earns better wages for the
plumber.
11. A toy manufacturer produces dolls and machine guns under the constraints:
(a) he cannot produce more than 6 dolls per day and more than 5 machine guns per day.
(b) he must use minimum of 3 hours of labour per day.
(c) the raw material available is atmost 63 units per day.
If 2 hours of labour is required to produce a doll or a machine gun whereas 7 units of raw
material is required for a doll and 9 units for a machine gun, construct the problem.
12. A dealer deals in two items; radio and two-in-one. A radio costs Rs 500 and a two-in-one
costs Rs 2,500. Construct the problem under the constraints that he has Rs 50,000/ to
spend and space for 60 pieces to store.
P-2022-CBSE-P1-MATHEMATICS-LINEAR INEQUALITIES
12
Objective:
Level – I
x3 x 1 x 2
2. The value of x for which x , 2 x > 2x 8
4 2 3
10 10
(A) 1, (B) 1,
3 3
(C) R (D) none of these
27 x 27 9x
3. The least integer satisfying 49.4 47.4 is
10 10
(A) 2 (B) 3
(C) 4 (D) none of these
1
4. If 0 < |3x + 1| < , then x belongs to
3
4 2 4 2
(A) , (B) ,
9 9 9 9
4 2 1 4 2 1
(C) , (D) ,
9 9 3 9 9 3
2
5. If 3 , then x belongs to
x
2 2
(A) , (B) ,
3 3
2
(C) , , 0 (D) none of these
3
8. If |3 x| = x 3, then x is
(A) x < 3 (B) x > 3
(C) x 3 (D) x 3
x 1
9. The value of x for which 2
x
(A) (0, 1) (B) (–, –1)
(C) (–, 0) (D) [–1, 0)
P-2022-CBSE-P1-MATHEMATICS-LINEAR INEQUALITIES
13
x3 x 1 x 2
10. The value of x for which x , 2 – x > 2x – 8
4 2 3
10 10
(A) 1, (B) 1,
3 3
(C) R (D) none of these
Level – II
7. If |x 1| |x 3|, then
(A) x 2 (B) x 2
(C) x [1, 3] (D) none of these
P-2022-CBSE-P1-MATHEMATICS-LINEAR INEQUALITIES
14
9. If |x 1| 3 and |x 1| 1, then x belongs to
(A) [2, 4] (B) (, 2) [4, )
(C) [2, 0] [2, 4] (D) none of these
1
10. If x 2 , then x is
x
(A) R {0} (B) R {1, 0, 1}
(C) R {1} (D) R {1, 1}
1
11. If 2 4 , then x belongs to
x
1 1
(A) , (B) ,
2 6
1 1 1 1
(C) , (D) , ,
2 6 2 6
x2 6
14. 1 , then x belongs to
5x
(A) (, 3) (B) (, 3) (3, )
(C) R (D) (, 3] [2, 0) (0, 2] [3, )
P-2022-CBSE-P1-MATHEMATICS-LINEAR INEQUALITIES
15
21. If a triangles has integral sides and length of two sides are 2014 and 2015, then number
of such triangles is
(A) 2400 (B) 4003
(C) 4027 (D) none of these
Assertion Reason
COMPREHENSIONS
Read the following write up carefully and answer the following questions:
The marks obtained by a student of class X in first and second terminal examination are 70 and
40, respectively, then
24. The minimum marks he should get in the annual examination to have an average of at
least marks 65 marks
(A) 71 (B) 81
(C) 85 (D) 95
25. Marks he should get in the annual examination to have average between 40 to 60
(A) 40 to 70 (B) 30 to 60
(C) 10 to 50 (D) 10 to 70
Read the following write up carefully and answer the following questions:
P-2022-CBSE-P1-MATHEMATICS-LINEAR INEQUALITIES
16
Matrix Match
Numerical Based
31. A solution of nitric acid is to be kept between 30° and 35° Celsius, then find the difference
between the highest and lowest temperature in degree Fahrenheit if conversion formula
5
is given by C = (F 32), where C and F represents temperature in degree Celsius and
9
degree Fahrenheit respectively.
P-2022-CBSE-P1-MATHEMATICS-LINEAR INEQUALITIES
17
33. If V denotes the volume of water that will have to we added to 1125 litres of the 45%
solution of acid so that the resulting mixture will contain more than 25% but less than
V
30% acid content, then find the value of max .
100
MA
34. If IQ of a student can be given by IQ = 100 where MA and CA represents the
CA
mental age and chronical age of the student respectively. If IQ of a class of 12 years old
students varies from 80 to 140 and M1 and M2 represents the minimum and maximum
mental age of the group, then find the value of M2 M1 0.2.
P-2022-CBSE-P1-MATHEMATICS-LINEAR INEQUALITIES
18
ANSWERS TO ASSIGNMENT PROBLEMS
2. (i) (ii)
x2y+4=0
(0, 2)
y +8=2x
(4, 0)
(iii) y
2x=63y
3. (i) y (ii)
x =y
x+3y=30
x
x +y=9
x =1 3x+y=60
(iii) y
2x+y=4
x
x +y =3
2x3y=6
P-2022-CBSE-P1-MATHEMATICS-LINEAR INEQUALITIES
19
(v)
(4, 6)
x+2y =16
x x+y =10
3x+2y =24
x+y =360
x =20
4x+8y=64
5x+2y =24
5
9. x ,
3
10. n<5
P-2022-CBSE-P1-MATHEMATICS-LINEAR INEQUALITIES
20
Objective:
Level – I
1. A 2. B 3. B 4. C
5. C 6. B 7. B 8. C
9. D 10. B
Level – II
1. A 2. C 3. B 4. C
5. D 6. B 7. B 8. D
9. C 10. B 11. D 12. A
13. A 14. D 15. D 16. A
17. C, D 18. A, B, C, D 19. A, B 20. C, D
21. C 22. D 23. A 24. C
25. D 26. C 27. D 28. A
29. (A) (q, s) (B) (q) (C) (p) (D) (p)
30. (I). D (II). C (III). D
31. 9 32. 3 33. 9 34. 7
P-2022-CBSE-P1-MATHEMATICS-LINEAR INEQUALITIES
PINNACLE-CBSE
LOGARITHMS
CONTENTS SYLLABUS
Introduction 1
Logarithmic function 1 CBSE: Definition, Graph of Logarithm Function and
Exercise 1 3
their Properties.
Taking Logarithms 4
Change of variable 4
Exercise 2 5
Exercise 3 6
Common logarithms 6
Miscellaneous Exercise 7
Answers to Exercises 8
Solved Problems 9
Chapter Practice Problem 23
Assignment Problems 25
Answers to Chapter 38
Practice Problems
Answers to Assignment
Problems 39
Introduction:
Logarithms and exponents are closely related. Repeated addition of the same number (or
quantity) evolves multiplication and the repeated multiplication of the same factor evolves
exponentiation. The inverse of these operations leads to extracting roots and taking logarithms.
9 3
e.g. 2 = 512, 9 = 729
i.e. ninth power of 2 is 512 and third power of 9 is 729. In general, for a positive real number a
and rational number m, we have am = b, where b is a real number. We can state that the mth
power of base a is b. We can also say that the logarithm of b to the base a is m.
Logarithmic Function:
If a > 0 and 1, then logarithm of a positive number m is defined as the index x of that power of
‘a’ which equals m i.e. loga m = x iff ax = m
aloga m m . ….(1)
The function defined by f (x) = loga x, a > 0, a 1 is called logarithmic function. Its domain is
(0, ) and range is R.
When base is ‘e’ then the logarithmic function is called natural logarithmic function and when
base is 10, then it is called common logarithmic function.
P-2022-CBSE-P1-MATHEMATICS-LOGARITHMS
2
Properties of Logarithmic Functions
2. logaxn = nlogax
Proof: Let logaxn = t
Using definition of logarithm xn = at
Now taking 1/n power, x = at/n
Using definition of logarithm
logax = t/n t = nlogax logaxn = n logax.
1
3. logam x loga x
m
Proof: logam x t x = (am)t x = amt
Now using definition of logarithm
1
logax = mt t = 1/mlogax logam x loga x
m
loga b
4. logc a = = logb a logc b
logb c
P-2022-CBSE-P1-MATHEMATICS-LOGARITHMS
3
x1 x
a y1 y2 y1 y2 = loga 1
x2 x2
x
i.e. logax1 logax2 = loga 1 .
x2
Remark:
1
If x1 = 1 and x2 = x then loga1 logax = loga
x
1
loga = logax. ( loga 1 = 0)
x
alogb c clogb a
Proof: Let k = alogb c so that
logc
log k = log alogb c logb c (loga) =
logb
loga logb a(logc) log clogb a
k = c logb a .
loga x
Illustration 1: Prove that = 1 + loga b, for permissible values of letters involved in the
logab x
result.
43
1
Illustration 2: Show that log n
log n 43 ! .
r 2 r
43
1 1 1 1
Solution: log n
log2 n log3 n
...
log43 n
r 2 r
43
1 1
Using the property loga b =
logb a
log n
logn 2 logn 3 ... logn 43
r 2 r
Exercise 1.
1 1 1
i) Prove that 1.
loga abc logb abc logc abc
ii) Solve loge (x a) + loge x = 1.
xy 1
iii) If for x, y R+, ln (ln x + ln y), show that x = y.
2 2
iv) If log216 = 4, find log162.
(v) Find the value of x in (loge2) (logx625) = (log1016) (loge10).
P-2022-CBSE-P1-MATHEMATICS-LOGARITHMS
4
Algorithm of Solving Logarithmic Equations:
Illustration 3: Find the value of x satisfying the equation log3(5 + 4log3(x – 1)) = 2.
Taking logarithms
In some equations, when we take logarithm on both sides the equation converts into a solvable
form.
Change of variable
We can substitute loga x t , a new variable and then solve the resulting equation.
2
Illustration 6: Solve for x: log2 x log2 x 3 2 0
P-2022-CBSE-P1-MATHEMATICS-LOGARITHMS
5
Passing to a new base
logb x
We can convert a log equation into a new base by the identity loga x
logb a
Illustration 8: Find the value of x for which log (x + 3) + log (x – 3) = log 27.
Exercise 2.
log a log b log c
i) If , then prove that aa bb cc = 1.
bc ca ab
ii) Solve for x: log 2 + log (x + 2) log (3x 5) = log 3.
loga x logb x
iii) Prove that logabx = .
loga x logb x
iv) logx+1(x2 + x 6)2 = 4.
v) log5120 + (x – 3) 2 log5(1 – 5x -3) = log5(0.2 – 5x – 4)
P-2022-CBSE-P1-MATHEMATICS-LOGARITHMS
6
loga x p x ap
loga x 0 0 x 1
loga x p 0 x ap
(ii) If 0 < a < 1 loga x > 0 0 < x < 1
loga x > p 0 < x < ap
loga x < 0 x > a
loga x < ap x > ap
4x 6
Illustration 9: Find the solution set of the inequality log1 / 5 0.
x
Illustration 10: Find the solution set of the inequality log 2 x 3 x 2 log 2 x 3 2x 3 .
3
Solution: Case1: 0 < 2x + 3 < 1 <x<1
2
x2 > (2x + 3)
x2 – 2x – 3 > 0
(x – 3) (x + 1) > 0 x (-, -1) (3, )
But interval in consideration is x ( 3/2, 1)
Hence x ( 3/2, 1)
(x – 3) (x + 1) < 0 x ( 1, 3)
Complete Solution: x (-3/2, 1) ( 1, 3)
Exercise 3:
Solve for x:
log 3 x 6 x2 x
i) 3 log3 x 1
3 3 ii) log0.5 log6 0
x 4
x 1 1 1
iii) log x 6 log2 0 iv) Prove that < log345 <
3
x2 3 2
loga logb logc
v) If a, b, c are positive reral numbers such that , then prove that
bc c a ab
b+c c+a a+b a b c
(A) a +b +c 3 (B) a + b + c 3.
Common Logarithms:
In common logarithms the base is always taken as 10.
P-2022-CBSE-P1-MATHEMATICS-LOGARITHMS
7
Note:
p
Any positive number k in decimal form is written in the form k = m 10 where p
is an integer and 1 < m < 10. This is called the standard form of k.
Miscellaneous Exercise
3/2
i) Show that 3 log 4 2 log 6 + log (18) = log 96 2 .
ii) If log0.3 (x – 1) < log0.09 (x – 1), find the interval in which x lies.
2
iii) Solve for x: x log x ( x 3 ) 16 .
8 2
iv) Find x if it is given by l og8 2 3 log8 x .
x
v) Find x if 3 loga x 3 x loga 3 2 .
P-2022-CBSE-P1-MATHEMATICS-LOGARITHMS
8
ANSWERS TO EXERCISES
Exercise 1.
a a2 1
(ii) x= e (iv)
2 4 4
(v) 5
Exercise 2.
19
(ii) x= (iv) x=1
7
(v) x=1
Exercise 3:
(i) (6, ) (ii) ( 4, 3) (8, )
(iii) ( 6, 5) ( 3, 2)
Miscellaneous Exercise
P-2022-CBSE-P1-MATHEMATICS-LOGARITHMS
9
SOLVED PROBLEMS
Subjective:
Level – 0
1 1 1
Problem 2. If n = 1000!, evaluate ...... .
log2 n log3 n log1000 n
Solution: We have
1 1 1
......
log2 n log3 n log1000 n
log2 log3 log1000
= ......
logn logn logn
log2 log3 log 4 ...... log1000
=
logn
log2 3 4 .........1000 log(1000)!
= = 1.
logn logn
10
Problem 3. If log 8
x , then find x.
3
10 log x log x 2log x
Solution: Here log 8
x =
3 log 8 (1/ 2)log8 log8
log810 = log x6 810 = x6
x = (8)10/6 = (8)5/3 = 25 = 32.
1
log2.5
Problem 4. Find the value of 0.16 2 .
1 1
log5 / 2 2log2.5
Solution: Let A = 0.16 2 = 0.4 2
2 2
1 1
log log
2 2.5 2 5 5 / 2 2
= =
5 2
log5 / 2 22
5
= = 22 = 4.
2
P-2022-CBSE-P1-MATHEMATICS-LOGARITHMS
10
Level – I
Problem 6. If in a right angled triangle, a and b are the lengths of sides and c is the length of
hypotenuse and c b 1, c +b 1, then show that
logc+ba + logcba = 2logc+ba . logcba .
=
loga c b loga c b
=
loga c 2 b2
loga c b loga c b loga c b loga c b
loga a2
= (using (1) )
loga c b loga c b
2
=
loga c b loga c b
= 2 logc+ba . log(c b) a = RHS.
Alternative Solution
lna lna
LHS = logc + ba + logc-ba =
ln(c b) ln(c b)
lna ln(c 2 b2 ) lna 2lna
= =
ln(c b)ln(c b) ln(c b)ln(c b)
= 2 logc+ba . logc-ba = RHS.
Problem 7. Given a positive term geometric progression a, a1, a2,... and an arithmetic
progression b, b1, b2,.... with positive common difference. Show that there exists
a system of logarithm for which logan bn=loga b (for any n). Find the base of
the system.
Solution: Let d be the common difference of the A.P. and r be the common ratio of the
G.P. so that,
n
an = ar , ....(1)
bn = b + nd. ....(2)
Taking logarithms on both sides of (1) to base ( 1, > 0), we get
log an log a nlog r . ....(3)
Operating (3) (2) leads to
P-2022-CBSE-P1-MATHEMATICS-LOGARITHMS
11
log an bn log a nlog r b nd . ....(4)
Now, in order that the RHS of (4) reduces to loga b, we must have
n logr nd = 0
or logr = d r = d =r1/d.
Hence there exists a system of logarithm to base r1/d such that
logan bn = loga b.
Alternative Solution:
Let d be the common difference of the A.P and r be the common ratio of the
G.P., so that
bn = b + nd and an = arn ln an = ln a + n ln r
b b
= ln a + n lnr
d
= ln a + (bn – b) ln r1/d logx an = logx a + bn- b, where x = r1/d.
Problem 9.
Solve for x, log7 log5 x + 5 + x = 0.
Solution: From the given equation, log5
x 5 x 70 1 log5 5
x5 x = 5
or x 5 5 x x + 5 = 25 + x - 10 x
or x = 2 x = 4.
Problem 10. Solve for x and y, the equations log100|x + y| = 1/2 and log10 y– log10 |x| = log100 4.
P-2022-CBSE-P1-MATHEMATICS-LOGARITHMS
12
Level – II
8 2
Problem 11. Solve for x, log8 2 3 log8 x .
x
Problem 12. Find the solution set of the inequality log0.3 x 1 < log0.09 x 1 .
5
1 1
6 log10 xloga 5 log10 x log2 2
a 3log10 x 1 = 9 2 2
5
6 loga 5 log10 x
3 10 3 10
log x 1 log x 1
a
5
6 log10 x
3 10 3 10
log x 1 log x 1
5
5
6 5log10 x 1 3 10 1 32
log x 1
log x 1 1
5 10 10 5
3 6 3
log 10 x – 1 = 1
2
log 10 x = 2 x = 10 = 100.
Problem 14. Find the value of 3 log 2 + log sin x + log cos x + log cos 2x + log cos 4x.
Solution: 3 log 2 + log sin x + log cos x + log cos 2x + log cos 4x
= log (8 sin x cos x cos 2x cos 4x)
= log (4 sin 2x cos 2x cos 4x)
= log (2 sin 4x cos 4x)
= log (sin 8x)
P-2022-CBSE-P1-MATHEMATICS-LOGARITHMS
13
1 1 1 1
Problem 15. Find the value of + + + …+ .
log2 N log3 N log4 N log2005 N
1 1 1 1
Solution: + …+
log2 N log3 N log4 N log2005 N
log N 2 + log N 3 + log N 4 + … + log N 2005
log N(2 3 4 2005)
1
n =
log 2005 N
P-2022-CBSE-P1-MATHEMATICS-LOGARITHMS
14
Objective:
Level – 0
True / False
1 1 1 n n 1
Problem 1. The sum of the series ..... upto n terms is loga2
log2 a log4 a log8 a 2
Solution: True
The given expression can be written as
loga2 + loga4 + loga8 + .... upto n terms
= loga2 + 2 loga2 + 3 loga2 + .... n loga2
n n 1
= (loga2) [1+ 2 + ... n] = loga 2 .
2
Solution: False
10
We have log 8
b 3 13 log23 / 2 b
3
2 10
log2 b log2b = 5 b = 25.
3 3
Solution: True
The given equation yields 1 + logb{1 + logc(1 + logpx)} = 1
logb{1 + logc(1 + logpx)} = 0 1 + logc(1 + logpx) = 1
logc(1 + logpx) = 0 1 + logpx = 1
logpx = 0 x = 1.
Problem 4. For 0 < x < 1, the value of log(1 + x) + log(1 + x2) + log(1 + x4) .... to is logx
Solution: False
2
The given expression = log [(1 + x) (1 + x ) ..... to ]
1
= log[ 1+ x + x2 + x3 + .... to ] = log
1 x
= log(1 x).
3x -4
Problem 5. The value of x satisfying 184x3 = 54 2 is 6
Solution: True
3x 4
Here 18
4x3
= 54 2 .
Taking log on both sides, we get
P-2022-CBSE-P1-MATHEMATICS-LOGARITHMS
15
(4x 3) log18 = (3x 4) log54 2
3
(4x 3) log18 = (3x 4) log(18)3/2 (4x 3) = (3x 4).
2
x = 6.
Solution: 2
Here log2(32x2 + 7) = log24 + log2(3x1 + 1)
x1
= logx 4(3 + 1)
2(x1) x1
3 4.3 + 3 = 0
x1 x1
(3 1) (3 3) = 0
3x1 = 1 = 30 and 3x1 = 3 = 31
x = 1 and x = 2.
Solution: 3(1 – c – d)
We have log308 = log3023 = 3 log30 30
15
= 3 log30 30 log30 3 5
= 3[1 c d]
log 9 36 4/log 7 9
Problem 8. The value of 81(1/log5 3) 27 3 is equal to _____
Solution: 890
81(1/log5 3) 27log9 36 34/log7 9
1
3. log3 36
3 4 log3 5 3 2 3 4 log9 7
4 3/2 4/ 2
3log3 5 3log3 36 3log3 7
5 4 363/2 72 890 .
Level – I
16 25 81
Problem 9. 7log 5log 3log is equal to
15 24 80
(A) 0 (B) 1
(C) log2 (D) log3
Solution: C
167 255 813
Given expression = log 7 . 5 . 3 log2 .
15 24 80
P-2022-CBSE-P1-MATHEMATICS-LOGARITHMS
16
Problem 10. If log4 5 a and log5 6 b, then log3 2 is equal to
1 1
(A) (B)
2a 1 2b 1
1
(C) 2ab 1 (D)
2ab 1
Solution: D
1
ab log4 5.log5 6 log4 6 log2 6
2
1
ab (1 log2 3) 2ab 1 log2 3
2
1
log3 2 .
2ab 1
Solution: C
loge a loge b loge c
log5 a.loga x 2 1 x 52 25 .
loge b loge c loge a
Solution: C
a2 4b2 12ab
a2 4b2 4ab 16ab (a 2b)2 16ab
2log(a 2b) log16 loga logb
1
log(a 2b) [loga logb 4log2]
2
Solution: C
A log2 log2 log4 256 + 2log2 1/ 2 2
1
log2 log2 log4 44 2 log2 2
(1/ 2)
log2 log2 4 4 log2 log2 22 4
log2 2 4 1 4 5 .
P-2022-CBSE-P1-MATHEMATICS-LOGARITHMS
17
Level – II
Solution: D
2 2
log1000 x 2 log103 x 2 2log103 x log10 x y .
3 3
Problem 15. If x loga (bc), y logb (ca),z logc (ab), then which of the following is equal to 1
(A) x y z (B) (1 x)1 (1 y)1 (1 z)1
(C) xyz (D) None of these
Solution: B
x loga bc 1 x loga a loga bc loga abc
(1 x)1 logabc a
(1 x)1 (1 y)1 (1 z)1 logabc a logabc b logabc c
logabc abc 1 .
Problem 16. If a log24 12,b log36 24 and c log48 36, then 1+abc is equal to
(A) 2ab (B) 2ac
(C) 2bc (D) 0
Solution: C
log12 2log2 log3
a log24 12
log24 3log2 log3
3log2 log3
b log36 24
2(log2 log3)
2(log2 log3)
c log48 36
4log2 log3
2 log2 log3
abc
4log2 log3
6log2 2log3 3log2 log3
1 abc 2. 2bc .
4log2 log3 4log2 log3
Solution: B
a x b x loga logb
logb
x loga b
loga
P-2022-CBSE-P1-MATHEMATICS-LOGARITHMS
18
Similarly y logb c,z logc a
x lna ylnb xy ln(ab) k (say) xyz loga b.logb c.logc a 1.
n
1
Problem 19. log (a)
n 1 2n
n(n 1) n(n 1)
(A) loga 2 (B) log2 a
2 2
(n 1)2 n2
(C) log2 a (D) None of these
4
Solution: A
n n
1
log loga 2n = x 1
n 1 2n
(a) n 1
n(n 1) n(n 1)
= loga 2. loga 2 .
2 2
P-2022-CBSE-P1-MATHEMATICS-LOGARITHMS
19
log (0.1 0.01 0.001......)
Problem 22. The value of (0.05) 20
is
1
(A) 81 (B)
81
1
(C) 20 (D)
20
Solution: A
0.1
2 log20
log (0.1 0.01......) 1 10.1
(0.05) 20
20
2
20 2 log20 (1/9) 202 log20 9 20log20 9 92 81 .
Problem 23. If a, b, c are distinct positive numbers, each different from 1, such that
[logb alogc a loga a] [loga blogc b logb b] [loga c logb c logc c] 0, then abc =
(A) 1 (B) 2
(C) 3 (D) None of these
Solution: A
[logb a.logc a loga a] [loga b.logc b logb b] [loga c logb c logc c] 0
9
x
4
1
[(lna)3 (lnb)3 (lnc)3 3lna.lnb.lnc] 0
lna.lnb.lnc
(lna)3 (lnb)3 (lnc)3 3lna.lnb.lnc 0
lna lnb lnc 0
ln(abc) = ln 1, [a3 b3 c 3 3abc 0
a b c 0] , abc 1 .
Solution: C
log27 3log3 2alog2
a log3
log12 log3 2log2 3a
log16 4log2 4log2 4(3 a) 3a
log6 16 4. .
log6 log2 log3 2alog2 3 a 2a 3a
log2
3a
P-2022-CBSE-P1-MATHEMATICS-LOGARITHMS
20
Solution: B, C
logk x.log5 k logx 5 log5 x logx 5
1
logx 5 1 2B 0 logx 5 1
logx 5
1
x 1 5 x 5, .
5
Solution: A, D
log3x = 1 + 2 logx 3
(log3x)2 – log3x – 2 = 0
log3 = 2, – 1
x = 9, 1/3.
Solution: B, C
32x–2 + 7 = 4(3x–1 + 1)
(3x–1)2 – 4.3x–1 + 3 = 0
3x–1 = 1, 3
x = 1, 2
Paragraph Type
Problem 28. For x (1, 5), f(x) is not defined at how many points
(A) 5 (B) 4
(C) 3 (D) 2
Solution: C
{x} 0 x 2, 3, 4.
Problem 29. If A = {x: x domain of f(x)} and B = {x: x domain of g(x)}, then A – B will be
(A) (2, 3) (B) (1, 3)
(C) (1, 2) (D) none of these
Solution: C
[x] 2 x [2, 0)
A = {x: x I+}
B = {x: x I+ (2, )}
A – B = (1, 2).
P-2022-CBSE-P1-MATHEMATICS-LOGARITHMS
21
Solution: C
{x} 0 x R – I.
Solution: A q; B p; C s; D r
log3x = t x = 3t
3 5 1
t3 t 2 t
34 4 32
3 2
3t + 4t – 5t – 2 = 0
(3t + 1)(t + 2)(t – 1) = 0
1 1
x 3 , ,3
3 9
x2 1
x 5
Decimal Type
P-2022-CBSE-P1-MATHEMATICS-LOGARITHMS
22
2
ln 2x 1 ln 3x 4 2x 1
4
ln 3x 4 ln 2x 1
2ln 2x 1 ln 3x 4
1 4
ln 3x 4 ln 2x 1
ln 2x 1
2t2 − 3t + 1 = 0 where t =
ln 3x 4
(t − 1) (2t − 1) = 0 t = 1 or t = 1/2
When t = 1, we have 3x + 4 = 2x + 1 x = −3
When t = 1/2, we have 3x + 4 = (2x + 1)2
4x2 + x − 3 = 0 (x + 1) (4x − 3) = 0 x = −1, 3/4
But x = −1 and −3 make the base of the original equation negative.
3
Only solution is x = = 0.75.
4
P-2022-CBSE-P1-MATHEMATICS-LOGARITHMS
23
CHAPTER PRACTICE PROBLEMS
Subjective:
Level – I
1. Given log 2 = .30103, then find the number of ciphers between the decimal point and first
1000
1
significant digit in .
2
Level – II
4. Find the value of x satisfying the simultaneous equations a2x b3y = m5 and a3x b2y = m10.
5. If a b and a + b 1, then find the value of x satisfying the equation (a4 – 2a2b2 + b4)x–1
= (a – b)2x (a + b)–2 .
Objective:
4
4. On simplifying log 729 3 9 127 4/3 is equal to
2
(A) 2 log 3 (B) log3
3
1
(C) log 3 (D) log 3
3
5. If a, b, c be the pth, qth and rth terms of a G.P., then (q – r) log a + (r – p)log b + (p – q) log
c is equal to
(A) 0 (B) abc
(C) ap + bq + cr (D) none of these
P-2022-CBSE-P1-MATHEMATICS-LOGARITHMS
24
6. If a2 + b2 = 34ab, then 2 log(a + b) – log a – log b is equal to
(A) 0 (B) log 6
(C) 2 log 6 (D) none of these
x
7. If log (x3 y4) = m + 2n and log (x4 y3) = n + 2m, then log is equal to
y
m
(A) (B) m n
n
mn
(C) (D) m + n
7
1 1 1
8. If x = log a (bc), y = log b(ca) and z = log c (ab), then is equal to
x 1 y 1 z 1
(A) 1 (B) log abc
(C) log abc(a + b + c) (D) none of these
a3b3 b3c 3 c 3 d3
9. log + log + log – 3 log b2 c is equal to
c3 d3 a3
(A) 3(log a + log b + log c) (B) 3 log a
(C) –3(log b + log c) (D) 0
a x 1
10. If a, b, c are in G.P. and = b2x, then x is equal to
c x 1
(A) 1 (B) log c b
(C) log ac (D) log b a
3 5
(log3 x)2 log3 x
3. If x 4 4 3 then x has
Numerical Based
P-2022-CBSE-P1-MATHEMATICS-LOGARITHMS
25
ASSIGNMENT PROBLEMS
Subjective:
Level – 0
10
1. Determine b if log 8
b .
3
2. Evaluate:
(A) log2(log381) (B) log927 log279 (C) log6 216 6
3. If log V = log 4 – log 3 + log + 3log r, write down the formula for V in terms of and r
4. Prove that
(yz)logy logz (zx)logz logx (xy)logx logy = 1
7.
Find the value of x, satisfying loga 1 logb 1 logc 1 logp x 0 .
1
8. Solve x if log81 3 for x > 0.
x2
16 25 81
11. Find the value of 7log 5log 3log .
15 24 80
1 1 1
12. Find the value of .
1 logb a logb c 1 logc a logc b 1 loga b loga c
x2 y2 z2
15. Prove that log log log 0 .
yz zx xy
P-2022-CBSE-P1-MATHEMATICS-LOGARITHMS
26
ab 1
16. If log loga logb , prove that a = b.
2 2
1 1 1 1
18. Show that .
log2 x log3 x log43 x log43 ! x
1 1
23. Find the value of .
log3 12 log 4 12
1 1
24. Prove that >2
log3 log4
25. If log1/ 2
sin x 0,x [0, 4], then the number of values of x which are integral multiples of
, is
4
P-2022-CBSE-P1-MATHEMATICS-LOGARITHMS
27
Level – I
2 loga b
1. If logax, logbx, logcx are in A.P. where x 1, then show that c = ac .
1 1
2. If a2 + b2 = 7ab, prove that log a b loga logb .
3 2
log10 x 1
4. Solve : x 1 = 100(x + 1).
1 1 1
9. If log3 2, log3 (2x – 5), log3 (2x – 7/2) are in A.P., determine the value of x.
P-2022-CBSE-P1-MATHEMATICS-LOGARITHMS
28
Level – II
1 1 1
log2.5 2 3
2. Find the value of (0.16) 3 3 3 .
2
3.
Solve for x : logx 1 x 2 x 6 4.
4.
Solve for x : log5 x2 11x 43 2 .
x y z x y z x y zx y z
5. If , prove that yzzy = zxxz = xyyx .
log x log y logz
2x 8
6. Solve for x : log1.5 0.
x2
7. Find all real numbers x which satisfy the equation 2log2 (log2 x) log1/ 2 (log2 (2 2x)) 1 .
8. Solve for x and y, the equations log10 x + log10 x1/2 + log10 x1/4 + ……. to = y
1 3 5 ....... (2y 1) 20
and .
4 7 10 ....... (3y 1) 7log10 x
3 5
(log2 x)2 log2 x
10. Solve the equation x 4 4 2.
P-2022-CBSE-P1-MATHEMATICS-LOGARITHMS
29
Objective:
Level – I
1 1
5. The value of is
log3 log4
(A) 2 (B) less than 2
(C) more than 2 (D) none of these
6. The value of |logba + logab| where a and b are positive numbers is always
(A) 2 (B) 2
(C) = 2 (D) none of these
16 25 81
7. The value of 7log 5log 3log is
15 24 80
(A) log2 (B) log 3
(C) 0 (D) none of these
2
8. If log0.3(x – x – 5)70, then x lies in the interval
(A) (3, -2) (B) (3, 2)
(C) (-3, 2) (D) (-2, 3)
1 1 1 1
9. ....... is equal to
log2 n log3 n log4 n log43 n
logn 1
(A) (B)
log(43!) log 43!n
(C) log43! n (D) none of these
10. If log0.3(x 1) < log0.09(x 1), then x will lie in the interval
(A) (2, ) (B) (1, 2)
(C) (2, 1) (D) none of these
P-2022-CBSE-P1-MATHEMATICS-LOGARITHMS
30
loga logb a
11. The value of is
logb loga b
(A) logab (B) logba
(C) logab (D) none of these
1 1 1
15. If x > 1, y > 1, z > 1 are in G.P, then , , are in
1 ln x 1 ln y 1 ln z
(A) A.P (B) G.P
(C) H.P (D) none of these
16.
log3 log2 log 3
81 is equal to
(A) 2 (B) 1
(C) 3 (D) none of these
ln x ln y ln z
17. For , xyz is equal to
bc c a ab
(A) 1 (B) abc
(C) 0 (D) none of these
log x 1
19. The value of x in 2 is
logx
5 1
(A) 1 (B)
2
5 1
(C) (D) 2
2
P-2022-CBSE-P1-MATHEMATICS-LOGARITHMS
31
2
20. If 2log(x + 1) – log(x – 1) = log2, then x =
(A) 1 (B) 0
(C) 2 (D) 3
Level – II
1. The value of log7 log7 7 7 7 is
(A) 3 log2 7 (B) 1 3 log7 2
(C) 1 3 log2 7 (D) 3 log7 2
log x
2. The number of roots of the equation 3 x log5 2 2 5 64 is
(A) 2 (B) 4
(C) 1 (D) 0
2x 1
3. The point on the graph y log 2 log6 {2 4} whose y coordinate is 1, is
(A) (12, 1) (B) (6, 1)
(C) (8, 1) (D) (1, 1)
4. The number of solution(s) for the equation 2 logx a logax a 3 loga2 x a 0 is (are)
(A) one (B) two
(C) three (D) four
5.
The number of values of x satisfying 1 log5 x 2 1 log5 x 2 4x 1 is
(A) 1 (B) 2
(C) 3 (D) infinitely many
7. If x1, x 2 , x3 ,...... are positive numbers in G.P., then log xn , log xn1, log xn 2 are in
(A) A.P. (B) G.P.
(C) H.P. (D) none of these
x
8. If log2 x log2 4 0, then x is equal to
16
(A) 4 (B) –4
1
(C) (D) 2
4
P-2022-CBSE-P1-MATHEMATICS-LOGARITHMS
32
9. The sum of series
x x x x x
logcos log cos 2 log cos 3 ....... log cos n log sin n is equal to
2 2 2 2 2
1 1
(A) n sin x (B) log n sin x
2 2
1
(C) n cos x (D) none of these
2
10. Which is the correct order for a given number > 1 in increasing order?
(A) log2 , log3 , loge , log10 (B) log10 , log3 , loge , log2
(C) log10 , loge , log2 , log3 (D) log3 , loge , log2 , log10
1
11. The sum of the series
r 0
log e
3
2r
is equal to
loga n logb n
12. If a > 0, c > 0, b = ac , a 1, c 1, ac 1 and n > 0, then the value of is
logb n logc n
equal to
loga n logn a
(A) (B)
logc n logn c
(C) logca (D) none of these
16.
If S x : logx 3x , where log3 x 1 , then
(A) S is a finite set (B) S
1
(C) S 0, (D) S properly contains ,
3
P-2022-CBSE-P1-MATHEMATICS-LOGARITHMS
33
17. If
2
log2 4x x 1 1 , then x may be
log2 x 1 2
2
(A) , (B) 1,
3
2
(C) , 0 (D) none of these
3
18. If | a || b |, b a 1 and a, b are the real roots of the equation x 2 | | x | | 0 , then
x
the equation log|b | 1 0 has
a
(A) one root lying in the interval (, a) (B) one root lying in the interval (b, )
(C) one negative root (D) one positive root
1
19. If log0.1 x 2 , then
2
1 1 1
(A) the maximum value of x is (B) x lies between and
10 100
10
1 1 1
(C) x does not lie between and (D) the minimum value of x is
100 10 100
log x 2 2 logx 9 7
20. If x 1 3 x 1 , then
(A) x = 81 (B) x = 2
1
(C) two real solutions of ‘x’ are possible (D) x
3
21. The inequation logx 2 log2x 2 log2 4x 1
(A) has a meaning for all x (B) has a meaning if x> 2
1
(C) is satisfied in 2 2 ,
2
(D) is satisfied in 1, 2 2
22. Let a
log100 10 log2 log4 2 log log 256 . Then the value of a is
4 2
2
log4 8 log8 4
1
(A) greater than (B) 2
2
6 2
(C) (D)
13 3
23. If x is a positive number different from 1 such that loga x, logb x and logc x are in A.P.,
then
2 loga logc ac
(A) logb (B) b
loga logc 2
loga b
(C) b a.c (D) c 2 a.c
P-2022-CBSE-P1-MATHEMATICS-LOGARITHMS
34
25. The number log27 is
(A) an integer (B) real number
(C) an irrational number (D) a rational number
Reason Assertion
26. Statement-1: The equation log 3 5 4 log 3 x 1 = 2 has only one solution.
log x log 2
Statement-2: 2 3 x 3 ; x 0 .
(A) Statement -1 is True, Statement -2 is True; Statement-2 is a correct explanation for
Statement-1
(B) Statement -1 is True, Statement -2 is True; Statement-2 is NOT a correct explanation
for Statement-1
(C) Statement -1 is True, Statement -2 is False
(D) Statement -1 is False, Statement -2 is True
COMPREHENSION
Read the following write up carefully and answer the following questions:
If f(x) > 0, then {f(x)} 2n = g(x) is equivalent to 2n logaf(x) = logag(x), a > 0 and a 1
1
28. Solution set of logx = log (x + 1) is
2
1 1
(A)
2
2 1
(B)
2
5 1
1 1 1
(C) ,
2 3
(D)
2
5 7
29. Solution set of equation log(8 – 10x – 12x2) = 3log (2x – 1) is
(A) {1} (B) {3, 2}
(C) {5} (D)
Read the following write up carefully and answer the following questions:
P-2022-CBSE-P1-MATHEMATICS-LOGARITHMS
35
31. The number of solutions of equation loga x logb x for some a 1 and 0 b 1 is/ are
(A) 0 (B) 2
(C) 1 (D) 3
32. The set of values of x 0 x satisfying the function f x log 0.9 cos x sin x is
2
(A) 0, (B) ,
4 4 2
(C) , (D) none of these
2
Matrix Match
P-2022-CBSE-P1-MATHEMATICS-LOGARITHMS
36
36. Match the following :
Column I Column II
(A) f x log10 x 2 8
(p) f x 0 for , 0 ,
5
log x 1 (q) Domain f x is x 1 or x 1
(B) f x 2 2
x5
(C) f x 2 log2 4 5x (r) f x >0, x 4,
(D) 2
f x log 1 x 6x 12 (s) Domain of f x is R
2
(t) f x > 0, x 8,
Numerical Based
40. If log10x + log10y 2 and the smallest possible value of x + y is 4k, then the value of k is
P-2022-CBSE-P1-MATHEMATICS-LOGARITHMS
37
Decimal Type
44.
The number of ‘n’ such that logn 32013 is an integer is, n N .
45. If x log10 k , where A is set of all positive divisors of 108 except 108 itself, then the
kA
P-2022-CBSE-P1-MATHEMATICS-LOGARITHMS
38
ANSWERS TO CHAPTER PRACTICE PROBLEMS
Subjective:
Level – I
1. 301
3
2.
2
3. –log(1 – x)
Level – II
4. 4logam
log a b
5.
log a b
Objective:
1. A 2. B 3. D 4. C
5. A 6. C 7. B 8. A
9. D 10. B
1. B, C 2. A, B, C, D 3. A, B, C
Numerical Based
1. 2 2. 2
P-2022-CBSE-P1-MATHEMATICS-LOGARITHMS
39
ANSWERS TO ASSIGNMENT PROBLEMS
Subjective:
Level – 0
1. b = 32
5 7
2. (A) 2 (B) (C)
6 2
4 3
3. r
3
5. (A) log R = log P + log V – log T (B) log F = log m1 + log m2 – 2 log d
(C) log T = log 2 + log + 1/2 [log l – log g] (D) log V = log + 2 log r + log h – log 3
1 1
6. If a2 + b2 = 7ab, prove that log a b loga logb .
3 2
7. 1 8. 2
9. 4 10. 1
11. log 2 12. 1
2
13. x = 10 14. x=
3
x
17. 19. 1
10
20. 32 21. 5
4log3
22. 23. 1
log7 log6
24. 2 25. 4
Level – I
3. 16 4. 99, -9/10
5. x = 2, 81 7. x = 20
9. x=3 10. 5
11. 1 12. 9
Level – II
P-2022-CBSE-P1-MATHEMATICS-LOGARITHMS
40
Objective:
Level – I
1. A 2. A 3. B 4. B
5. C 6. A 7. A 8. D
9. B 10. A 11. C 12. D
13. B 14. B 15. C 16. B
17. A 18. B 19. C 20. D
21. C
Level – II
1. B 2. C 3. A 4. B
5. D 6. A 7. A 8. A
9. B 10. B 11. D 12. A
13. B 14. B 15. C 16. C
17. A, B 18. C, D 19. A, B, D 20. A, B, C
21. C, D 22. A, C 23. A, D 24. B, D
25. B, C 26. B 27. A 28. B
29. D 30. B 31. C 32. A
33. D 34. C
35. (A) (q) (B) (p) (C) (s) (D) (r)
36. (A) (q) (B) (r) (C) (p) (D) (p), (s)
37. (I). B (II). A (III). C
38. 5 39. 6 40. 5 41. 1
42. 1 43. – 0.25 44. 8.00 45. 316.00
P-2022-CBSE-P1-MATHEMATICS-LOGARITHMS
PINNACLE-CBSE
Exercise 5 12
Exercise 6 16
Miscellaneous Exercise 17
Answers to Exercises 18
Solved Problems 19
Assignment Problems 41
Answers to Chapter 53
Practice Problems
Answers to Assignment 54
problems
QUADRATIC EQUATION
2
An equation of the form ax + bx + c = 0, where a 0, is called a quadratic equation. The
numbers a, b, c are called the coefficients of the quadratic equation. A root of the quadratic
equation is a number (real or complex) such that a2 + b + c = 0.
b b2 4ac
The roots of the given quadratic equation are given by x = .
2a
Proof: Dividing the given equation by a, we get
b c
x2 + x 0
a a
2 2
2 b b c b
or x + 2 x 0
2a 2a a 2a
2
b b2 4ac
or x
2a 4a2
b b2 4ac b b2 4ac
x+ x= .
2a 2a 2a
Basic Results:
The quantity D (D = b2 – 4ac) is known as the discriminant of the quadratic equation. For a, b, c
real,
The quadratic equation has real and equal roots if and only if D = 0 i.e. b2 – 4ac = 0.
The quadratic equation has real and distinct roots if and only if D > 0 i.e. b2 – 4ac > 0.
The quadratic equation has complex roots with non–zero imaginary parts if and only if
D < 0 i.e. b2 – 4ac < 0.
If p + iq (p and q being real) is a root of the quadratic equation where i = 1 , then
p – iq is also a root of the quadratic equation.
If p + q is an irrational root of the quadratic equation, then p – q is also a root of the
quadratic equation provided that all the coefficients are rational.
The quadratic equation has rational roots if D is a perfect square and a, b, c are
rational. If a = 1 and b, c are integers and the roots of the quadratic equation are
rational, then the roots must be integers.
Proof: Let b be even i.e. b = 2m. Then D = 4m2 4c is even if D is a perfect square
b D
( 2m D) is even is an integer.
2
If b is odd i.e. b = 2m + 1 then D = 4m2 4 1 4c is odd if D is a perfect square
b D
((2m + 1) D) is even is an integer.
2
If the quadratic equation is satisfied by more than two numbers (real or complex), then it
becomes an identity i.e. a = b = c = 0.
P-2022-CBSE-P1-MATHEMATICS-QEE
2
2
Proof: Let , , be three different roots of the quadratic equation ax + bx + c = 0. Hence
a2 + b + c = 0, … (1)
2
a + b + c = 0, … (2)
a2 + b + c = 0. … (3)
Subtract (2) and (3) from (1) and get
a (2 2) + b ( ) = 0 and a (2 2) + b ( ) = 0. … (4)
Since , we get
a ( + ) + b = 0 and a ( + ) + b = 0. … (5)
Again subtracting one equation of (5) from the other, we have
a ( ) = 0 a = 0, as
and hence from (5), b = 0, which in turn gives that c = 0
from (1), (2) or (3).
Factorization Method:
In the factorization method we write the coefficient of x as a sum of two numbers l and m such
that lm = ac i.e. we write
b = l + m with lm = ac
so that the equation ax2 + bx + c = 0 becomes
lm
ax2 + (l + m) x + 2 2
0 or a x + a (l + m) x + lm = 0
a
or (ax + l) (ax + m) = 0 ax + l = 0 or ax + m = 0.
l m
Hence the roots are x = and x = . In order to obtain l and m, we write
a a
lm= (l m)2 4lm b2 4ac .
With l + m = b, we get
b b2 4ac b b2 4ac
l= and m =
2 2
b b2 4ac
so that the two roots are .
2a
b
Let and be two roots of the given quadratic equation. Then + = – and
a
c
= .
a
A quadratic equation, whose roots are and can be written as (x – ) (x – ) = 0
i.e., ax2 + bx + c a(x – ) (x – ).
P-2022-CBSE-P1-MATHEMATICS-QEE
3
b D 5 1 b D 5 1
= 3, 2.
2a 2 2a 2
(ii) We have a = 1, b = 1, c = 1, so that
D = b2 4ac = 1 4 = 3 < 0.
Hence we obtain the pair of complex roots as
b i D 1 i 3 1 i D 1 i 3
= and = .
2a 2 2a 2
2 2
(iii) We have a = pq, b = p q , c = pr so that
D = b 4ac = (p q ) 4pq ( pq) = (p2 q2)2 + 4p2q2 = (p2 + q2)2 > 0.
2 2 2 2
Illustration 2. Prove that the roots of the quadratic equation ax2 – 3bx – 4a = 0 are real and
distinct for all real a and b.
Problems based on value of symmetrical functions of and , where and are roots of a
quadratic equation.
b c
Solution: Here + = – and = .
a a
2 2
Now, a
a 3 3 b 2 2
b =
3 2
a 3 b 2
=
b 3 c b b 2 c
a 3 b 2 bc
a a a a a
= = a b.
c/a c/a
P-2022-CBSE-P1-MATHEMATICS- QEE
4
Exercise 1.
i) Show that if the roots of the equation (a2 + b2)x2 + 2x (ac + bd) + c2 + d2 = 0 are real,
they will be equal.
2 2 2 2 2
ii) Show that the roots of the equation (a + b) x – 2(a – b )x + (a – b) = 0 are equal.
iii) Prove that the roots of the equation bx2 + (b –c)x + (b –c –a) = 0 are real if those of
ax2 + 2bx + b = 0 are imaginary and vice versa.
2
iv) If , are the roots of the equation x – px + q = 0 and 1, 1 the roots of equation
1 1 1 1
x2–qx+p=0, form the quadratic equation whose roots are and .
1 1 1 1
v) Find the sum of real roots of the equation x2 + |x| 6 = 0.
Illustration 6. Find the value of p if the equations 3x2 – 2x + p = 0 and 6x2 – 17x + 12 = 0 have
a common root.
2
Solution: Let be the common root. Then 3 –2 + p = 0, ....(1)
and 62 –17 + 12 = 0 ....(2)
2
1 1 24 17p
2 =
24 17p 6p 36 51 12 39 39
2 2
36 6p 24 17p 36 6p 12 2p
and = =
39 39 39 13
144 48p 4p2
= 312 –221p = 432 –144p + 12p2
169
2 15 8
12p + 77p + 120 = 0 (4p + 15) (3p + 8) = 0 p = – , .
4 3
P-2022-CBSE-P1-MATHEMATICS-QEE
5
3 2 2
Illustration 7. If x 3x 3 x 2 0 and ax bx c 0 , a, b, c R have two common roots
then the roots of the equation ax2 + (a + b)x + c = 0 are
(A) imaginary (B) real and unequal
(C) real and equal (D) None of these
Solution:
x 3 3x 2 3x 2 x 2 x 2 x 1
2
the common roots are ,
a = b = c = 1.
Now, ax2 + (a + b)x + c = 0
2
x + 2x + 1 = 0
2
(x + 1) = 0
roots are real and equal.
Illustration 8. The value of ‘a’ for which the equation x3 + ax + 1 = 0 and x4 + ax + 1 = 0 have a
common root, is
(A) 2 (B) –2
(C) 3 (D) –3
Exercise 2.
i) If two equations x2 + cx + ab = 0 and x2 + bx + ca = 0 have a common root, show
that a + b + c = 0.
ii) Prove that equations (q –r)x2 + (r –p)x + p –q = 0 and (r –p)x2+ (p –q)x + q –r = 0
have a common root.
iii) If the equations x2 x p = 0 and x2 + 2px 12 = 0 have a common root, find it.
iv) If the equations x2 + px + q = 0 and x2 + px + q = 0, (p p, q q) have a common
pq p q q q
root, prove that it is equal to or .
qq p p
v) If equations ax2 + bx + c = 0 and cx2 + bx + a = 0 have a negative common root,
then find the value of (a b + c).
P-2022-CBSE-P1-MATHEMATICS- QEE
6
an2
12 + 13 + 14 + …. + 23 + … + n – 1n = ,
an
………………………………
r anr
12 . . . . . r + …. + n–r+1n–r+2 … n = ( –1) ,
an
…….…………………………
a0
12 … n = (–1)n a .
n
4 3 2
e.g. If , , and are the roots of ax + bx + cx + dx + e = 0 then
+ + + = –b/a,
+ + + + + = c/a,
+ + + = –d/a,
= e/a.
Notes:
A polynomial equation of degree n has n roots (real or imaginary).
If all the coefficients are real then the imaginary roots occur in pairs i.e. number of
complex roots is always even.
If the degree of a polynomial equation is odd then the number of roots will also be odd.
It follows that at least one of the roots will be real.
If is a repeated root (repeating r times) of a polynomial equation f(x) = 0 of degree n
i.e. f(x) = (x – )r g(x) , where g(x) is a polynomial of degree n – r and g() 0, then
f() = f() = f() = ... = f (r–1)() = 0 and f r () 0 .
Remainder Theorem: If we divide a polynomial p(x) by x – , the remainder obtained is
p(). Note that if p() = 0, then x – is a factor of p(x).
If a polynomial equation of degree n has n + 1 roots say x1,...xn + 1, (xi xj if i j), then
the polynomial is identically zero. ie. p(x) = 0, x R.
(In other words, the coefficients a0, .... an are all zero).
If p(a) and p(b) (a < b) are of opposite sign, then p(x) = 0 has odd number of roots in (a,
b), i.e. it have at least one root in (a, b).
If coefficients in p(x) have `m’ changes in signs, then p(x) = 0 have at most `m’ positive
real roots and if p(–x) have `t’ changes in sign, then p(x) = 0 have at most `t’ negative
real roots. By this we can find maximum number of real roots and minimum number of
complex roots of a polynomial equations p(x) = 0.
Illustration 9. Let a and b be two roots of the equation x3 + px2 + qx + r=0 satisfying the relation
ab + 1=0. Prove that r2 + pr + q + 1=0. (r 0)
P-2022-CBSE-P1-MATHEMATICS-QEE
7
2 3 2
Illustration 10. If b < 2ac, then prove that ax + bx + cx + d = 0 has exactly one real root.
Illustration 11. Total number of roots of (x2 + x + 1)2 + 2 = (x2 + x + 1) (x2 – 2x – 6), belonging to
(–2, 4) is
(A) 2 (B) 3
(C) 1 (D) none of these
2y
Solution: (x2 –5x + 4) (y2 + y + 1) < 2y y R x2 –5x + 4 < 2
y y 1
2
Now least value of RHS is equal to –2 x –5x + 4 < –2 x (2, 3).
Exercise 3.
2 1 1
i) Find the roots , , of x3 –11x2 + 36x –36 = 0 if .
ii) If the roots of x3 + px2 + qx + r = 0 are in G.P., find the relation between p, q, r.
iii) If 2 and 3 are the roots of the equation 2x3 + mx2 13x + n = 0, find the third root.
iv) Solve the equation x3 11x2 + 38x 40 = 0, given that the ratio of two of its roots is
2 : 1.
v) If , and are the roots of the equation x(1 + x2) + x2(6 + x) + 2 = 0, then find the
value of (1 + 1 + 1).
P-2022-CBSE-P1-MATHEMATICS- QEE
8
The Method of Intervals (Wavy Curve Method)
The Method of intervals (or wavy curve method) is used for solving inequalities of the form
n1 n2 nk
x a1 x a2 .... x ak
f(x) = mp
> 0 ( < 0, 0, or 0) where n1, n2,…, nk , m1, m2, ...mp
x b1 m x b2 m
1 2
.... x bp
are natural numbers and the numbers a1, a2, … , ak ; b1, b2,…bp are any real numbers such that
aI bj , where i = 1, 2, 3,…, k and j = 1, 2, 3,…, p .
It consists of the following steps:
All zeros1 of the function f(x) contained on the left hand side of the inequality should be
marked on the number line with inked (black) circles.
2
All points of discontinuities of the function f(x) contained on the left hand side of the
inequality should be marked on the number line with un–inked (white) circles.
Check the value of f(x) for any real number greater than the right most marked number
on the number line.
From right to left, beginning above the number line (in case the value of f(x) is positive
in step (iii), otherwise, from below the number line), a wavy curve should be drawn to
pass through all the marked points so that when it passes through a simple point3, the
curve intersects the number line, and, when passing through a double point4, the curve
remains located on one side of the number line.
The appropriate intervals are chosen in accordance with the sign of inequality (the
function f(x) is positive whenever the curve is situated above the number line, it is
negative if the curve is found below the number line). Their union represents the
solution of the given inequality.
Remark:
(i) Points of discontinuity will never be included in the answer.
(ii) If you are asked to find the intervals where f(x) is non–negative or non–positive then
make the intervals closed corresponding to the roots of the numerator and let it remain
open corresponding to the roots of the denominator.
1
The point for which f(x) vanishes (becomes zero) are called function zeros e.g. x = ai.
2
The points x = bj are the point of the discontinuity of the function f(x).
3
If the exponent of a factor is odd then the point is called a simple point.
4
If the exponent of a factor is even then the point is called a double point.
x 3 x 2 x 5
Illustration 13. Let f(x) = . Find intervals, for which f(x) is positive or
x 1 x 7
negative.
Solution:
-5 -2 -1 3 7
P-2022-CBSE-P1-MATHEMATICS-QEE
9
f(x) > 0 x (–5, –2) (–1, 3) (7, )
and f(x) < 0 x (– ,–5) (–2, –1) (3, 7).
2x 1
Illustration 14. Find the set of all x for which 2
2x 5 x 2 x 1
2x 1 3x 2
Solution: We have 2
0 0
2x 5x 2 x 1 (x 1)(2x 1)(x 2)
3x 2 2
0
(x 1)(2x 1)(x 2)(3x 2)
(3x 2)2
0. …..(1)
1 2
2.3(x )(x )(x 1)(x 2)
2 3
There are five intervals x < –2, –2 < x < –1, –1 < x < – 2/3, – 2/3<x<–1/2, x>–1/2.
The inequality (1) will hold for –2 < x < –1 and for –2/3 < x < –1/2.
Hence –2 < x < –1 and –2/3 < x < –1/2.
x3
Illustration 15. The greatest integer x for which the inequality 2
0 is satisfied, is
x 9x 22
equal to
(A) – 12 (B) –11
(C) 2 (D) 3
x 3
0
x 3 0
Solution: 2
x 11x 2x 22 x 11 x 2
x2 x 6
Illustration 16. If 1 , then x belongs to the set
x 2 2 x 3
3
(A) (2, 3) (B) 1,
2
3
(C) R (D) R- 1,
2
P-2022-CBSE-P1-MATHEMATICS- QEE
10
Exercise 4.
x 1
i) Solve the inequality 2
< 1.
x 4x 3
3x2 7 x 8
ii) Solve 1 2.
x2 1
x 2 2x 3
iii) Find x R, for which .
x 2 4x 1
x2 3x 2
iv) Let f (x) = . Find the intervals for which f (x) is negative.
x2 1
x2 4
v) Let f(x) = 3 , then find the interval for which f(x) is positive.
x 1
Quadratic Expression
The expression ax2 + bx + c is said to be a real quadratic expression in x where a, b, c are real
and a 0. Let f(x) = ax2 + bx + c where a, b, c, R (a 0).
2 2
b 4ac b2 b D 2
f(x) can be rewritten as f(x) = a x =a x , where D = b –4ac
2a 2 2a 2
4a 4a
is the discrimnant of the quadratic expression. Then y = f(x) represents a parabola whose axis is
b D
parallel to the y – axis, with vertex at A , .
2a 4a
Depending on the sign of a and b2 – 4ac, f(x) may be positive, negative or zero. This gives rise to
the following cases:
x x
2
(ii) a > 0 and b – 4ac = 0
f(x) 0 x R.
In this case the parabola touches the
x – axis at one point and remains concave
upwards. x x
(iii) a > 0 and b2 – 4ac > 0.
Let f(x) = 0 have two real roots and (<).
Then f(x) > 0 x (–, )(, ),
and f(x) < 0 x (, ).
x x
x x
2
(iv) a < 0 and b – 4ac < 0
f(x) < 0 x R.
In this case the parabola always remains below
the x–axis.
P-2022-CBSE-P1-MATHEMATICS-QEE
11
2
(v) a < 0 and b – 4ac = 0
x x
f(x) 0 x R.
In this case the parabola touches the
x – axis and lies below the x – axis.
2
(vi) a < 0 and b – 4ac > 0
Let f(x) = 0 have two real roots and x x
( < ). Then f(x) < 0 x (–, )(, )
and f(x) > 0 x (, ).
Illustration 18. If f (x) = (a1x + b1)2 + (a2x + b2)2 + ......+ (anx + bn)2,
then prove that (a1b1 + a2b2 + ....+anbn)2 a12 a22 ... an2 b 2
1
b22 ... bn2 .
2 2 2
Solution: Given f (x) = (a1x + b1) + (a2x + b2) + ......+ (anx + bn) ...(1)
or f(x)= a12 a22 ... an2 x 2
2 a1b1 a2 b2 ... an bn x
b12 b22 ... bn2 . …(2)
From (1), f (x) 0 x R
from (2)
a 2
1
a22 ..... an2 x 2 2 a1b1 a2 b2 ..... an bn x
b12 b22 ..... bn2 0, x R.
Hence its discriminant D 0 [ coefficient of x2 is positive]
2
4 a1b1 a2 b2 ..... an bn 4 a12 a22 ..... an2 b b ..... b
2
1
2
2
2
n
2
a1b1 a2 b2 ..... an bn a12 a22 ..... an2 b b ..... b .
2
1
2
2
2
n
2
Illustration 19. If ax – bx + 5 = 0 does not have 2 distinct real roots, then find the minimum
value of 5a +b.
Solution: Let f(x) = ax2 –bx + 5.
Since f(x) = 0 does not have 2 distinct real roots, we have either
f(x) 0 x R or f(x) 0 x R.
But f(0) = 5 > 0 f(x) 0 x R.
P-2022-CBSE-P1-MATHEMATICS- QEE
12
In particular f(–5) 0
25a + 5b + 5 0 5a + b – 1.
Hence the least value of 5a + b is – 1.
2
Illustration 20. If is a positive integer and the roots of the equation 6x –11x + = 0 are
rational numbers, then the smallest value of is
(A) 4 (B) 5
(C) 6 (D) none of these
Solution: Since roots of 6x2 –11x + = 0 are rational and coefficients are rational
2
121 –24 = k k I
Clearly k is odd, let k = 21 + 1
1( 1 1)
=5– , for minimum value of , 1 = 3 min = 3.
6
Exercise 5.
i) a, b, c are three distinct real numbers and they are in G.P. If a + b + c = xb, then
prove that x < –1 or x > 3.
x 2 bc
ii) Show that if x is real, the expression has no value between b and c.
2x b c
x 1
iii) If x is real, show that 2 always lies in the interval , 1 .
x 5x 9 11
c b
iv) If the equation ax2 + bx + c = 0 has non-real roots, prove that 1 0
a a
v) If the roots of the equation x2 2ax + a2 + a 3 = 0 are less than 3 then find the set
of all possible values of a.
P-2022-CBSE-P1-MATHEMATICS-QEE
13
Exactly one of the roots lies in the given interval (k1, k2) if f(k1) . f(k2) < 0.
A given number k will lie between the roots if a.f(k) < 0.
In particular, the roots of the equation will be of opposite signs if 0 lies between the roots
a.f(0) < 0. It also implies that the product of the roots is negative.
Illustration 21. Find the values of the parameter a for which the roots of the quadratic equation
2
x + 2(a – 1)x + a + 5 = 0 are
(i) real and distinct, (ii) equal,
(iii) not real, (iv) opposite in sign,
(v) equal in magnitude but opposite in sign,
(vi) positive, (vii) negative,
(viii) such that one root is greater than 3, and the other is smaller than 3,
(ix) greater than 3, (x) smaller than 3,
(xi) such that exactly one root lies in the interval (1, 3),
(xii) such that both the roots lie in the interval (1, 3),
(xiii) such that one root is greater than 3 and the other root is smaller than 1.
P-2022-CBSE-P1-MATHEMATICS- QEE
14
a < –8/7 and a (–, –1) (4, )
8
a , .
7
(ix) In this case
B
– 3 , A.f (3) > 0 and D 0
2A
–(a – 1) > 3, 7a + 8 > 0 and a (–, –1] [4, )
a < –2, a > –8/7 and a (–, –1] [4, ).
Since no value of a can satisfy these conditions simultaneously, there can be no
value of a for which both the roots will be greater than 3.
(x) In this case
B
– < 3, A.f(3) > 0 and D 0
2A
8 8
a > –2, a > – and a (–, –1] [4, ) a , 1 4, .
7 7
(xi) In this case f(1).f(3) < 0 and D 0
4 8
(3a + 4)(7a + 8) < 0 and a (–, –1] [4, ) a , .
3 7
B
(xii) In this case 1 < – < 3, A.f(1) > 0, A.f(3) > 0, D 0
2A
1 < –1(a –1) < 3, 3a + 4 > 0, 7a + 8 > 0, a (–, –1] [4, )
–2 < a < 0, a > –4/3, a > –8/7, a (–, –1] [4, )
8
a , 1 .
7
(xiii) In this case D > 0 and f(1) < 0 i.e. 3a + 4 < 0
a < –4/3
and f(3) < 0 i.e. 7a + 8 < 0
a < –8/7 and a (–, –1) (4, ).
Combining we get, a (–, –4/3).
Illustration 22. If is a root of the equation ax2 + bx + c = 0 and is a root of the equation
–ax2 + bx + c = 0, then prove that there will be a root of the equation
a 2
x bx c 0 lying between and .
2
a 2
Solution: Let f(x) = x bx c
2
a a
f() = 2 + b + c = a2 + b + c – 2
2 2
a 2
=– (As is a root of ax2 + bx + c = 0).
2
a 2 2 3 2
Also f() = + b+ c = – a + b + c + a
2 2
3
= a2 (As is a root of – ax2 + bx +c = 0).
2
P-2022-CBSE-P1-MATHEMATICS-QEE
15
3 2 2 2
Now f(). f() = a < 0 f(x) = 0
4
has one real root between and .
9
Illustration 23. Find the values of ‘a’ for which 4t – (a – 4) 2t + a < 0 t (1, 2).
4
t 2 9
Solution: Let 2 =x and f(x) = x – (a – 4) x + a.
4
We want f(x) < 0 x (21, 22) i.e. x (2, 4).
(i) Since coefficient of x2 in f(x) is positive, f(x) < 0 for some x only when roots of
f(x) = 0 are real and distinct
D > 0 a2 –17a +16 > 0 a < 1 or a > 16. ... (1)
(ii) Since we want f(x) < 0 x (2, 4), one of the roots of f(x) = 0 should be less
than 2 and the other must be greater than 4 i.e. f(2) 0 and f(4) 0.
a –48 and a 128 /7, which is not possible.
Hence no such ‘a’ exists.
Illustration 24. Find all the values of the parameter a for which the inequality
4x –a2x – a +3 0 is satisfied by at least one real x.
Illustration 25. Let S denote the set of all possible values of parameter a for which
x+ x2 a a posses solution, then find set S.
Solution: x+ x2 a a …(1)
2 2 2
x + a = x 2ax + a
2x = a 1
Putting it in (1), we get (a 1) + |a + 1| = 2a a 1
S = [1, ).
P-2022-CBSE-P1-MATHEMATICS- QEE
16
Exercise 6.
i) For what values of a R, the quadratic equation
(a2 + 1)x2 –(a + 1)x + (a2 –a –2) = 0 will have roots of opposite sign.
2
ii) For what values of a, does the equation ax –(a + 1)x + 3 = 0, have roots lying
between 1 and 2.
x 2 mx 2
iii) For what value of m, 3 2 for all x R.
x2 x 1
2
iv) If the roots of the equation x + 2(a 3)x + 9 = 0 lies between 6 and 1, find the
integral part of a.
v) Find the least integral value of k for which (k 2)x2 + 8x + k + 4 > 0 x R.
P-2022-CBSE-P1-MATHEMATICS-QEE
17
Miscellaneous Exercise
i) If the constant term in a quadratic equation is zero, prove that one of its roots is
zero.
2
ii) If r is the ratio of the roots of the quadratic equation ax + bx + c = 0, prove that
(1 + r)2 ac = b2r.
x 2 bx m 1
iii) For what value of m ( 1) will the equation have roots equal in
ax c m 1
magnitude but opposite in sign.
2 2
iv) If the roots of the equation x lx + m = 0 differ by 1, prove that l = 4m + 1.
3
v) Prove that the roots of the equation 2x2 x = 0 are irrational.
2
a x ax a
vii) Solve for x: .
ax ax x
x+1
viii) Solve for x: 7 + 71x = 50.
ix) Show that the equation (x a) (x b) = c2 has real roots.
x) Prove that the roots of the equation (x a) (x b) + (x b) (x c) + (x c) (x a) = 0
are equal if and only if a = b = c.
xi) Let a, b, c be real numbers such that a + 2b + c = 4. Find max(ab + bc + ca) .
xii) If and are the roots of the equation ax2 + bx + c = 0 express the roots of the
equation a3x2 –ab2x + b2c = 0 in the terms of and .
xiii) If |2x –3| + |2x +3| = 6, then find x.
xiv) Find the values of the parameter a 0 for which one of the roots of the quadratic
equation x2 –x + 3a = 0 is double of one of the roots of the equation x2 –x + a = 0.
xv) If the roots of the equation x2 + px + q = 0 differ from the roots of the equation
2
x + qx + p = 0 by the same quantity, then show that p + q + 4 = 0.
P-2022-CBSE-P1-MATHEMATICS- QEE
18
ANSWERS TO EXERCISES
Exercise 1:
Exercise 2:
(iii) 2 (v) 0
Exercise 3:
(i) 6, 3, 2
3 3
(ii) rp = q
5
(iii) 2
(iv) 4, 2, 5
(v) 1/2
Exercise 4:
(i) (–, 1) (1, 3) (4, )
(ii) [1, 6]
1
(iii) ( , 2) , 1 (4, )
4
(iv) ( 1, 2) { 1}
(v) (2, 1) (2, )
Exercise 5:
(v) (, 2)
Exercise 6:
(i) –1 < a < 2
(ii) no real values
(iii) 1m2
(iv) 6
(v) 5
Miscellaneous Exercise:
a b
(iii) , (vii) x = a, (viii) x = 1, 1.
ab
3 3
xiii) − x xiv) a = –2 xv) p+q+4=0
2 2
P-2022-CBSE-P1-MATHEMATICS-QEE
19
SOLVED PROBLEMS
Subjective:
Level – 0
Problem 1. The real numbers x1, x2, x3 satisfying the equation x3 – x2 + x + = 0 are in A.P.
Find the intervals in which and lie.
Problem 2. Find the value of ‘a’ for which ax2 + (a – 3)x + 1<0 for at least one positive real x.
x x
x x x Fig. 2 x
Fig. 1 Fig. 3
y
y
x x x x
Fig. 4 Fig. 5
2
For this D > 0, i.e. (a – 3) – 4a > 0 a < 1 or a > 9. … (1)
P-2022-CBSE-P1-MATHEMATICS- QEE
20
Both the roots are non–positive sum 0 and product 0
a 3 … (2)
at least one roots is positive if a < 3, and (1) is satisfied.
Combining (1) and (2) , we get a < 1 so that, 0 < a< 1.
Case (iii) If a = 0, the given inequality is satisfied for at least one x > 0.
Hence the required set of values of a is (–, 1).
Problem 3. Find the conditions on a, b, c, d such that the equations 2ax3 + bx2 + cx + d = 0
and 2ax2 + 3bx + 4c = 0 have a common root.
x 2 34 x 71
Problem 4. Find the range in which the value of the function lies for all real
x 2 2x 7
values of x.
x2 34x 71
Solution: Let =y
x 2 2x 7
x2(y – 1) – 2x(17 – y) + 71 – 7y = 0.
For real values of x, (17 – y)2 – (y – 1)(71 – 7y) 0
8y2 – 112y + 360 0 y2 – 14y + 45 0
(y – 9)(y – 5) 0 y 5 or y 9.
Thus for real values of x, the given expression is either 5 or 9.
Level – I
Problem 5. Let a, b, c be real numbers with a 0 and let , be the roots of the equation
ax2 + bx + c = 0. Express the roots of a3x2 + abcx + c3 = 0 in terms of , .
b c
Solution: Here + = and = so that
a a
a3x2 + abcx + c3 = 0 leads to
3
2 b c c
x + x 0
a a a
or x2 ( + ) x + 33 = 0
i.e. x2 (2 + 2)x + 33 = 0
or (x 2) (x 2) = 0
x = 2, 2.
P-2022-CBSE-P1-MATHEMATICS-QEE
21
Alternative Solution:
Dividing by c2, the second equation can be written as
2
a a
a x b x c 0
c c
ax c c
so that , or x = , = 2, 2.
c a a
P-2022-CBSE-P1-MATHEMATICS- QEE
22
2
Problem 8. Let a, b, c be real. If ax + bx + c = 0 has two real roots and where
c b
< – 1 and >1, then show that 1 0.
a a
Solution: – 1 + E1 = – 1 where E1 0.
Also 1 – E2 = 1 where E2 0.
c b
Now 1+ 1 1 1 E1 1 E 2 E2 E1
a a
= 1 – 1 – E1 – E2 – E1E2 + E2 – E1
= – E1 –E2 – E1E2 + E2 – E1 if E2 E1.
= – E1 – E2 – E1E2 + E1 – E2 if E1 E2.
Hence L.H.S = – 2E1 – E1E2 or – 2E2 – E1E2.
c b
In both the cases 1 0. (E1, E2 > 0)
a a
Alternative Solution:
b c
Let f(x) = x2 + x + .
a a -1 1
From graph, f (– 1) < 0 and f (1) < 0
c b c b c b
1+ < 0 and 1 + + <0 1+ + < 0.
a a a a a a
Problem 9. If , are the roots of x2 + px + q = 0, and also of x2n + pnxn + qn = 0, and if is a
root of xn + 1 + (x + 1)n = 0, then prove that n must be an even integer,
where n n.
Problem 10. For what value of , 1 lies between the roots of the quadratic equation
3x2 – 3 sin x – 2 cos2 = 0.
P-2022-CBSE-P1-MATHEMATICS-QEE
23
Level – II
Solution: x+y+z=6
xy + yz + zx = 7 ….(1)
(1) z = 6 – x – y ….(2)
Putting the value of z in (2)
xy + y(6 – x – y) + x(6 – x – y ) = 7
or, y2 + y(x –6) + x2 – 6x + 7 = 0
since y is real, so
(x – 6)2 – 4 (x2 – 6x + 7) 0
or, 3x2 – 12x – 8 0
6 2 15 6 2 15
x
3 3
since (1) and (2) are symmetrical in x, y and z. So all the variables lie in the
interval
6 2 15 6 2 15
,
3 3
3 2 2
Problem 13. If equations ax + 2bx + 3cx + 4d = 0 and ax + bx + c = 0 have a non-zero
2 2
common root, then prove that (c – 2bd) (b – ac) 0.
P-2022-CBSE-P1-MATHEMATICS- QEE
24
Problem 14. If the equation x4 – 4x3 + ax2 + bx + 1 = 0 has four positive roots, then find a
and b .
Solution: Let the roots be x1, x2, x3 and x4 . then
x1+ x2 + x3 + x4 = 4 and x1 x2 x3 x4 = 1
A.M. = G.M ( for roots) x1= x2 = x3 = x4
xi = 1, i x4 – 4 x3 + ax2 + bx +1 = (x – 1)4
a= 6, b=-4.
2 2
Problem 15. Find the range of real values of x and y if 2x + 6xy + 5y = 1
Solution: 2x2 + 6xy + 5y2 = 1 …. (1)
Equation (1) can be rewritten as
2x2 + (6y) x + 5y2 – 1 = 0
Since x is real
2 2
36y – 8(5y –1) 0
2
y 2 2 y 2
Equation (1) can also be rewritten as 5y2 +(6x)y +2x2-1 = 0
Since y is real, 36x2 – 20 ( 2x2 –1) 0
36x2 40x2 + 20 0
4x2 20 x2 5 5 x 5 .
Problem 16. If a, b, c are in G.P, then the equations ax2 + 2bx + c = 0 and dx2 + 2ex + f = 0
d e f
have a common root, then prove that , , are in A.P.
a b c
Solution: x 2
4x 3 x 2 6x 8 0
x 2 1 2x 2 3 3 8 0
2
Discriminant. D 4 2 3 4 1 3 8
D 4 2 1
if R then D 0
so root of given quadratic always real
Solution: x 2 2ax b c x R
2
x 2ax b c 0 , D 4a2 4 b c 0 a2 b c
P-2022-CBSE-P1-MATHEMATICS-QEE
25
Objective:
Level – 0
True / False
Solution: True
4x a 3 2x a 4 0
x 0, Let y 2x
y2 a 3 y a 4 0
The roots of Quadratic must lie b/w (0, 1]
Problem 2. If the equation 5 x 2 10x log1 / 5 a 0 has real roots then the minimum
1
value of a is .
56
Solution: False
5x 2 10x log1/ 5 a 0 has real roots.
100 20log1/ 5 a 0
5
1
log1/ 5 a 5 log1/ 5
5
1
a 0 and a 5
5
1
minimum value of a
55
P-2022-CBSE-P1-MATHEMATICS- QEE
26
i.e. f(1) 0 and f(1) 0
1 + 3 + a 0 and 1 – 3 + a 0
a 2 and a 2
a (2, 2)
1
Problem 5. If the expression mx 1 is non-negative for all positive real x ,then the
x
minimum value of m is ______
Solution: 1/4
We know that ax2 + bx + c 0 if a > 0 and b2 – 4ac 0.
1 mx 2 x 1
So, mx – 1 + 0 0
x x
mx2 – x + 1 0 as x > 0.
Now, mx2 – x + 1 0 if m > 0 and 1 – 4m 0
or if m > 0 and m 1/4.
Thus, the minimum value of m is 1/4.
Problem 6. If x2 – 4x + log1/2a = 0 does not have two distinct real roots, then maximum value
of a is _______
Solution: 1/16
Since x2 – 4x + log1/2a = 0 does not have two distinct real roots,
discriminant 0
16 – 4 log1/2 a 0 log1/2 a 4 a 1/16.
x2 1
Problem 7. The largest negative integer which satisfies 0 is ______
x 2 x 3
Solution: –2
By wavy curve method .
x2 1
0 x (–, –1) (1, 2) (3, ).
x 2 x 3
Therefore largest negative integer is –2.
Level – I
Problem 8. If and are the roots of the equation 2x2 – 3x – 6 = 0, then equation whose
2 2
roots are + 2, + 2 is
(A) 4x2 + 49x + 118 = 0 (B) 4x2 – 49x + 118 = 0
2
(C) 4x – 49x – 118 = 0 (D) x2 – 49x + 118 = 0
Solution: B
P-2022-CBSE-P1-MATHEMATICS-QEE
27
Here + = 3/2, = –6/2 = –3 so that
2 2 2 49
S = + + 4 = ( + ) – 2 + 4 = ,
4
2 2 2 2 2 2 2 118
P = + 2( + ) + 4 = + 4 + 2[( + ) – 2 ] = .
4
2 49 118 2
Therefore, the equation is x – x 0 4x – 49x + 118 = 0.
4 4
2 2
Alternate: Let y = x +2, then 2x – 3x – 6 = 0
(3x)2 = (2x2 – 6)2 [2(y – 2) – 6]2 = 9(y – 2) 4y2 – 49y + 118 = 0.
Solution: B
Suppose the equation x2 – px + q = 0 has the roots + 1
and then + 1+ = p 2 = p – 1 . . . . (1)
and (+1) = q 2 + = q. . . . .. (2)
Putting the value of from (1) in (2), we get
2
p 1 p 1 2
q (p – 1) + 2(p – 1) = 4q
4 2
p2 – 1 = 4q p2 = 4q + 1.
Alternative: Let and be the roots. | – | = 1 ( + )2 – 4 = 1
p2 – 4q = 1, or p2 = 1+ 4q.
Problem 10. The quadratic equation whose roots are A.M. and H.M. between the roots of the
equation ax2 + bx + c = 0 is
(A) abx2 + (b2 + ac)x + bc = 0 (B) 2abx2 + (b2 + 4ac)x + 2bc = 0
(C) 2abx2 + (b2 + ac)x + bc = 0 (D) none of these
Solution: B
Let (, ) be the roots of the given equation, then
b c
+ = – , =
a a
2 2 2
Required equation is x – x =0
2 2
2abx2 + (b2 + 4ac)x + 2bc = 0.
Problem 11. The expression ax2 + by2 + cz2 + 2ayz + 2bzx + 2cxy can resolved into rational
factors, then a3 + b3 + c3 is
(A) abc (B) a + b + c
(C) 1 (D) none of these
Solution: D
The given expression is ax2 + by2 + cz2 + 2ayz + 2bzx + 2cxy
P-2022-CBSE-P1-MATHEMATICS- QEE
28
x 2 y
2
y x x y
= z2 a b c 2a 2b 2c
z z z z z z
x y
= z2 [aX2 + bY2 + 2cXY + 2bX + 2aY + c] … (1) (where X = ,Y )
z z
The given expression can be resolved into rational factors if the expression within
brackets in (1) is expressible into rational factors, the condition for which is abc +
2abc a. a2 b. b2 c. c2 = 0 a3 + b3 + c3 = 3abc.
2
Problem 12. Let p and q be the roots of the equation x – 2x + A = 0 and let r and s be the
2
roots of the equation x – 18x + B = 0. If p < q < r < s are in arithmetic
progression then the values of A and B are given by
(A) A = 3, B = 77 (B) A = 3, B = 7
(C) A = –3, B = 77 (D) A = 3, B = –7
Solution: C
Let the four numbers in A.P. be a – 3d, a – d, a + d and a + 3d corresponding to
the numbers p, q, r and s respectively.
We have p + q = 2, pq = A for x2 – 2x + A = 0
and r + s = 18, rs = B for x2 – 18x + B = 0.
Therefore p + q +r + s = 4a = 20 a = 5.
Also p + q = 2 10 –4d = 2 d = 2.
Hence the numbers are –1, 3, 7, 11.
Thus we have pq = A = –3 and rs = B = 77.
Level – II
Problem 13. If roots of the equation 2 x 2 4x 2 sin 1 0 are of opposite sign, then
belongs to
5 5
(A) , (B) 0, ,
6 6 6 6
13 17
(C) , (D) none of these
6 6
Solution: B
2x 2 4x 2 sin 1 0
coefficient of x 2 is positive
f(0) 0
2sin 1 0
1
sin
2
5
0, ,
6 6
P-2022-CBSE-P1-MATHEMATICS-QEE
29
2
Problem 14. The set of values of p for which the roots of the equation 3x + 2x + p(p – 1) = 0
are of opposite sign is
(A) (–, 0) (B) (0, 1)
(C) (1, ) (D) (0, )
Solution: B
Since the roots of the given equation are of opposite signs, product of the
roots< 0
p p 1
0 p(p – 1) < 0 0 < p < 1.
3
For real roots, 4 12p(p 1) 0 3p2 3p 1 0
1 7 1 1 7
1 p 0 < p < 1.
2 3 2 2 3
mx 2 3x 4
Problem 15. If the inequality 5 is satisfied for all x R, then
x2 2 x 2
(A) 1 < m < 5 (B) –1 < m < 5
71
(C) 1< m < 6 (D) m <
24
Solution: D
We have x2 + 2x + 2 = (x + 1)2 + 1 > 0, x R.
mx2 3x 4 2 2
Therefore, 2 5 mx + 3x + 4 < 5(x + 2x + 2)
x 2x 2
(m – 5)x2 – 7x – 6 < 0, x R.
This is possible if D = b2 – 4ac = 49 + 24(m – 5) < 0 and m – 5 < 0
71
m< .
24
Problem 16. The value of p, for which both the roots of the equation 4x2 20px + (25p2 + 15p
66) = 0 are less than 2, is
4 4
(A) , 2 (B) 1,
5 5
(C) (2, ) (D) ( , 1)
Solution: D
22
(i). D 0 15p – 66 0 p …(1)
5
2
(ii) 4 f (2) > 0 p – p – 2 0 p (–, –1) (2, ) …(2)
b 20p 4
(iii) – <2 <4p< …(3)
2a 4 5
Taking intersection of (1), (2) and (3), we get p (–, –1).
P-2022-CBSE-P1-MATHEMATICS- QEE
30
Solution: D
Since x2, 5|x| and 6 are positive so x2 + 5|x| + 6 = 0 does not have any real root.
Therefore sum does not exist.
Problem 18. If c > 0 and 4a + c < 2b then ax2 – bx + c = 0 has a root in the interval
(A) (0, 2) (B) (2, 4)
(C) (0, 1) (D) (–2, 0)
Solution: A
Let f(x) = ax2 – bx + c
f(0) = c > 0 and f(2) = 4a –2b + c < 0 so that f(x) = 0 has a root in the interval
(0, 2).
Solution: We have ax 2 bx c 0 x R
2
a1x b1x c1 0 x R
2
So, b 4ac 0 , a 0, c 0
and b12 4a1c1 0 , a1 0, c1 0
b2 4ac and b12 4a1c1
b2b12 16a1c1ac
2
bb1 4a1acc1 12a1ac c1 (positive)
b2b12 4aa1cc1 is discriminate of aa1x2 bb1x cc1 0
If may be negative, zero, or positive that way we can’t say nature of the root.
P-2022-CBSE-P1-MATHEMATICS-QEE
31
Problem 21. 5
2
log5 x 9x 24
x 1
(A) x R (B) x ( 0, )
(C) x ( -, 0) (D) none of these
Solution: A, B, C
5
5
log x2 9x 24
x 1 x – 9x +2 4 > x –1
2
2 2
x – 10x + 25 > 0 ( x – 5) > 0.
Which is true for x R.
Problem 22. Let f(x) is a quadratic expression with positive integral coefficients such that for
every , R, > , f x dx > 0. Let g(t) = f(t) f(t), and g(0) = 12, then
(A) 16 such quadratics are possible
(B) f(x) = 0 has either no real root or distinct roots
(C) minimum value of f(1) is 6
(D) maximum value of f(1) is 11
Solution: A, C, D
Let f(x) = ax2 + bx + c. Given f x dx > 0 for all > , so f(x) = 0 has no real
roots.
Now g(0) = f(0) f(0) = 2ac ac = 6.
a and c are positive integers. So the possible values are a = 6, c = 1; a = 1,
c = 6; a = 2, c = 3; a = 3, c = 2.
Again b2 < 4ac or b2 < 24. So b can be 1, 2, 3, 4.
In all 4 4 = 16 such quadratic are possible.
Now f(1) = a + b + c, then maximum value of (1) = 7 + 4 = 11 and minimum value
of f(1) = 5 + 1 = 6.
Problem 23. 0 < c < b < a and the roots , of the equation cx2 + bx + a = 0 are imaginary,
then
|||| 1 1
(A) = || || (B)
4 || ||
1 1
(C) 2 (D) none of these
|| ||
Solution: B, C
Since roots are imaginary.
So, discriminant < 0
b i 4ac b2
=
2c
b i 4ac b2
=
2c
b2 4ac b2 a
|| = || = 2
2
1.
4c 4c c
P-2022-CBSE-P1-MATHEMATICS- QEE
32
Assertion/Reasoning
Direction: Read the following carefully and give the answers as follows:
Option (A) if both the statements are TRUE and STATEMENT-2 is the correct explanation of
STATEMENT-1
Option (B) if both the statements are TRUE but STATEMENT-2 is NOT the correct
explanation of STATEMENT- 1
Option (C) if STATEMENT-1 is TRUE and STATEMENT-2 is FALSE.
Option (D) if STATEMENT-1 is FALSE and STATEMENT-2 is TRUE.
Problem 24. Statement–1: Least degree of polynomial expression with integer coefficients
1/7 1/7
3 5
one of whose zeros is is 7.
5 3
2
Statement–2: x (3y – 5) = y + 1 has no integral solution.
Solution: C
1/7
5
For A, Let t
3
and T = t + t –1
t 3 t 3 T 3 3T
t 5 t 5 T 5 5 T 3 3T 10T T 5 5T3 5T
and 7
t t 7
T 7
7 T 5
5T 3 5T 21 T 2 3T 35T
7 5 3
T 7T 14T 7T
where T is a zero of the polynomial.
3 5
t 7 7t 5 14t 3 7t and multiplying it by 15 we get the required result.
5 3
deg. 7
A is true.
In R. one of solution (1, 1) sin a(1, 1) satisfy it
R is false.
Problem 25. Statement–1: If f(x) = x4 + ax3 + bx2 + cx + 8, a, b, c R and f(x) = 0 has 4 real
roots, then f(x) = 0 has then real roots.
Statement–2: If f(x) = 0 of n degree has n real roots then
f ’ (x) = 0 (n – 1) real roots.
Solution: A
A and R are correct and R is the correct explanation of A.
Comprehension–I
If a group of p consecutive terms is missing from the polynomial equation f(x) = 0,
then
(i) If p is even, the polynomial equation f(x) = 0 has at least p imaginary roots.
(ii) If p is odd, there are at least (p + 1) or at least (p – 1) imaginary roots of f(x)
= 0, according as the terms which immediately preceed and follow the
group, have like or unlike signs (1 – 4):
18 17 2
Problem 26. The no. of at least imaginary roots of the equation x + 2x – 7x + 13x + 5 = 0
is
(A) 15 (B) 14
(C) 16 (D) 13
P-2022-CBSE-P1-MATHEMATICS-QEE
33
Solution: B
14 Terms are missing between the terms 2x17 and -7x2. Hence at least no. of
imaginary roots = 14.
Problem 27. The no. of maximum real roots of the equation 2x88 + 3x87 – 13x2 + 5x + 9 = 0 is
(A) 3 (B) 5
(C) 4 (D) 2.
Solution: D
At least no. of imaginary roots = 84.
Max. real roots = 88 – 84 = 4.
Problem 28. The no. of at least imaginary roots of the equation x8 + 7x2 + 43x + 7 = 0 is
(A) 4 (B) 6
(C) 5 (D) 2.
Solution: B
No. of missing terms between x8 and 7x2 is 5. Sign of x8 and 7x2 are same.
at least no. of imaginary roots = (5 + 1) = 6.
Problem 29. The no. of maximum real roots of the equation
3x17 + 4x16 + 3x15 – x3 + 5x2 + 6x + 8 = 0 is
(A) 7 (B) 8
(C) 10 (D) 12.
Solution: A
No. of missing terms between 3x15 and –x3 is 11. But sign of 3x15 and –x3 are
different. Therefore at least no. of imaginary roots = 11 – 1 = 10.
No. of max. real roots = 17 – 10 = 7.
Comprehension–II
Solution: A
If f(x) touches x-axis at
3, 0 and also touches at 3, 0
roots of f(x) are 3, 3, 3, 3 .
Problem 31. Suppose f(x) touches x-axis at only one point then the point of touching is
(A) always a rational quantity
(B) may or may not be a rational number
(C) never a rational number
(D) none of these
Solution: A
P-2022-CBSE-P1-MATHEMATICS- QEE
34
f(x) touches x-axis at only one irrational point then f(x) = (x )2 g(x) where is
irrational
coefficients of f(x) cannot be rational
If f(x) is moving rational coefficients then point of touching is also rational.
Matching Type
P-2022-CBSE-P1-MATHEMATICS-QEE
35
Problem 33. If f(x) = x + ax + bx + c = 0 has three distinct integral roots and (x + 2x + 2)3 +
3 2 2
Solution: D
(P). The roots are real and distinct k < 32
f(3) f(4) < 0 (k – 30)(k – 32) < 0
f(4) f(5) < 0 (k – 32)(k – 30) < 0
Integral value of k is 31.
2
(Q). q 4p
If p = 1, q can be either 2, 3, 4, 5
If p = 2, q can be either 3, 4, 5
If p = 3, q can be either 4, 5
If p = 4, q can be 5
If p = 5, q can be 5
P-2022-CBSE-P1-MATHEMATICS- QEE
36
possible ordered pairs (p, q) = 11.
2 2
(R). S = (1 + cot x) + 25(1 + tan x)
= 26 + (cotx – tanx) + 2 = 28 + (cotx – tanx)2
2
Numerical based
Problem 35. The least positive integral value of real so that the equation (x – a)(x – c)(x – e)
+ (x – b)(x – d) = 0, (a > b > c > d > e) has distinct real roots is
______________
Solution: 1
f x x a x c x e x b x d
f a a b a d
f b b a b c b e 0
f c c b c d
f d d a d c d e 0
f e e b e d
If 0 f a 0 , a root lies between b and a.
If < 0 f e < 0, a root lies between e and d.
Always a root lies between d and b all roots are real and distinct as exactly
two can’t be real. If = 0 roots are a, c and e.
n
2
Problem 36. The number of real roots of the quadratic equation x k 0 n 1 is.
k 1
Solution: 0
n
(x k)2 0
k 1
number of real root is zero.
Problem 37. The least integral value of k such that k 2 x 2 8 x k 4 is positive for all
real values of x is
Solution: 5
k 2 x 2 8x k 4 0 x R
D = 64 – 4(k – 2) (k + 4) 0 and k – 2 0
(k + 56) (k – 4) 0 k2
k 6 or k 4
k4
k=5
P-2022-CBSE-P1-MATHEMATICS-QEE
37
Decimal Type
Problem 38. If the equation x4 + px3 + qx2 + rx + 5 = 0 has four positive real roots, then find
the minimum value of pr.
Solution: 80.00
Let , , , be the four positive real roots of the given equation. Then
+ + + = – p.
+ + + + + = q.
+ + + = –r.
= 5.
Using A.M G.M
4
4 33 3 3 5
4 4
p r
5 pr 80 minimum value of pr = 80.
4 4
Solution: 0.33
1
Let a = , so that the given equation becomes x2 – 4bx + 9b = 0.
b
This equation has integral roots if b is an integer and 16b2 – 36b is a perfect
square
Let b(4b – 9) = k2 4b2 – 9b – k2 = 0
2
9 81
2
2b k (8b – 9)2 – 16k2 = 81
4 16
(8b – 9 – 4k) (8b – 9 + 4k) = 81 = 327.
Since b and k are integers, 8b – 9 – 4k = 3 and 8b – 9 + 4k = 27
1
16b – 18 = 30 b = 3 a = = 0.33.
3
For any other factorization of 81, b will not be an integer.
P-2022-CBSE-P1-MATHEMATICS- QEE
38
CHAPTER PRACTICE PROBLEM
Subjective:
Level – I
1. If , are the roots of the equation ax2 + bx + c = 0, then find the equation whose roots
1 1
are , .
a b a b
2.
If 2 i 3 is a root of the equation x 2 px q 0 where p and q are real,
then find (p, q).
3. If the roots of the equation x 2 a2 8x 6a are real, then find the interval of a.
Level – II
4. In a triangle PQR, R . If tan(P/2) and tan(Q/2) are the roots of the equation
2
ax 2 bx c 0 where a 0, then find the relation in a, b and c.
prove that
B 2
4AC = (A/a)2.
b 2
4ac
Objective:
3.
The values of ‘a’ for which the quadratic equation 3x 2 2 a 1 x a2 3a 2 0
possesses roots of opposite sign are
(A) 1< a < 2 (B) a (2, )
(C) 1 < a < 3 (D) None of these
P-2022-CBSE-P1-MATHEMATICS-QEE
39
2
4. In copying a quadratic equation of the form x px q 0 a student wrote the coefficient
of x incorrectly and the roots were found to be 3 and 10,another student wrote the same
question but he wrote the constant term incorrectly and thus he found the roots to be 4
and 7. The roots of the correct equation are.
(A) 5, 6 (B) 4, 6
(C) 4, 5 (D) None of these
2
6. The sum of the real roots of the equation x 2 x 2 2 0 , is
(A) 2 (B) 6
(C) 4 (D) 8
x 2 .2
x 3 2 x 3 4
7. The number of negative integral solution of x 2 .2x 1 2 2x l is
(A) 4 (B) 2
(C) 1 (D) 0
8 If c, d are the roots of the equation x a x b k 0. , then the roots of the equation
x c x d k 0 are
(A) c, d (B) a, c
(C) b, d (D) a, b
9. The values of ‘m’ so that the equations 3x 2 2mx 4 0 and x x 4m 2 0 may
have a common root, is
1 1
(A) (B)
2 2 2
1 1
(C) (D)
2 2
1. Let A, G and H be the arithmetic mean, geometric mean and harmonic mean of two
positive numbers a and b, then the quadratic equation whose roots are A and H may be
(A)
Ax 2 A 2 G2 x AG2 0
(B) Hx 2 H2 G2 x HG2 0
(C) Ax 2 A 2
H x AH
2 2
0 (D) all above
2. Let the positive numbers a, b, c be in G.P. and be a complex cube root of unity then the
equation ax 2 bx c 0 must have
(A) ratio of roots as 1 : (B) ratio of roots as b : ac
(C) real roots (D) imaginary roots
P-2022-CBSE-P1-MATHEMATICS- QEE
40
2x 1
3. If S is the set of all real x such that is positive, then S contains
2x 3x 2 x
3
3 3 1
(A) , (B) ,
2 2 4
1 1 1
(C) , (D) , 3
2 4 2
O X
(x1,0) (x2,0)
Numerical Based
1
7. If x 7 4 3 , then x
x
P-2022-CBSE-P1-MATHEMATICS-QEE
41
ASSIGNMENT PROBLEMS
Subjective:
Level – 0
2. (a) For what values of m, will the following equations have equal roots:
(i) 3m 1 x 2 2 m 1 x m 0 ,
(ii) m 1 x 2 2(m 3)x m 8 0 .
(b) If the roots of the equation p q r x 2 q r p x r p q 0 are equal, show
1 1 2
that
.
p r q
(c) Determine the values of m and n, m, n > 0, such that both the equations
x 2 mx n 0 and 2x 2 4x mn 0 have equal roots.
3. A number exceeds its positive square root by 12. Find the number.
4. Find x, if x = 30 30 30 ...........to
6. If , are the roots of the equation x 2 px q 0 and 4 , 4 are the roots of the
equation x 2 rx s 0 , show that x 2 4qx 2q2 r 0 has real and distinct roots.
P-2022-CBSE-P1-MATHEMATICS- QEE
42
7. Find the value of a so that the equations x 2 11x a 0 and x 2 14x 2a 0 have a
common root.
10. Find the condition that one root of ax 2 bx c 0 be the reciprocal of a root
of px 2 qx r 0 .
11. Prove that the arithmetic mean of the roots of x2 – 2ax + b2 = 0 is equal to the geometric
2 2
mean of the roots of the equation x – 2bx + a = 0, and vice–versa.
12. If the roots of the equation x2 – px + q = 0 differ by unity, then prove that, p2 – 4q = 1
14. If the roots of the equation (b – c)x2 + (c – a)x + (a – b) = 0 be equal, then prove that a, b,
c are in arithmetic progression.
15. If c > 0 and 4a + c < 2b then find the interval in which one root of equation ax2– bx + c = 0
lies.
16. If the sum of roots of quadratic equation ax2+ bx + c = 0 is equal to the sum of the
b2 bc
squares of their reciprocals then find .
ac a2
17. Find the quadratic equation whose roots are sec2 and cosec2 .
18. Find the values of ‘a’ for which the quadratic expression x2 ax + 4 is nonnegative for
all real values of x.
19. Let S be the set of values of ‘a’ for which 2 lie between the roots of quadratic equation
2
x + (a + 2) x – (a + 3) = 0. Then find the set S.
20. If one root of equation x2 − 3ax + f(a) = 0, is double of the other then find f(x).
2 2
21. If both roots of the equation x − 2ax + a − 1 = 0 lies between − 3 and 4, then find [a],
where [.] denotes greatest integer function.
2 2
22. Find the nature of the roots of equation (ax + bx + c)(ax – dx – c) = 0, x 0.
2 2
23. Find the sum of the non-real roots of the equation (x + x 2)(x + x 3) = 12.
24. If both the roots of the quadratic equation x2 6ax + 9a2 2a + 2 = 0 exceed 3, then find
set of all possible values of a.
P-2022-CBSE-P1-MATHEMATICS-QEE
43
Level – I
tan x
1. Show that the value of , whenever defined, never lies between 1/3 and 3.
tan3x
2. For what integral values of a, the equation x2 x(1 a) – (a + 2) = 0 has integral roots.
Find the roots
3. For what value of , 1 lies between the roots of quadratic equation 3x23sinx 2cos2=0.
2
4. The coefficient of x in the quadratic equation x + px + q = 0 was taken as 17 in place of
13. Its roots were found to be –2, and –15. Find the roots of the original equation.
2
6. If the ratio of the roots of the quadratic equation x + px + q = 0 be equal to the ratio of the
2 2 2
roots of x + lx + m = 0, then prove that p m = l q.
7. If , are the roots of the equation 3x2 + 2x + 1 = 0, then find the equation whose roots
1 1
are , .
1 1
8. If the quadratic equation ax2 + bx + c = 0 has real roots of opposite sign in the interval
c b
(–2, 2), then prove that 1 + 0.
4a 2a
9. Show that no real values of x and y other than 4 and 4 respectively will satisfy the
equation x2 – xy + y2 = 4(x + y – 4).
10. If a . 3tanx + a . 3tanx 2 = 0 has real solutions, x , 0 x , then find the set of all
2
possible values of parameter ‘a’.
11. The equations ax2 + bx + a = 0, x3– 2x2+ 2x– 1 = 0 have two roots in common. Then find
the value of a + b.
2 2
12. Find the set of values of p, for which x px + p 4 < 0 for at least one x < 0.
P-2022-CBSE-P1-MATHEMATICS- QEE
44
Level – II
2. For what real values of a do the roots of the equation x2 – 2x – (a2 – 1) = 0 lie between
the roots of the equation x2 – 2(a +1)x + a (a – 1) = 0 ?
3. If a1, a2, a3, …….an (n 2) are real and (n – 1)a12 – 2na2 < 0 then prove that at least two
roots of the equation xn + a1xn-1 + a2xn-2+ ... + an = 0 are imaginary.
4. Find all real value of a for which the equation x4 + (a – 1)x3 + x2 + (a – 1)x + 1 = 0
possesses at least two distinct positive roots
5. Find out minimum non-negative real values of a, b and c given that the equation
x4 + ax3 + bx2 + cx + 1 = 0 has real roots.
x2 ax c
6. If x is real and a and b are unequal, then prove that the expression can take
x 2 bx c
any value when c 0.
7. Show that if p, q, r, s, are real numbers and pr = 2(q + s) then at least one of the
equations x2 + px + q = 0, x2 + rx + s = 0 has real roots.
8. The equation ax2 + bx + c = 0 has real and positive roots. Prove that the roots of the
2 2
equation a x + a(3b – 2c)x + (2b – c)(b – c) + ac = 0 are real and positive.
10. If one root of the equation (l–m) x2 + lx + 1 = 0 is double of the other and l is real, find the
greatest value of m.
P-2022-CBSE-P1-MATHEMATICS-QEE
45
Objective:
Level – I
1. Let f(x) = x2 + bx + c, where b, c R. If f(x) is a factor of both x 4 + 6x2 + 25 and 3x4 + 4x2
+ 28x + 5, then the value of b is
(A) 2 (B) 1
(C) –1 (D) –2
2. Let a, b, c be the sides of a triangle. No two of them are equal and R. If the roots of
the equation x2 + 2(a + b+ c) x + 3 (ab + bc + ca) = 0 are real, then
4 5
(A) (B)
3 3
1 5 4 5
(C) , (D) ,
3 3 3 3
6. If the smaller root of the equation x2 + ax 4 = 0 lies between 1 and 2, then a belongs
to
(A) (0, ) (B) (0, 3)
(C) (, 3) (D) (, 3) (0, )
8. The set of values of a for which the inequality x2 + ax + a2 + 6a < 0 is satisfied for all
x (1, 2) lies in the interval
(A) (1, 2) (B) [1, 2]
(C) [–7, 4] (D) None of these
9. Let S denote the set of all real values of a for which the roots of the quadratic equation
x2 2ax + a2 1 = 0 lies between 5 and 10, then S will be
(A) (1, 2) (B) (2, 9)
(C) (4, 9) (D) (6, 9)
P-2022-CBSE-P1-MATHEMATICS- QEE
46
x2 14x 9
10. If x is real then the greatest value of is
x 2 2x 3
(A) 2 (B) 3
(C) 4 (D) 5
14. If (m2 –3)x2 + 3mx + 3m + 1 = 0 has roots which are reciprocals of each other, then the
value of m equals
(A) 4 (B) –2
(C) 2 (D) None of these
15. If ax2 + bx + 6 = 0 does not have two distinct real roots, then the least value of 3a + b is
(A) 2 (B) –2
(C) 1 (D) –1
16. Let two numbers have A.M.= 9 and G.M. 4. Then these numbers are the roots of the
quadratic equation
(A) x2 + 18x + 16 = 0 (B) x2 18x + 16 = 0
2
(C) x + 18x 16 = 0 (D) x2 18x 16 = 0
18. The number of real roots of the equation (x 1)2 + (x 2)2 + (x 3)2 = 0 is
(A) 2 (B) 1
(C) 0 (D) 3
19. If , are the roots of the equation ax2 + bx + c = 0 then the value of 3 + 3 is
3abc b3 a3 b3
(A) (B)
a3 3ab
3 3
3abc b b 3abc
(C) 3
(D)
a a3
20. The roots and of the quadratic equation ax2 + bx + c = 0 are real and of opposite sign.
Then the roots of the equation (x – )2 + (x – )2 = 0 are
(A) positive (B) negative
(C) real and of opposite sign (D) imaginary
P-2022-CBSE-P1-MATHEMATICS-QEE
47
2 4 2
21. If the equation (a 12)x 8x 4 = 0 has no real solution, then
(A) 4 a 4 (B) 2 3 < a < 2 3
(C) a 4 (D) none of these
Level – II
2. The value of ‘p’ for which the sum of the square of the roots of 2x2 – 2(p – 2)x – p –1= 0
is least, is
(A) 1 (B) 3/2
(C) 2 (D) – 1
3. If the equation (a – 5)x2 + 2(a – 10)x + a + 10 = 0 has roots of opposite signs, then
(A) a > 10 (B) 15 < a < 5
(C) 10 < a < 5 (D) none of these
4. If sin, cos are the roots of equation cx2 + bx + a =0, then a, b, c are connected by the
relation
(A) b2 + 2ac – c2 = 0 (B) c2 + 2ac + b2 = 0
2 2
(C) b – 2ac – c = 0 (D) 2ac – b2 – c2 = 0
5. The set of values of ‘a’ for which 1 lies between the roots of x2 – ax – a + 3 = 0 is
(A) (–, –6) (B) (–, +6)
(C) (–, –6) (2, ) (D) (2, )
6. > 0 and < 0 are the roots of the equation x2 px + q = 0, whereas > 0 and < 0
are the roots of the equation x2 px + q + k = 0 where k > 0. Then
(A) > , > (B) < , <
(C) > , < (D) < , >
7. Let a, b, c be three distinct positive real numbers. Then the number of real roots of
ax2 + 2b|x| + c = 0 is
(A) 0 (B) 1
(C) 2 (D) 4
8. Let S denote the set of real values of ‘a’ for which the roots of the equation
x2 – ax – a2 = 0 exceed ‘a’. Then S belongs to
(A) (– , 0) (B) (–2, – 1/2)
(C) (–1/2, 1/4) (D) null set
9. If a and b are distinct real numbers and difference between the roots of x2 + ax + b = 0
and that of x2 + bx + a = 0 is same then the value of a + b is
(A) 2 (B) – 2
(C) – 4 (D) 0
P-2022-CBSE-P1-MATHEMATICS- QEE
48
2
11. One root of the equation 5x + 13x+ k = 0 is the reciprocal of the other, if
(A) k = 0 (B) k = 5
(C) k = 1/6 (D) k = 6
2
12. If the sum of the roots of the equation ax + bx + c = 0 is equal to the sum of the squares
2 2 2
of their reciprocals, then bc , ca , ab are in
(A) A.P. (B) G.P.
(C) H.P. (D) none of these
2 2
13. If the equation formed by decreasing each root of ax + bx + c = 0 by 1 is 2x + 8x + 2 = 0,
then
(A) a = b (B) b = c
(C) c = a (D) b + a + c
2
14. The equation whose roots are opposite in sign to those of the equation x 3x 4 = 0 is
given by
(A) 4x2 3x + 1 = 0 (B) x2 + 3x 4 = 0
2
(C) x + 3x + 4 = 0 (D) none of these
x 2 ax 3
17. If takes all real values for possible real values of x, then
x2 x a
(A) 4a3 + 39 0 (B) 4a3 + 39 < 0
(C) a < 1/4 (D) a ¼
18. If the quadratic equation ax2 + bx + c = 0 (a > 0) has two roots and such that < 2
and > 2, then
2
(A) b 4ac > 0 (B) c < 0
(C) a + |b| + c < 0 (D) 4a + 2|b| + c < 0
19. Let a, b and c be three positive real numbers and are in G.P. and 1 be a cube root of
unity, then the equation ax2 + bx + c = 0 has
(A) real roots (B) imaginary roots
(C) imaginary roots with negative real part (D) roots are , 2 where = b/a
20. The values of a for which x4 – 2ax2 + a2 – a = 0 has all real roots are
(A) – 1 (B) 1
(C) 2 (D) 3
21. If ordered pair (, ) where , I satisfy the equation 2x2 – 3xy – 2y2 = 7, then value of
+ can be
(A) 5 (B) 4
(C) – 4 (D) 3
P-2022-CBSE-P1-MATHEMATICS-QEE
49
1/3 1/3 1/3
22. The solution of x + (2x – 3) = [3(x – 1)] is
(A) 0 (B) 3/2
(C) 1 (D) none of these
23. If f(x) = 0 is a polynomial whose coefficients all 1 and whose roots are all real, then the
degree of f(x) can be equal to
(A) 1 (B) 2
(C) 3 (D) 4
Reason Assertion
25. STATEMENT I : If a>b>c and a3 + b3 +c3 = 3abc then the equation ax2 + bx + c = 0 has
one positive and one negative real root.
STATEMENT II : If roots are of opposite nature, then product of roots< 0 and
sumof roots 0
COMPREHENSIONS
(I)
Read the following write up carefully and answer the following questions:
2 2
26. Let 1, 2 be the roots of x – x + p = 0 and 3 , 4 be the roots of x – 4x + q = 0. If
1, 2 , 3 , 4 are in G.P then the integral values of p & q respectively are
(A) 2, 32 (B) 2, 3
(C) 6, 3 (D) 6, 32
27. If the roots of the equation a(b – c) x2 + b(c – a) x + c(a – b) = 0 be equal, then a, b, c, are
in
(A) A.P. (B) G.P.
(C) H.P. (D) None of these
P-2022-CBSE-P1-MATHEMATICS- QEE
50
(II)
Read the following write up carefully and answer the following questions:
29. The equation 2sin2x2 – 3sinx + 1 = 0, , has one root lying in the interval :
4 2
(A) (0, 1) (B) (1, 2)
(C) (2, 3) (D) ( 1,0)
30. If f(x) = ax2 + bx + c, such that c < 0 and a – 2b + 4c > 0, then f(x) has
1 1
(A) One root in the interval 0, (B) One root in the interval ,0
2 2
(C) both root are positive (D) None of these
(III)
Read the following write up carefully and answer the following questions:
and are the roots of the equation ax2 + bx + c = 0 and 4, 4 are the roots of the equation
lx2 + mx + n = 0 (, are real and distinct). Let f (x) = a2lx2 4aclx + 2c2l + a2m = 0, then
(IV)
Read the following write up carefully and answer the following questions:
n n1
Let a0x + a1x + + an1x + an = 0 be the nth degree equation with a0, a1, an integers. If p/q is
a rational root of this equation, then p is a divisor of an and q is a divisor of a0. If a0 = 1, then every
rational root of this equation must be an integer.
P-2022-CBSE-P1-MATHEMATICS-QEE
51
3 2
35. At least one integral root of the equation x 13x + 15x + 189 = 0 exceeds another root
by
(A) 1 (B) 2
(C) 3 (D) none of these
Matrix Match
37. If , are the roots of the equation x2 px + r = 0 and , 2 are the roots of the
2
equation x2 qx + r = 0, then
Column I Column II
(A) , 2 are the roots of the equation (p) 36x2 6(p + q)x + (2p q)(2q p) = 0
(B) /2, are the roots of the equation (q) 9x2 6(p + q)x + 4(2p q) (2q p) = 0
3 (r) 9x2 3(p + q)x + (2p q) (2q p) = 0
(C) , 3 are the roots of the
2
equation
2
(s) x (p + q)x + (2p q) (2q p) = 0
(D) , are the roots of the equation
4 2
P-2022-CBSE-P1-MATHEMATICS- QEE
52
39. Matching List Type(3 Column & 4 Rows)
Column – 1 Column – 2 Column – 3
(I) y = |x|2 – 2|x| – 3 (i) Df = (–,–1] [3, ) (P) Ranges = [–4, )
2
(II) y = |x – 2x – 3| (ii) Df = R (Q) Range = R
2
(III) y = x – 2x – 3 (iii) Df = [–4, ) (R) Range = [0, )
2
(IV) y = ||x| – 2|x| – 3| (iv) Df = [0, ) (S) Range = (–, –1] [3,
)
(V) |y| = x2 – 2x – 3 (v) Df = (–, –1) (3, ) (T) Range = (–4, )
(I). Which of the following is correct combination
(A) (V) (i) (Q) (B) (V) (v) (T)
(C) (V) (ii) (Q) (D) (V) (ii) (R)
(II). Which of the following is correct combination
(A) (IV) (ii) (P) (B) (IV) (ii) (R)
(C) (IV) (ii) (Q) (D) (IV) (ii) (T)
(III). Which of the following is not correct combination
(A) (I) (ii) (P) (B) (II) (ii) (R)
(C) (III) (ii) (P) (D) (I) (iii) (R)
Numerical Based
k
40. Find the possible value of (k – 5) for which the equation (x m 1) (x + m) = 10k has
m 1
solution and + 1 for some .
x3
41. Number of points with integral coordinates on the curve y = is __________
x2 x
Decimal Type
42. Find the value of k such that the equation |x2 – 5 |x| + 6| = k is satisfied by six distinct
values of x.
P-2022-CBSE-P1-MATHEMATICS-QEE
53
ANSWERS TO CHAPTER PRACTICE PROBLEMS
Subjective:
Level – I
1. acx2 bx + 1 = 0
2. ( 4, 7)
3. a 2,8
Level – II
4. a+b=c
Objective:
1. A 2. D 3. A 4. A
5. C 6. C 7. D 8 D
9 A
1. A, B 2. A, D 3. A, C, D 4. A, D
Numerical Based
1. 2 2. 4 3. 2 4. 6
5. 4 6. 1 7. 4 8. 4
9. 1
P-2022-CBSE-P1-MATHEMATICS- QEE
54
ANSWERS TO ASSIGNMENT PROBLEMS
Subjective:
Level – 0
5 5 q p pq 2
1. (i) (ii) , (iii) ,
10 p q pq pq
4 9
(iv) p + q – r, p – q + r (v) –1, 2 (vi) ,
13 13
3i 7
(vii) –1, 8 (viii) 2 2, 1 i (ix) 1, 2,
2
(x) a,
a 1 17
2
1 1
2. (a) (i) , 1 (ii) (c) m = 2, n = 1
2 3
3. 16 4. x = 5, 6
2
5. (i) c = a (ii) mb2 ac 1 m (iii) ac a c b3 2abc
1 1
(iv) an c n 1
ac n n1 b 0 (v) mnb2 ac(m n)2
7. a = 0, 24 8. x1 8, x2 6
9. 9
10. cr ap 2 (aq br)(bp cq) . 13. a = 4, b = 12
Level – I
P-2022-CBSE-P1-MATHEMATICS-QEE
55
2
4. –3, –10 7. x – 2x + 3 = 0
4
11. 0 12. <p<2
3
Level – II
1
2. , 1 4. a (–, –1/2)
4
5. 4, 6, 4
Objective:
Level – I
1. D 2. A 3. B 4. C
5. B 6. C 7. B 8. D
9. D 10. C 11. C 12. B
13. A 14. A 15. B 16. B
17. C 18. C 19. C 20. C
21. B
Level – II
1. C 2. B 3. C 4. C
5. D 6. C 7. A 8. D
9. C 10. D 11. B 12. A
13. B 14. B 15. A 16. A, B
17. B, C 18. A, B, C, D 19. B, C, D 20. B, C, D
21. B, C 22. A, B, C 23. A, B, C 24. C
25. A 26. A 27. C 28. B
29. B 30. B 31. B 32. B
33. C 34. A 35. B 36. B
37. (A) (q); (B) (r); (C) (s); (D) (p)
38. (A) (p, q, r); (B) (q, r, s); (C) (r); (D) (t)
39. (I). A (II). B (III). D
40. 6 41. 2 42. 0.25 43. 7.00
P-2022-CBSE-P1-MATHEMATICS- QEE
PINNACLE-CBSE
Set 1
CBSE: Sets and their representations finite and
Operation on sets 3
infinite sets, Equal sets, Subsets of the set of real
Applications 6 numbers especially intervals with notations,
Power set, Universal set, Venn diagrams,
Exercise 1 8 Complement of a set. Operations on sets (union,
intersection and difference of two sets).
Relation 8 Applications of sets. Ordered pairs, Cartesian
product of sets. Number of elements in the
Function 9
Cartesian product of two finite sets, Relation,
Exercise 2 9 pictorial diagrams, domain, co-domain and range
of a relation. Real valued function of the real
Definitions of function, domain variable, domain and range of these functions,
constant, identity, polynomial, rational, modulus,
and range 9
signum and greatest integer function and their
Exercise 3 11 graphs. Sum, difference, product and quotient of
functions.
Graphs of some elementary
Functions 12
Algebra of Functions 13
Answers to Exercises 14
Solved Problems 15
Assignment Problems 25
Answers to Chapter 36
Practice Problems
Answers to Assignment 37
Problems
[[[ [
Set:
A set is a collection of well defined objects i.e. the objects follow a given rule or rules. If we say
that we have a collection of short students in a class, then this collection is not a set as “short
students” is not well defined. If, however, we say that we have a collection of students whose
height is less then 5 feet, then it represents a set.
Elements of a set:
The members of a set are called its elements. A set is usually denoted by capital letters A, B, C
etc, where as the elements of a set are generally denoted by lower case letters a, b, c, d etc. If an
element x is in set A, we say that x belongs to A and write x A. If the element x is not in A then
we write x A.
Examples of sets:
1. The set of vowels in the alphabet of English language.
2. The set of all points on a particular line.
3. The set of all lines in a particular plane.
4. The set of all odd natural numbers.
5. The set of all real numbers.
The elements in a set can be written in any order e.g.
A = {1, 3, 5, 7, 9, …}, B = {11, 9, 5, 7}, C = {…, 2, 1, 0, 1, 2, …}, D = {Amar, Aman, Ajay}.
This is called the roster method of representing a set. A set can also be represented by stating
the properties within braces, which are satisfied by the elements of the set e.g.
A = {x: x = 2n + 1, n 1, n N}, A = {x: 6 x 12, x N}.
This method of representing a set is called the set builder method.
Solution: (i) The general term, here, is 2n + 1. For n = 0, 1, 2, 3, 4, we get the elements of
the set. Hence, we write the given set as
{x: x = 2n + 1, n N + {0} 0 n 4}.
1
(ii) The general term is . For n 1, 2, 3, 4 we get the elements of the
2n 12
1
set. Hence, we write the given set as x : x , n N, n 4 .
2n 12
P-2022-CBSE-P1-MATHEMATICS-SRF
2
Some special sets:
(i) Finite and infinite sets:
A set A is finite if it contains only a finite number of elements; we can find the exact number of
elements in the set. Otherwise, the set is said to be an infinite set; e.g. The set 1, 3, 5, 7, 9 is a
finite set, having five elements. The set of letters in the word MISSISSIPI is a finite set containing
four letters M, I, S, P , although there are, in all, eleven letters in the word; only distinct elements
are considered in a set. Other examples are:
(1) The set of all odd natural numbers is an infinite set.
(2) The set of all points on a particular straight line is an infinite set.
(3) The following sets are all infinite sets:
N = set of all natural numbers = 1, 2, 3, ...
Z = set of all integers = ..., 2, 1, 0, 1, 2,...
p
Q = set of all rational numbers = :p, q Z,q 0
q
R = set of all real numbers = {x : x is a rational and an irrational number}
C = set of all complex numbers = x iy ; x, y R
(vi) Subsets:
If each element of a set A is also an element of a set B, then A is called a subset of B, and we
write A B , e.g. A a, k, l, o, u , B a, b, c,......x, y, z . Here A B . And if A 2, 4, 6,
B 1, 2, 3, 4, 5, 6, C 4, 6 , then A B, C A, C B . Note that N Z, N Q, R C .
P-2022-CBSE-P1-MATHEMATICS-SRF
3
(vii) Proper subsets:
A set A is called a proper subset of B if and only if each element of A is an element of B and there
is at least one element of B which is not in A i.e. A B and A B and we write A B e.g. if
A a, b, c, d , B a, b, c, ... , x, y, z , then A B , and if
A 2, 4, 6, , B 1, 2, 3, 4, 5, 6 , C 4, 6 , then A B , C A, C B .
Note: The null set is a subset of every set and every set is a subset of itself.
Theorem: If a finite set has n elements, then the power set of A has 2n elements.
Operations on sets:
The operations on sets, by which sets can be combined to produce
new sets, can be best illustrated through Venn diagram. A set S is
called a universal set if every set we consider, is a subset of S. In A
B
Venn diagram, the universal set is shown in the form of a rectangle
and the subsets of the universal set are generally shown as circles and AB
ellipses. If A is a proper subset of B i.e. A B , we say that A is
contained in B.
(i) Union of sets:
The union of two set A and B is defined as the set of all elements AB
P-2022-CBSE-P1-MATHEMATICS- SRF
4
Note : A A A (i.e. union of sets is idempotent), A S S and A A. Also
A B B A and A B C A B C .
Illustration 3. Find the union of the sets A 5, 7, 9 , B 9, 13, 15 and
C 13, 15, 17 . Also show that A B C A B C .
Solution: We have A B 5, 7, 9 9, 13, 15 5, 7, 9, 13, 15 and
B C 9, 13, 15 13,15,17 9, 13, 15, 17
A B C 5, 7, 9, 13, 15 13, 15,17 5, 7, 9, 13, 15, 17 and
A B C 5, 7, 9 9, 13, 15, 17 5, 7, 9, 13, 15, 17
A B C .
Note: A A A, A S A and A .
The commutative, associative and distributive laws hold for intersection of two sets i.e.
A B B A
A B C A B C
A B C A B A C
A B C A B A C
The intersection of n sets A1, A 2 ........An is written as
n
A i A1 A 2 A 3 ......... A n x : x A i for all i, 1 i n .
i1
P-2022-CBSE-P1-MATHEMATICS-SRF
5
iii) Difference of sets: S A-B
c
Note: A
c
A, Sc , A A c , A A = S.
c
Illustration 5. For the sets S 10, 11, 12, 13, ...., 17, 18 , A 11, 13, 15 , B 12, 14, 16 ,
show that B c Ac A B A B c .
Note: The properties of the complement of sets are known as DeMorgan laws, which are
(i) A c Bc B A
c
(ii) A B A c Bc
c
(iii) A B A c Bc
c
To prove (iii), we have A B x : x A B x : x A or x B
x : x A c or x Bc A c Bc .
Note: A B C A B A C ; A B C A B A C .
A
A
A A – (B C) A
A-C
A-B
(B C)
B
B
B
B
C C
C C
P-2022-CBSE-P1-MATHEMATICS- SRF
6
A
(A – B) (A – C)
B
C
(v) Applications:
Let A be a finite set. The number of elements in A is denoted by n A . Let A and B be two finite
sets. If A and B are two disjoint sets, then n A B n A n B .
If A and B are not disjoint, then
(i) n A B n A n B n A B
(ii) n A B n A B n B A n A B
(iii) n A n A B n A B
(iv) n B n B A n A B
S A-B A B
B-A
S
A
B
A B
Illustration 6. Set A has 3 elements and set B has 6 elements. Find the maximum and
minimum number of elements in A B .
Illustration 7. In a certain examination, the candidates can offer papers in English or Hindi or
both the subjects. The number of candidates who appeared in the examination is
1000 of whom 650 appeared in English and 200 both in English and Hindi. Find
the number of candidates who offered paper in
(i) Hindi, (ii) English only, (iii) Hindi only.
P-2022-CBSE-P1-MATHEMATICS-SRF
7
1000 650 n B 200 n B 550 .
Also the number of candidates who offered paper in English only
n A B n candidates A, and candidates B
n A B n A n A B 650 200 450 .
Similarly, the set of candidates who offered paper in Hindi only is B – A
n B A n B n A B 550 200 350 .
Note: (i) A B C A B A C
(ii) A B C A B A C
(iii) A B C A B A C
(iv) A B C A C B C
(v) A B C A C B C
(vi) A B C A C B C
P-2022-CBSE-P1-MATHEMATICS- SRF
8
A B A C 2, 3 , 2, 4 , 2, 7 , 2, 8 , 5, 3 , 5, 4 , 5, 7 , 5, 8
A B C .
Exercise 1.
i) Prove by using venn diagram that ( A B ) A B .
ii) If A a, e, i , o, u , B b, c, d , then find A B B .
iii) If S a, b, c, d , e, . f , g , A a, b, c, d , B c, d , e, f , then verify that
c
A B Ac B c .
iv) If A = {1, 2, 3, 4} and B = {2, 3, 5}, prove that A B A .
v) If A 2 , B 5 , C 3, 4, 6 , then verify that A B C A B A C .
vi) In a conference attended by 550 delegates tea and coffee are served. The number
of delegates who drink tea is 400 and the number of delegates who drink tea only
is 200. Find (a) how many of them drink coffee,
(b) how many drink coffee only.
Relation:
Let A and B be two sets. A relation R from the set A to set B is a subset of the cartesian product
A B . Further, if x, y R , then we say that x is R-related to y and write this relation as x R y.
Hence R x, y ; x A, y B, x R y .
As an example, consider A 1, 2, 3 and B 1, 8, 27 , so that
A B 1, 1 , 1, 8 , 1, 27 , 2, 1 , 2, 8 , 2, 27 , 3, 1 , 3, 8 , 3, 27 .
Consider now a subset R of A B , as R 1, 1 , 2, 8 , 3, 27 .
We notice that in every ordered pair of R, the second element is the cube of the first element i.e.
the element of the ordered pairs of R have a common relation- ship which is “cube”.
In case we take A 2, 4, 6 , B 1, 5 ,
then A B 2, 1 , 2, 5 , 4, 1 , 4, 5 , 6, 1 , 6, 5 .
Here the first element in each of the ordered pair is greater than the second element. Hence the
relationship is “greater than”. Obviously, from the definition, x R y and y R x are not the same,
since R x, y : x A, y B, x R y and R x, y : x B, y A, x R y are different.
Domain and Range of a relation: Let R be a relation defined from a set A to a set B,
i.e. R A B . Then the set of all first elements of the ordered pairs in R is called the domain of
R. The set of all second elements of the ordered pairs in R is called the range of R. That is,
D = domain of R x : x, y R or x : x A and x, y R ,
R = range of R y : x, y R or y : y B and x, y R .
Clearly D A and R* B .
For example, for R given in 1, 1 , 2, 8 , 3, 27 above, domain of R 1, 2, 3 ,
P-2022-CBSE-P1-MATHEMATICS-SRF
9
Function:
Let A and B be two non-empty sets. Let to each element of A, there correspond exactly one
element of B. This correspondence between the elements of A and B is called a function from A
to B. Function is a special case of a relation, since a relation may relate an element of A to more
than one elements in B. A function from A to B is usually denoted by the symbols f, g etc. and we
write f : A B . We also say that “f is a mapping from A to B”.
The set A is called the domain of the function f and
B is called the co domain of the function f. f
a
b
A B
Let an element a A correspond to b B under the function f. Then we say that b is the image
of a under f and a is the pre-image of b. We then write f a b . By definition, if f is a function
from A to B, then each element of A has unique image in B. However, every element of B
needs not be an image of some a in A. The subset of B (co domain) which contains all the
images of the elements of A is called the range of the function and is denoted by f A
i.e. f A f a : a A .
Exercise 2.
i) Let R 1, 3 , 2, 5 , 3, 7 , 4, 9 , 5, 11 be a relation in the set
A 1, 2, 3, ...... . Find the domain and range of R.
P-2022-CBSE-P1-MATHEMATICS- SRF
10
Set of images of different elements of the set X is called the range of ‘f’. It is obvious
that range could be a subset of the co-domain as we may have some elements in the
co-domain which are not the images of any element of X (of course, these elements of
the co-domain will not be included in the range). Range is also called domain
of variation.
If R is the set of real numbers and X and Y are subsets of R, then the function
f (x) is called a realvalued function or a real function.
Domain of a function ‘f’ is normally represented as Domain (f). Range is represented as
Range (f). Note that some times domain of the function is not explicitly defined. In these cases
domain would mean the set of values of ‘x’ for which f (x) assumes real values that is if y = f (x)
then Domain (f) = {x: f (x) is a real number}.
e.g. Let X = {a, b, c}, Y = {x, y, z}. Suppose f(a) = y, f(a) = x, f(b) = y, f(c) = z. Then f is not a
function of X into Y since a X has more than one f-images in Y.
On the other hand, if we set f(a) = x, f(b) = x and f(c) = x, then f:X Y is a function since each
element in X has exactly one f-image in Y.
Consider the following examples:
(i) Let X = R, Y = R and y = f(x) = x2.
Then f : X Y is a function since each element in X has exactly one f - image in Y. The
range of f = {f(x) : x X} = {x2 : x R} = [0, ).
(ii) Let X = R+, Y = R+ and y = x. Then f : X Y is a function. The range of f is R+
(iii) Let X = R, Y= R and y2 = x. Here f(x) = x i.e. f is not a function of X into Y since each
x > 0 has two f-images in Y, and further, each x<0 has no f-image in Y.
We are primarily interested in functions whose domains and ranges are (sub) sets of real
numbers.
1
e.g. Let the function f be defined by f(x) = .
2x 6
In this case we must have 2x + 6 > 0 x > 3. Therefore, the domain of f is (–3, ), the range of
1
f = (0, ). Thus we have the function f : (–3, ) (0, ) defined by f(x) = .
2x 6
x
Let the function f be defined by f(x) = . The function makes sense for all values of x
x 1 x 2
except x = 1 and x = 2. Therefore, the domain of f is R – {1, 2}.
Illustration 10. Find the domain and the range of the function y = f (x), where f (x) is given by
2
(i) x 2x 3, (ii) x 2 2x 3 , (iii) sin x,
(iv) tan x, (v) log10 (x).
Solution: (i) Here y = (x 3) (x + 1). The function is defined for all real values of x
2
its domain is R. Also x 2x 3 y = 0 for real x
P-2022-CBSE-P1-MATHEMATICS-SRF
11
4 + 4 (3 + y) 0 4 y < .
Hence the range of the given function is [ 4, ).
(ii) Here y = (x 3)(x 1) (x 3) (x + 1) 0 so that x 3 or x 1.
Hence the domain is R ( 1, 3) or ( , 1] [3, ).
Since f(x) is nonnegative in the domain, the range of f(x) is the interval [0, ).
(iii) The function f(x) = sin x, (x in radians) is defined for all real values of x
domain of f(x) is R. Also 1 sin x 1, for all x,
so that the range of f (x) is [ 1, 1].
sin x
(iv) The function f(x) = tan x = is not defined when
cos x
cos x = 0, or x = (2n + 1) , n = 0, 1, 2, …
2
Hence domain of tan x is R (2n 1) , n 0, 1, 2,.... , and its range is R.
2
(v) The function f (x) = log10 x is defined for all x > 0. Hence its domain is (0, ) and
range is R.
Illustration 11. Find the domain of definition of the following functions:
x 1 x 2
(i) f(x) = log 1 2x 3 , (ii) f(x) =
2
x 3 x 4
Solution: (i) For f(x) to be defined log1/2(2x – 3) 0
2x – 3 1 x 2. . . . (1)
3
Also 2x – 3 > 0 x > . . . . (2)
2
Combining (1) and (2) we get the required values of x. Hence the domain of
3
definition of f(x) is the set , 2
2
x 1 x 2
(ii) For f(x) to be defined 0 and x 3, 4.
x 3 x 4
By wavy - curve method the domain of definition of f(x) is the set
x (– , – 2] [1, 3) (4, ).
Exercise 3.
i) Find the domains of the functions:
x 1 1
(a) , (b) , (c) x 1 3 x ,
x 2 2x 3
x2 1
(d) , (e)
1 x2 | x | x
ii) Find the domain of the function f(x) = loge(x – 2).
1
(iii) Prove exist when x < 0.
x x
(iv) Prove that the domain of f(x) = ( 9 x 2 ) ( x 2 4 ) is [3, 2] [2, 3].
P-2022-CBSE-P1-MATHEMATICS- SRF
12
Graphs of Some Elementary Functions
A convenient and useful method for studying a function is to study it through its graph. To draw
the graph of a function f : X Y, we choose a system of coordinate axes in the plane such that
to each x X, there corresponds the ordered pair (x, f(x)) which determines a point in the plane.
The set of all points {(x, f(x)) : x X} is the graph of f. We consider some examples of functions
and their graphs.
P-2022-CBSE-P1-MATHEMATICS-SRF
13
consists of infinitely many Range: [0, 1) y
Algebra of Functions:
Given functions f : D R and g : D R, we define functions f + g, f - g, gf and f/g as follows:
f + g : D R is a function defined by (f + g)(x) = f(x) + g(x) for all x D
f – g : D R is a function defined by (f – g) (x) = f(x) – g(x) for all x D
fg : D R is a function defined by (f g) (x) = f(x) g(x) for all x D
f x
f/g:CR is a function defined by (f/g) (x) = , g x 0 ,where C={xD:g(x) 0}. i.e.
g x
x D {x : g (x) = 0}.
If k is any scalar, then kf is a function from DR defined by (kg)(x) = kf(x), xD.
3 3
Solution: (f + g) (x) = f(x) + g(x) = 1 x2 + x + 1, (f g) (x)= f(x) g(x) = 1 x2 x 1,
f f x 1 x2
(fg) (x) = f(x) g(x) = (x3 + 1) 1 x2 and x 3 , x -1 .
g gx x 1
The domain of each of f + g. f - g and fg is [-1, 1] and that of f/g is
{x [1, 1] : g(x) 0} = (1, 1].
Note: The sum f + g, the difference f g, the product fg and the quotient f/g are defined only
when f and g are real functions having the same domain. In case f and g, have different domains,
these operations are defined for those values of x which are common to the domains of both f
and g i.e. x to the intersection of the domains of f and g.
P-2022-CBSE-P1-MATHEMATICS- SRF
14
ANSWERS TO EXERCISES
Exercise 1.
(ii) (vi) (a) 350 (b) 150
Exercise 2.
(i) Domain = {1, 2, 3, 4, 5}, range = {3, 5, 7, 9, 11}
Exercise 3.
3
(i) (a) R {2} (b) ,
2
(c) [ 1, 3] (d) R
(e) (, 0)
(ii) x (2, )
P-2022-CBSE-P1-MATHEMATICS-SRF
15
SOLVED PROBLEMS
Subjective:
Level – 0
Problem 1. If A is the null set, find the number of elements in the power set P P( A) .
Problem 2. Find the subsets that can be formed from the set A 4, 5, 6 and the number
of subsets.
Solution: The subsets that can be formed from the given set
are , 4 ,5 , 6 , 4, 5 , 4, 6 , 5, 6 , 4, 5, 6 . These are 8 subsets.
Problem 3. Of the 20 teachers of mathematic and physics, 12 teach mathematic and 4 teach
both the subjects. Find the number of teachers teaching physic.
Problem 4. X, Y are two sets and X has 40 elements, X Y has 60 elements and X Y
has 10 elements. Find the number of elements in Y is
Also n X Y 40 10 30 .
n Y 60 30 30
Level – I
x2 x 1
Problem 5. Find the domain of the following function f(x) =
x2 4x 3
x2 x 1
Solution: Here f(x) = and f(x) is defined for all x R other than numbers,
x2 4x 3
where
x2 + 4x + 3 = 0 x = -3, -1.
Hence the domain of f(x) = R - {-3, -1}.
Solution: f(x) = 2cos2x + 3 sin2x + 1 = 2sin 2x + 2 or 2cos 2x + 2.
6 3
So range is from [0, 4].
P-2022-CBSE-P1-MATHEMATICS- SRF
16
Problem 7. Prove by venn diagram that A B A B is equal to A B B A .
A
B
AB
and A B is
A
B
so that A B A B is
A
B
which is same as A B B A .
Solution: n(M P C) = 50
n(M) = 37, n(P) = 24, n(C) = 43
n(M P) 19, n(M C) 29, n(P C) 20
n(M P C) = n(P) + n(C) + n(M) – n(P C) – n(C M) – n(M P) + n(P C
M)
50 37 + 24 + 43 – 19 – 29 – 20 + n(P C M).
n(P C M) 14.
x2 2x 1
Problem 10. Find the range of the function g(x) = .
x
x2 2x 1
Solution: Let =y
x
2
x + (2 − y) x + 1 = 0
Since x R
(2 − y)2 − 4 0
y2 − 4y 0
i.e. y R − (0, 4) = range
P-2022-CBSE-P1-MATHEMATICS-SRF
17
Objective:
Level – 0
True / False
Problem 1. In a town of 10,000 families it was found that 40% families buy newspaper A,
20% families buy newspaper B and 10% families buy newspaper C, 5% families
buy A and B, 3% buy B and C, 4% buy A and C. If 2% families buy all the three
newspapers, then the number of families which buy A only is 3300.
Solution: True
We have, N = 10,000, n(A) = 40% of 10,000 = 4000,
n(B) = 2000, n(C) = 1000, n(A B) = 500, n(B C) = 300,
n(C A) = 400, n(A B C) = 200.
n ( A B C ) = n(A (B C)) = n(A) n(A (B C))
= n(A) n((A B) (A C))
= n(A) {n(A B) + n(A C) n(A B C)}
= 4000 (500 + 400 200) = 3300.
Problem 2. A has 3 elements and B has 6 elements such that A B. The number of
elements in A B is 6.
Solution: False
A B A B = A n(A B) = n(A) = 3.
Problem 4. Let A and B have 3 and 6 elements respectively. The minimum number of
elements in A B is ______
Solution: 6
A B will contain minimum number of elements if A B and in this case
n(A B) = n(B) = 6.
Solution: 8
n(M P) = 20, n(M) = 12, n(M P) = 4
n(M P) = n(M) + n(P) – n(M P)
n(P) = 12
n(P) only = n(P) – n(M P) = 8.
P-2022-CBSE-P1-MATHEMATICS- SRF
18
Level – I
Problem 8. Let F1 be the set of all parallelograms, F2 the set of rectangles, F3 the set of
rhombuses, F4 the set of squares and F5 the set of trapeziums in a plane, then F1
is equal to
(A) F2 F3 (B) F2 F3 F4 F1
(C) F3 F4 (D) none of these
Problem 9. A and B are two sets having 3 and 4 elements respectively and having 2
elements in common. The number of relations which can be defined from A to B
is
(A) 25 (B) 210 – 1
12
(C) 2 – 1 (D) none of these
Solution: n(A B) = 12
Hence the number of subsets of A B is 212.
Problem 11. A survey shows that 63% of the Americans like cheese where as 76% like
apples. If x% of the Americans like both cheese and apples then
(A) x = 39 (B) x = 63
(C) 39 x 63 (D) none of these
P-2022-CBSE-P1-MATHEMATICS-SRF
19
n(A C) 39 …(1)
But A B A and A B B
n(A B) n(A) and A B B
n(A B) 63 and n(A B) n(B)
n(A B) 63 …(2)
from (1) and (2)
39 n(A B) 63.
Problem 12. Number of relations that can be defined on the set A = {a, b, c, d, e} is
(A) 24 (B) 16
4 25
(C) 4 (D) 2
25
Solution: Since n(A) = 5, therefore, n(A × A) = 25. Hence number of subsets of A × A = 2 .
25
So, number of relations on A = 2 .
Problem 13. If f = {(1, 4), (2, 5), (3, 6)} and g = {(4, 8), (5, 7), (6, 9)}, then gof is
(A) { } (B) {(1, 8), (2, 7), (3, 9)}
(C) {(1, 7), (2, 8), (3, 9)} (D) none of these
2x 1
Problem 16. If S is the set of all real x such that 0 , then S contains which of the
x3 2x2 x
following intervals :
3 1 3 1
(A) , (B) ,
2 2 2 4
1 1 1
(C) , (D) , 3
4 2 2
P-2022-CBSE-P1-MATHEMATICS- SRF
20
2x 1 2x 1
Solution: 3 2
0 0
x 2x x x(x 1)2
2x 1
0, x 1, 0
x
2x 1 > 0 and x > 0 or, 2x 1 < 0 and x < 0
1
x or, x < 0
2
1
Hence S = ( , 0) , { 1}
2
1
Thus , 3 S.
2
Problem 17. Let f : [0, 1] [0, 1] and g : [0, 1] [0, 1] be two functions defined by
1 x
f(x) = and g(x) = 4x(1 x), then (fog) (x) =
1 x
1 4x 4x2 4(1 x)
(A) 2
(B)
1 4x 4x 1 x
8x(1 x)
(C) (D) none of these
(1 x)2
Problem 18. A survey shows that 63% of the Americans like cheese whereas 76% like apples.
If x % of the Americans like both cheese and apples, then
(A) x = 39 (B) x = 63
(C) 39 x 63 (D) none of these
Solution: Let A denotes the set of Americans who like cheese and B denote who like
apples. Let the population of Americans is 100. Then,
n(A) = 63 and n(B) = 76.
Now, n(A B) = n(A) + n(B) n(A B)
n(A B) = 139 n(A B).
But n(A B) 100, so that n(A B) 39.
Further, A B A and A B B imply that n (A B) n(A) and n(A B) n(B)
that is
n(A B) 63 and n(A B) 76]
Hence, 39 n(A B) 63 39 x 63.
1
Problem 19. If f(x) , then f(f(f(x))) is an (x 0, 1)
1 x
(A) a quadratic function (B) an identity function
(C) an exponential function (D) not a function
P-2022-CBSE-P1-MATHEMATICS-SRF
21
1 x 1
Solution: fof(x) , so
1 x
1
1 x
1
fofof(x) x.
x 1
1
x
Problem 20. Let A, B, C be subsets of the universal set U. If n(U ) = 692, n(B ) = 230, n(C ) =
370, n(B C) = 20, n(A B C ) = 10. Then n(A B C ) is equal to
(A) 172 (B) 170
(C) 370 (D) 140
P-2022-CBSE-P1-MATHEMATICS- SRF
22
CHAPTER PRACTICE PROBLEM
Subjective:
Level – I
1. Find the number of relations that can be defined on the set A = {a, b, c, d}.
3
3. Find range of the function f(x) = 2
.
2x 4x 3
Level – II
1
4. Find domain of the function f(x) = 9 x2 x 2
.
log10
Objective:
1. Let A = {1, 2, 3}, then the relation R = {1, 1), (1, 3)} on A is
(A) domain R is {1, 2, 3} (B) domain R is {1, 2}
(C) Range of R is {1, 3} (D) none of these
2. Let A = {1, 2, 3}, then the domain of the relation R = {(1, 1), (2, 3), (2, 1)} defined on A is
(A) {1, 2} (B) {1, 3}
(C) {1, 2, 3} (D) none of these
4. f(x) = x + 1, x 0
2x – 3, x < 0, then
(A) f(1) = 3 (B) f(–2) = –7
(C) f(0) = –3 (D) none of these
1
5. Domain of the function f (x) = contains the points
log10 (x 2)
(A) 2, 3, 4 (B) 3, 4, 5
(C) all natural numbers (D) 4, 5, 6
P-2022-CBSE-P1-MATHEMATICS-SRF
23
7. A B = A iff
(A) A B (B) B A
(C) A = B (D) A B =
8. A B = A if
(A) A B (B) B A
(C) A = B (D) A B =
9. A B is equal to
(A) B A (B) A B
(C) A B (D) A (A B)
2. Let X {1,2,3,4,5} and Y {1,3,5,7,9} . Which of the following is/are relations from X to Y
(A) R1 {(x,y) | y 2 x, x X, y Y} (B) R2 {(1,1),(2,1),(3,3),(4,3),(5,5)}
(C) R3 {(1,1),(1,3)(3,5),(3,7),(5,7)} (D) R 4 {(1,3),(2,5),(2,4),(7,9)}
3. Let A = {1, 2, 3, 4} and R be a relation in A given by R = {(1, 1), (2, 2), (3, 3), (4, 3), (1, 2),
(2, 1), (3, 1), (1, 3)}, then
(A) domain R is {1, 2, 3} (B) range R is {1, 2, 3, 4}
(C) domain R is {1, 2, 3, 4} (D) range R is {1, 2, 3}
x2
4. Let f(x) = , then
2 | x |
(A) domain of f(x) is R (B) domain of f(x) is R – {–2, 2}
(C) f(–1) = 1/3 (D) f(1) = 1
Numerical Based
x
1. If f(x) sinlog x , then the value of f(xy) f 2f(x).coslog y is equal to _____
y
1 x
2. If f(x) cos(logx) , then the value of f(x).f(4) f f(4x) is ______
2 4
3x 3 1
3. Numerical value of the expression for x 3 is _____
2x 2 2
P-2022-CBSE-P1-MATHEMATICS- SRF
24
4. If f be the greatest integer function and g be the modulus function, then
5 5
(gof ) (fog) __
3 3
2x 1
5. If f(x) , then (fof )(2) is equal to _______
3x 2
9. Sets A and B have 3 and 6 elements respectively. What can be the minimum number of
elements in A B
10. Let A = {1, 2, 3, 4, 5}; B = {2, 3, 6, 7}. Then the number of elements in (A × B) (B × A)
is
P-2022-CBSE-P1-MATHEMATICS-SRF
25
ASSIGNMENT PROBLEMS
Subjective:
Level – 0
1. Let R = {(1, 2), (2, 3), (3, 4), (4, 1), (1, 4)} be a relation in the set A = {1, 2, 3, 4}. Find the
domain and range of R.
Directions (36) Find the domain and range of the following functions:
3. f (x) = log2 x2 1 .
x 2 2x 3
4. f x .
x 2 2x 2
3
5. f x 1 cos x .
6. f x 1 1 x
7. If n (S) = 70,
n (AC B) = 30
n (A) = 30
then find (i) n (A B) (ii) n (AC BC).
9. If n (S) = 700, n (A) = 200, n (B) = 300 and n (A B) = 100, where S is a universal set,
then find n (AC BC).
10. Let A = {2, 3, 4, 5}, B = {3, 5, 7}, C = {2, 5, 8}. Verify that
(i) A (B C) = (A B) (A C)
(ii) A (B C) = (A B) (A C).
13. A and B are two sets having 3 and 5 elements respectively and having 2 elements in
common, then find the number of elements in A B.
14. Let A = {x: x R, |x| < 1}, B = {x: x R, |x – 1| 1} and A B = R – D, then find set D.
15. Let A = {x: x R, x 2} and B = {x: x R, x < 4}, then find set A B.
16. For any natural number a, we define aN = {ax : x N}. If b, c, d N such that
bN cN = dN, then prove that d is the lcm of both b and c.
P-2022-CBSE-P1-MATHEMATICS- SRF
26
17. If A = {1, 2, 3}; B = {2, 4}, then prove that A = A (A B)
20. Let U = {1, 2, 3, 4, 5, 6, 7, 8, 9, 10), A = {1, 2, 5), B = (6, 7), then find A B.
x 2 ; x 0,3
22. The relation f defined by f(x) = , prove that f is function.
3x; x 3,10
1 x 3x x3
25. If f(x) = log and g(x) , then prove fog(x) = 3f(x).
1 x 1 3x 2
Level – I
3. The Cartesian product A A has 9 elements among which are found (–1, 0) & (0, 1). Find
set A
4. Participation in sports is compulsory in a school and A class has 80 student out of which
60 students play football and 40 students play basketball. Find how many students
(a) play football only,
(b) play basketball only,
(c) Play both football and basketball.
7. If A and B are two sets then prove that (A B) (A B) = A.
8. If R is a relation from a finite set A having m elements to a finite set B having n elements,
then find the number of relations from A to B.
9. Let A = {1, 2, 3, 4} and R be a relation in A given by R = {(1, 1), (2, 2), (3, 3), (1, 2), (2, 1),
(3, 1), (1, 3)}. Then find range of the relation R.
P-2022-CBSE-P1-MATHEMATICS-SRF
27
Level – II
2. In a certain town 25% families own a phone and 15% own a car, 65% families own
neither a phone nor a car, 2000 families own both a car and a phone. How many families
live in the town?
x 2 ; x 0,2
6. The relation g is defined by g(x) = . Prove that g is not a function.
3x; x 2,10
7. Let f = {(1, 1), (2, 3), (0, –1), (–1, –3)} be a function from Z to Z defined by f(x) = ax + b for
some integers a, b. Find a & b.
8. Let A = {9, 10, 11, 12, 13} and let f : A N be defined by f(n) = then highest prime factor
of n. Find the range of f.
9. Out of 800 boys in a school, 224 played cricket, 240 played hockey and 336 played
basketball. Of the total, 64 played both basketball and hockey; 80 played cricket and
basketball and 40 played cricket and hockey ; 24 played all the three games. Find the
number of boys who did not play any game.
P-2022-CBSE-P1-MATHEMATICS- SRF
28
Objective:
Level – I
5. The range of f (x) = sin3 x in domain , is
2 2
(A) [ 1, 1] (B) [ 1, 0]
(C) [0, 1] (D) none of these
1
6. The range and domain of y = are
1 x2
(A) R, R (B) (0, 1], R
(C) R+, R (D) none of these
7. The range of y = sin2x + 2 sinx + 5 contains
(A) 3 integers (B) 4 integers
(C) 5 integers (D) none of these
P-2022-CBSE-P1-MATHEMATICS-SRF
29
2
x 2
12. Range of the function f (x) = is
2x 3
(A) ( , 1] [2, ) (B) [1, 2]
(C) ( , 0) [2, ) (D) (0, 2]
13. Domain and range of the relation R = {(x, y) : x + y = 2, x > 0, y > 0} are
(A) (0, 3), (0, 3) (B) [0, 3), [0, 3)
(C) (0, 2], (0, 2] (D) (0, 2), (0, 2)
x 1
17. If f(x) , then f(2x) is
x 1
f(x) 1 3f(x) 1
(A) (B)
f(x) 3 f(x) 3
f(x) 3 f(x) 3
(C) (D)
f(x) 1 3f(x) 1
2
20. The value of the function f(x) 3 sin x 2 is defined when x
16
3
(A) , (B) 0 ,
4 4 2
(C) (3, 3) (D) none of these
P-2022-CBSE-P1-MATHEMATICS- SRF
30
Level – II
2. Two finite sets A and B have m and n elements. Number of subsets of A is 56 more than
that of B. The value of m and n are
(A) 6, 3 (B) 7, 6
(C) 7, 3 (D) none of these
2 2
3. Let P = {x; x N and x satisfies x + 2x – 3 = 0} and Q = {x; x I and x satisfies 2x – 3x
+ 1 = 0}, then
(A) P = Q (B) P Q =
(C) P Q = R (D) none of these
5. If Q is the set of rational numbers and P is the set of irrational numbers, then
(A) P Q = (B) P Q
(C) Q P (D) P Q =
6. If A and B are finite sets with m and n elements respectively, then the number of relations
from A to B is
(A) mn (B) 2mn
m+n
(C) 2 (D) none of these
2
x ; x 0,3
7. The relation f defined by f(x) = , then f(1) f(5) is equal to
3x; x (3,10]
(A) 10 (B) 5
(C) 15 (D) 20
8. The range of the relation R = {(–1, 1), (–1, 2), (2, 3), (1, 5)}
(A) {–1, 1, 2} (B) {–1, 1}
(C) {1, 2, 3} (D) {1, 2, 3, 5}
9. Suppose that A1, A2, A3, …, A30 are thirty sets, each containing 6 elements and B1, B2, B3,
30 n
…, Bn are n sets, each containing 3 elements. If Ai S Bi and each element of S
i 1 i 1
belongs to exactly 10 Ai’s and to exactly 9 Bi’s, then n =
(A) 45 (B) 54
(C) 18 (D) none of these
2n+2
10. If X = {3 8n 9 : n N} and Y = {64n : n N}, then
(A) Y X (B) X Y
(C) X = Y (D) none of these
11. If X = {23n 1 : n N} and Y = {7n : n N}, then
(A) X = Y (B) X Y
(C) Y X (D) none of these
P-2022-CBSE-P1-MATHEMATICS-SRF
31
2n
12. If P = {3 1 : n N} and Q = {8n : n N}, then
(A) P Q (B) P = Q
(C) Q P (D) none of these
17. A (B C) equals to
(A) A B A C (B) A B A B C
(C) B C A C (D) B C A B C
22. Let R = {x; x I & 2 < |x| < 5} and R = {x; x N & x < 3}, then
(A) R = {3, 4} (B) R = {1, 2, 3, 4…. 8}
(C) R = {–4, –3, 3, 4} (D) R = {0, 1, 2, 3 ….. 8}
x ex
23. Let f(x) = and g(x) = , then
1 x2 1 | x |
(A) domain f(x) is R (B) domain g(x) is R – {1}
(C) domain f(x) is R – {1} (D) domain g(x) is R – {–1, 1}
P-2022-CBSE-P1-MATHEMATICS- SRF
32
2x 1
24. If S is the set of all real numbers x for which 0 , and P is the subset of S,
2x 3x 2 x
3
then P can be
3 1 1
(A) , (B) , 0
2 4 2
1
(C) , 3 (D) (0, )
2
x 1; x 0
25. Let f x 2 , then
x ; x 0
(A) f(–2) = 4 (B) f(3) – f(–2) = 0
(C) f(f(–2)) = 5 (D) none of these.
26. Let f(x) sin and D = {x : f(x) > 0}, then D contains
x
1 1 1 1
(A) , (B) ,
3 2 5 4
1 1
(C) 1, (D) ,
2 2
28. Let A = {1, 2, 3, 5} & B = {4, 6, 9} relation R from A to B is defined by R = {(x, y); the
difference between x & y is odd; x A, y B}, then
(A) domain of R is {1, 2, 5} (B) domain of R is {1, 2, 3, 5}
(C) range of R is {4, 6, 9} (D) domain of R is {4, 6}
Reason Assertion
P-2022-CBSE-P1-MATHEMATICS-SRF
33
COMPREHENSIONS
(I)
Read the following write up carefully and answer the following questions:
All possible real values of x for which a function y = f (x) is defined, is called Domain of f (x).
e.g. y = logx e is defined for all possible values of x except x = 1.
Domain = {x : x (0, ) {1}}.
1
31. Domain of f (x) = is given by
x 2 x2 2x
(A) (–1, 2) (B) (–1, )
(C) (2, ) (D) (–, –1)
(II)
Read the following write up carefully and answer the following questions:
x ; x 2, 1
Let f(x) = , g x 2 sin x; x ,
x 1; x (1,2] 2 2
(III)
Read the following write up carefully and answer the following questions:
For finding range we sometimes use the quadratic equation for example f(x) = x 2 + 2x + 2 will
have range [1, ) as f(x) = x2 + 2x + 2 = (x + 1)2 + 1 1.
2
35. Range of sin x + 2sinx + 2 is
(A) [1, 5] (B) [1, )
(C) [1, 1] (D) (0, )
P-2022-CBSE-P1-MATHEMATICS- SRF
34
Matrix Match
P-2022-CBSE-P1-MATHEMATICS-SRF
35
Numerical Based
40. A market research group conducted a survey of 1000 consumers and reported that 720
consumers like product A and 450 consumers like product B. Then minimum number of
consumers who liked both product is 100a + 10b + c, then a + b + c is _______
41. Two finite sets have m and 3 elements respectively. The total number of subsets of first
set is 56 more than the total number of subsets of the second set. The value of m is
__________
42. Sets A and B have 3 and 6 elements each. The maximum possible number of elements
in A B is __________
P-2022-CBSE-P1-MATHEMATICS- SRF
36
ANSWERS TO CHAPTER PRACTICE PROBLEM
Subjective:
Level – I
16
1. 2
2. [–1, 4]
3. (0, 3]
Level – II
4. (–2, 3] – {–1}
5. (−, 1)
Objective:
1. C 2. A 3. B 4. B
5. D 6. A 7. D 8. B
9. D 10. B
1. A, B 2. A, B 3. C, D 4. B, D
5. B, C
Numerical Based
1. 0 2. 0 3. 4 4. 1
5. 2 6. 9 7. 6 8. 2
9. 6 10. 4
P-2022-CBSE-P1-MATHEMATICS-SRF
37
ANSWERS TO ASSIGNMENT PROBLEMS
Subjective:
Level – 0
1. Domain {1, 2, 3, 4}
Range {1, 2, 3, 4}
2. Range : {0, 1, 2 , 3 , 2}
Domain: {0, 1, 2, 3, 4}
3. Domain: R
Range [0, )
4. Domain : R
Range (1, 2]
5. Domain : R
Range : [0, 8]
6. Domain: [-1, )
Range: [1, )
7. n (A B) = 60
n (AC BC) = 10
sin x
8. (a) sin x + x, sin x x, x sin x, ,x0
x
(b) cos x + ex, cos x ex, ex cos x, ex cos x
9. 300
11. {(1, 3), (2, 3), (3, 3), (8, 3)} 12. {(4, 2), (4, 3), (4, 4), (4, 5)}
20. A 23. 3
Level – I
P-2022-CBSE-P1-MATHEMATICS- SRF
38
Level – II
1. 9 2. 40,000
7. a = 2, b = – 1 8. {3, 5, 11, 13}
9. 160 10. {(2, 4), (3, 4)}
Objective:
Level – I
1. B 2. B 3. B 4. A
5. A 6. B 7. C 8. B
9. D 10. B 11. D 12. A
13. D 14. A 15. A 16. D
17. B 18. B 19. B 20. A
Level – II
1. D 2. A 3. A 4. C
5. A 6. B 7. C 8. D
9. C 10. B 11. B 12. A
13. B 14. B 15. B 16. A, D
17. A, B 18. A, B, C 19. A, B, C, D 20. A, B, C
21. A, B, C 22. B, C 23. A, D 24. B, C
25. A, B, C 26. A, B, C 27. A, B, D 28. B, C
29. B, D 30. C 31. A 32. C
33. C 34. D 35. A 36. B
37. (A) (q, r) (B) (r) (C) (p, q) (D) (r, s)
38. C
39. (I). C (II). D (III). C
40. 8 41. 6 42. 9
P-2022-CBSE-P1-MATHEMATICS-SRF
PINNACLE-CBSE
Angle:
Let a revolving line, starting from OX, revolve about O in a plane in the direction of the arrow (in
anticlockwise direction) and occupy the position OP. Then it is said to trace an angle XOP. OP is
called the final (or terminal) position of OX which is the initial position. The point O is called the
vertex. More precisely an angle may be defined as a measure of the rotation of a half ray about
its origin. An angle XOP is positive if it is traced by a ray revolving in the anticlockwise direction
and negative if it is traced by a ray revolving in the clockwise direction.
P Initial Position
X
O
Terminal Position Negative Angle
Positive Angle
Terminal Position
X
O Initial Position P
Measurement of an Angle:
P-2022-CBSE-P1-MATHEMATICS-TRI
2
Circular System (radian measure):
In this system an angle is measured in radians. A radian is an angle subtended at the centre of a
circle by an arc whose length is equal to its radius.
Let AB be an arc of a circle of radius r such that length of the
B
arc AB = r. Then AOB = 1 radian (written as 1C).
Since the whole circle subtends an angle of 3600 (4 right
r
angles) at the centre and the angles at the centre of a circle O
are in the ratio of subtending arcs, r
AOB arc AB A
so that
4 right angles circumference
arc AB 4 right angles
AOB =
2r
r 2
1 radian = 4 right angles = right angle radians = 2 right angles = 1800 = 200g
2r
180 180 7 6300
1 radian = degree = = 5701622 nearly.
22 11
Illustration 1. Express in radians the fourth angle of a quadrilateral which has three angles
0 0 0
60 , 55 and 100 .
22
Illustration 2. The difference between two acute angles of a right angled triangle is
35
radians. Find these angles in degrees.
P-2022-CBSE-P1-MATHEMATICS-TRI
3
22 22 180
x–y= radian = degrees
35 35
180 0
or x – y = = 36 . ……(1)
5
0
Also x + y = 90 ……(2)
0 0
x = 63 and y = 27 .
g
4x 2 x
Illustration 3. The angles of a triangle are 3x0, and radians. Find all the angles.
3 75
0 g 0
g 0 g 9 4x 36x
Solution: Since 100 = 90 1 =
10 3 30
1800
and C = 1800 1C =
C 0
2x 24x 36x 24x
3x + = 180
75 5 30 5
9x = 1800 x = 200
0 0 0
60 , 24 , 96 are angles of the triangle.
Illustration 4. A circular wire of radius 3 cm is cut and bent so as to lie along a circle of radius
48 cm. Find the angle subtended by the wire at the centre of the circle.
Y Y
P P
r
r
y
X X X
X O x M M O
Y Y
P-2022-CBSE-P1-MATHEMATICS-TRI
4
Y Y
M O M
X X X
O X
P P
Y Y
Trigonometric ratios (or functions) may also be defined with respect to a triangle.
In a right angled triangle ABC, CAB = A and BCA = 90° = /2. AC is the base, BC the altitude
and AB is the hypotenuse. We refer to the base as the adjacent side and to the altitude as the
opposite side. With reference to angle A, the six trigonometric ratios are:
BC opposite side B
is called the sine of A, and written as sinA.
AB hypotenuse
AC adjacent side
is called the cosine of A, and written as cosA.
AB hypotenuse
BC opposite side
is called the tangnet of A, and written as tanA. A C
AC adjacent side
sinA
Obviously, tan A= . The reciprocals of sine, cosine and tangent are called the cosecant,
cosA
secant and cotangent of A respectively. We write these as cosecA, secA, cotA respectively. Since
the hypotenuse is the greatest side in a right angle triangle, sinA and cosA can never be greater
than unity and cosecA and secA can never be less than unity. Hence sin A 1,
cos A 1, cosec A 1, sec A1, while tan A and cot A may take any numerical value.
Notes:
All the six trigonometric functions have got a very important property in common that is
of periodicity.
Remember that the trigonometrical ratios are real numbers and remain same as long as
angle A is real.
P-2022-CBSE-P1-MATHEMATICS-TRI
5
Trigonometrical Identities:
An identity is a relation which is true for all values of the independent variable. There are three
fundamental identities involving trigonometrical ratios:
i) sin2 + cos2 = 1 ii) 1 + tan2 = sec2
2 2
iii) 1 + cot = cosec
cos A sin A
Illustration 5. Prove that = sin A + cos A.
1 tan A 1 cot A
1 cos
Illustration 6. Prove that = cosec + cot .
1 cos
Illustration 7. Eliminate ‘’ from the equations x = a cos + b sin , y = a sin – b cos .
Solution: Squaring and adding the given equations we get:
2 2 2 2 2 2 2 2 2 2
x +y = a cos +b sin +2ab cos sin +a sin + b cos – 2ab cos sin
x2+y2 = a2(cos2 + sin2 ) + b2(sin2 + cos2 ) x2+y2 = a2 + b2.
n n
Illustration 8. If un = cos + sin , then prove that 2u6 - 3u4 + 1 = 0.
4 4 2 2
Also u4 = cos + sin = 1 – 2cos sin
2 2 2 2
2u6 – 3u4 + 1 = 2 – 6cos sin – 3 + 6cos sin + 1 = 0.
P-2022-CBSE-P1-MATHEMATICS-TRI
6
Illustration 9. Express tan in terms of cos.
Solution: By definition y
OA x B
cos x where OB is taken as
OB 1 1
unity and OA = x. 1 x 2
Hence in AOB, OA = x, OB = 1, O
x A x
AB 1 x 2 . Fig. 3
By definition,
AB 1 x2 1 cos2
tan = .
OA x cos
21 5
Illustration 10. If sin = and ‘’ lies in the second quadrant, show that sec + tan = – .
29 2
Solution: Since lies in the second quadrant, cos is negative and tan is also negative.
2 2
2 2 21 400 20
Now cos = 1 sin = 1
29 (29)2 29
20 sin 21
cos = and hence tan = .
29 cos 20
29 21 50 5
Hence sec + tan = .
20 20 20 2
Exercise 1.
1 1 1 1
i) Prove that .
secA tanA cosA cosA secA tanA
2 2 3 2 3/2
ii) If tan = 1 – e then show that sec + tan cosec = (2 – e ) .
4xy
iii) For what real values of x and y is the equation sec2 = possible?
2
x y
iv) Prove the following trigonometric identities:
1 sin 2
(a) sec (1 – sin )(sec + tan ) = 1 (b) = (sec – tan )
1 sin
sin x 1 cos x tan A sec A 1
(c) = 2 cosec x (d) =secA+tanA
1 cos x sin x tan A sec A 1
(e) (cosec – sec )(cot – tan ) = (cosec + sec )(sec cosec – 2).
p p sin q cos p 2 q 2
v) If tan = show that .
q p sin q cos p 2 q 2
P-2022-CBSE-P1-MATHEMATICS-TRI
7
Trigonometric Ratios of Standard Angles
P-2022-CBSE-P1-MATHEMATICS-TRI
8
Trigonometric Ratios of 900:
0 Y
Let the revolving line OX, trace POX = 90 . From P, perpendicular
PM is drawn to OX so that M coincides with O. Hence P
PM y OP y
sin 900 = = 1, cosec900 = 1
OP y PM y 900
X
OM 0 y O M
cos 900 = = 0, sec 900 = (not defined),
OP y 0
PM y 0
tan 900 = (not defined), cot 900 = = 0.
OM 0 y
Illustration 12. If sec + tan = 4, find sin , cos . Also find the quadrant in which ‘’ lies.
Illustration 14. If A, B, A – B and A + B are positive acute angles, find the values of A and B from
1 1
the equations sin (A – B) = , cos (A + B) = .
2 2
1
Solution: The equations are sin (A – B) = = sin 300 A – B = 300. …(1)
2
1 0 0
And cos (A + B) = = cos 60 A + B = 60 . ...(2)
2
From (1) and (2), A = 450 and B = 150.
P-2022-CBSE-P1-MATHEMATICS-TRI
9
Allied Angles
Trigonometrical Ratios of Angle (–) in Terms of :
From the figure, we find that
y Y
sin (–) = = – sin ,
r P
x
cos (–) = = cos , r
r y
y x
tan (–) = = – tan ,
x X
O – M
x
cot (–) = = – cot , r –y
y
r
cosec (–) = = – cosec , P
y
r
sec (–) = = sec .
x
Trigonometrical Ratios of Angle (900 – ) in Terms of :
P-2022-CBSE-P1-MATHEMATICS-TRI
10
Trigonometrical Ratios of (1800 – ) in Terms of :
P-2022-CBSE-P1-MATHEMATICS-TRI
11
Table – I
Note:
Angle and 90° – are complementary angles, and 180° – are supplementary
angles
sin(n + (– 1)n) = sin, n
cos(2n ± ) = cos, n
tan(n + ) = tan, n
i.e. sine of general angle of the form n + (–1)n will have same sign as that of sine of angle and
so on. The same is true for the respective reciprocal functions.
Illustration 15. Prove that sin (–6900) cos (–3000) + cos (–7500) sin (2400) = 1.
Solution: We have sin (–6900) = – sin 6900 = – sin (3600 + 3300) = – sin 3300
1
= – sin (3600 – 300) = sin 300 = ,
2
1
cos (–3000) = cos 3000 = cos (3600 – 600) = cos 600 = ,
2
3
cos (–7500) = cos 7500 = cos (2 360 + 30)0 = cos 300 = ,
2
3
sin (–2400) = – sin 2400 = – sin (180 + 60)0 = sin 600 = .
2
1 1 3 3
Hence sin (–6900) cos (–3000) + cos (–7500) sin (–2400) = + = 1.
2 2 2 2
3 5 7
Illustration 16. Prove that cos2 + cos2 + cos2 + cos2 = 2.
8 8 8 8
5
Solution: We know that cos = cos = – sin
8 2 8 8
7 3 3
and cos = cos = – sin
8 2 8 8
2 2 3 2 5 2 7 2 2 3 2 2 3
cos +cos +cos +cos =cos +cos +sin +sin =2.
8 8 8 8 8 8 8 8
Illustration 17. Find x from the equation cosec (900 + ) + x cos cot (900 + ) = sin (900 + ).
Solution: The given equation cosec (900 + ) + x cos cot (900 + ) = sin (900 + )
P-2022-CBSE-P1-MATHEMATICS-TRI
12
sec + x cos (– tan ) = cos sec – x sin = cos
1 cos2 sin2
x sin = sec – cos x = = tan .
cos sin cos sin
1 1
Illustration 18. Find the general value of satisfying both sin and tan .
2 3
Exercise 2.
i) Find the value of tan1 tan2 tan3 …. tan89.
1
ii) Prove that sin25 + sin210 +…. + sin290 = 9 .
2
iii) If cos – sin = 2 sin , show that cos + sin = 2 cos .
iv) Prove that tan700 = 2 tan500 + tan200.
Compound Angle:
An angle made up of algebraic sum of two or more angles is called compound angle. If A, B and
C are given angles, then A + B, A – B, A + B + C, A – B + C, A + B – C etc. are compound
angles.
P-2022-CBSE-P1-MATHEMATICS-TRI
13
In OQM, Z
Q A
QM QH PL QH PL
sin(A + B) = =
OQ OQ OQ OQ A
QH QP PL OP Y
= = cos A sin B + sin A cos B
QP OQ OP OQ H P
A
OM OL LM OL HP
and cos(A + B) =
OQ OQ OQ OQ
OL OP HP PQ B
=
OP OQ PQ OQ A
X
= cosA cosB – sinA sinB . O M L
QH PL PQ PL
QM QH PL OL OL
Moreover, tan(A + B) = OP OL .
OM OL ML PH PH PL
1 1
OL PL OL
tan A tanB QH PH PQ
= since in similar trianlges HPQ and OPL .
1 tan A tanB OL PL OP
sin A B sin A cosB cos A sinB tan A tanB
Alternatively: tan(A + B) = .
cos A B cos A cosB sin A sinB 1 tan A tanB
Subtraction Formulae:
P-2022-CBSE-P1-MATHEMATICS-TRI
14
PL QH PL QH
QM HM QH PL QH OL OL
Moreover, tan (A – B) = OL OL .
OM OL LM OL PH PH PH PL
1 1
OL PL OL
From similar triangles QPH and POL
QH PH PQ
OL PL OP
PL PQ
tan A tanB
tan (A – B) = OL OP = .
PQ PL 1 tan A tanB
1
OP OL
Subtraction formulae can also be derived from the addition formulae by writing
sin (A B) = sin (A + ( B)) = sin A cos ( B) + cos A sin ( B) = sin A cos B cos A sin B
and so on.
Illustration 19. Find the values of (a) cos 150 (b) sin 1050.
Solution: We know that cos 150 = cos (45 – 30)0 = cos 450 cos 300 + sin 450 sin 300
1 3 1 1 3 1
= .
2 2 2 2 2 2
3 1
Also sin 1050 = sin (45 + 60)0 = sin 450 cos 600 + cos 450 sin 600 = .
2 2
Illustration 20. Prove that cos 180 – sin 180 = 2 sin 270.
0 0 0 0 0 0
Solution: RHS = 2 sin 27 = 2 sin (45 – 18) = 2 (sin 45 cos 18 – cos 45 sin 18 )
cos180 sin180 0 0
= 2 = cos 18 – sin 18 = LHS.
2
0 0 0
Illustration 21. Prove that tan 70 = 2 tan 50 + tan 20 .
tan 700 = tan 700 tan 500 tan 200 + tan 500 + tan 200
0 0 0 0 0 0 0 0
tan 70 = cot 20 tan 50 tan 20 + tan 50 + tan 20 = 2 tan 50 + tan 20 .
P-2022-CBSE-P1-MATHEMATICS-TRI
15
Transformation Formulae:
(a) Transformation of products into sums or differences:
(i). 2 sin A cos B = sin (A + B) + sin (A – B)
(ii). 2 cos A sin B = sin (A + B) – sin (A – B)
(iii). 2 cos A cos B = cos (A + B) + cos (A – B)
(iv). 2 sin A sin B = cos (A – B) – cos (A + B)
The above formulae can be easily derived by taking the sum and the difference of the addition
and subtraction formulae.
(b) Transformation of sums or differences into products:
(i). sin (A + B) + sin (A – B) = 2 sin A cos B
CD C D
or, sin C + sin D = 2 sin cos
2 2
(ii). sin (A + B) – sin (A – B) = 2 cos A sin B
CD C D
or, sin C – sin D = 2 cos sin
2 2
(iii). cos (A + B) + cos (A – B) = 2 cos A cos B
CD C D
or, cos C + cos D = 2 cos cos
2 2
(iv). cos (A – B) – cos (A + B) = 2 sin A sin B
CD DC
or, cos C – cos D = 2 sin sin
2 2
CD DC
Here A – B = C and A + B = D A = and B = .
2 2
sin(A B)
(v). tan A + tan B = .
cos A cosB
P-2022-CBSE-P1-MATHEMATICS-TRI
16
(iv). Here sin 3A = sin (A + 2A) = sin A cos 2A + cos A sin 2A
2 2 3
= sin A (1 – 2 sin A) + 2 sin A (1 – sin A) = 3 sin A – 4 sin A
= sin A [ 3 2 sin A] [ 3 + 2 sin A]
3 3
= 4 sin A sin A sin A
2 2
= 4 sin A (sin2 600 sin2 A)
= 4 sin (600 A) sin A sin (600 + A).
(v). Here cos 3A = cos (A + 2A) = cos A cos 2A – sin A sin 2A
2 2 3
= cos A (2 cos A – 1) – 2 cos A (1 – cos A) = 4 cos A – 3 cos A
3 3 2 2 0
= 4 cos A cos A cos A = 4 cos A (cos A sin 60 )
2 2
0 0
= 4 cos (60 A) cos A cos (60 + A).
tan A tan 2A
(vi). Also tan 3A =
1 tan A tan 2A
2 tan A
tan A 3 3
= 1 tan2 A tan A tan A 2 tan A = 3 tan A tan A .
2 tan A 2 2
1 tan A 2 tan A 2
1 3 tan A
1 tan A
1 tan2 A
tan A tanB
Solution: Since tan(A + B) = 1,
1 tan A tanB
tanA + tanB + tanA tanB + 1= 1 + 1
or tanA(1+ tanB) + (1+ tanB) = 2 or (1+ tanA) (1+ tanB) = 2.
Illustration 23. Find the values of (i) sin 18°, (ii) tan 15°.
Solution: (i) Let = 18°. Then 2 = 36° = 90° – 54° = 90° – 3.
Now sin2 = 2sin cos and
sin(90° – 3) = cos3 = 4cos3 – 3cos.
Hence we have 2sin cos = cos (4cos2 – 3) = cos (1 – 4sin2)
2
or 2 sin = 1 – 4sin (as cos 0)
2 2 4 16 1 5
4sin + 2sin – 1 = 0 sin = .
2.4 4
5 1 5 1
Since sin > 0, we have sin = i.e. sin18° = .
4 4
(ii) Let = 15° 2 = 30°.
2 tan 1
Since tan 2 2
tan 30 ,
1 tan 3
2 2 3 12 4 2 3 4
tan + 23 tan – 1 = 0 tan = 3 2.
2 2
For tan > 0, we ignore negative value
and get tan = 2 – 3 i.e. tan15° = 2 – 3.
P-2022-CBSE-P1-MATHEMATICS-TRI
17
Alternative solution:
2
Solution: Since , and are in A.P., 2 = + cot = cot
2
cos 2 cos sin
= 2 2 2 = sin sin .
cos cos
sin 2 sin sin
2 2 2
Exercise 3.
i) Show that cot x cot x = 1.
4 4
ii) Given 3 tan tan = 1, show that 2cos( + ) = cos( – ).
2
iii) Prove that 2sin +4cos( + ) sin. sin + cos2( + ) is independent of .
iv) If tan = b/a, then find the value of acos2 + bsin2.
P-2022-CBSE-P1-MATHEMATICS-TRI
18
A
2 tan
(iii). tan A = 2
2 A
1 tan
2
A A
(iv). sin A = 3 sin – 4 sin3
3 3
A A
(v). cos A = 4 cos3 – 3 cos
3 3
A A
3 tan tan3
(vi). tan A = 3 3
2 A
1 3 tan
3
A A
sin cos 1 sin A.
2 2
A 3
A A if 2n 2n
or sin + cos = 1 sin A 4 2 4
2 2 otherwise
A A
sin cos 1 sin A
2 2
A 5
A A if 2n 2n
or sin – cos = 1 sin A 4 2 4
2 2 otherwise
A tan2 A 1 1
tan =
2 tan A
A
The ambiguities of signs are removed by locating the quadrant in which lies.
2
A A
sin + cos is + ve
2 2
A A
sin – cos is + ve
2 2
A A A A
sin 2 + cos 2 is – ve sin 2 + cos 2 is + ve
/4
A A A A
sin 2 –- cos 2 is + ve sin 2 – cos 2 is – ve
A A
sin 2 + cos 2 is – ve
A A
sin 2 –- cos 2 is – ve
| a cosA + b sinA | a2 b2
Also cosA sinA =
2 sin A 2 cos A .
4 4
Notes:
A
Any formula that gives the value of sin in terms of sinA shall also give the value of
2
n
n -1 A
sine of .
2
P-2022-CBSE-P1-MATHEMATICS-TRI
19
Any formula that gives the value of cos(A/2) in terms of cosA shall also give the value of
2n A
cos of .
2
Any formula that gives the value of tan(A/2) in terms of tanA shall also give the value of
n A
tan of .
2
Exercise 4.
i) Find the value of cos22(1/2)º.
1e cos e
ii) If tan tan , prove that cos = .
2 1e 2 1 e cos
iii) Prove that (cos – cos)2 + (sin – sin)2 = 4sin2 .
2
2 4
iv) Prove that sin + sin + sin = 0.
3 3
P-2022-CBSE-P1-MATHEMATICS-TRI
20
Identities
A trigonometric equation is an identity if it is true for all values of the angles involved. A given
identity may be established by reducing either side equal to the other one, or reducing each side
to the same expression, or any convenient modification of these.
A B C
Solution: We have , so that
2 2 2 2
A B
tan tan
A B C 2 2 1
tan tan
2 2 2 2 A B C
1 tan tan tan
2 2 2
A B B C C A
tan tan tan tan tan tan 1
2 2 2 2 2 2
2 A B C 1 A A B
tan tan2 tan2 1 = 2 tan2
2 tan 2 tan 2
2 2 2 2 2
2 2 2
1 A B B C C A
= tan tan tan tan tan tan 0 .
2 2 2 2 2 2 2
P-2022-CBSE-P1-MATHEMATICS-TRI
21
sin 2A sin 2B sin 2C
Illustration 30. If A + B + C = , prove that = 8cos A/2 cosB/2 cosC/2.
cos A cos B cos C 1
Illustration 31. Find the maximum and minimum values of a cos + b sin.
P-2022-CBSE-P1-MATHEMATICS-TRI
22
Exercise 5:
2 4 8 16 1
i) Prove that cos .cos cos .cos = .
15 15 15 15 16
ii) Prove that 5cos + 3cos + 3 lies between –4 and 10.
3
iii) Find minimum and maximum value of y = 7cos + 24sin.
3 5 7 3
iv) Prove sin 4 sin 4 sin 4 sin 4 .
16 16 16 16 2
Solution: Since sin x is periodic with period 2, it is sufficient to sketch its graph only for
0 x 2. It can be easily extended by repeating it over the intervals of length
2. We have the following table:
x 0 2 5 7 4 3 5 11 2
6 3 2 3 6 6 3 2 3 6
sin x 0 1 3 1 3 1 0 1 3 1 3 1 0
2 2 2 2 2 2 2 2
-/2 3/2
-
x
-2 -3/2 0 /2 2
Solution : tan x is periodic with period . As x increases from 0 to /2, tan x keeps
increasing from 0 to . As x crosses the value /2, tan x becomes negative and
increases from - to 0 with x increasing from /2 to . We have the following
table:
2 3 5
x 0
6 4 3 3 4 6
1 1
tan x 0 1 3 3 -1 0
3 3
P-2022-CBSE-P1-MATHEMATICS-TRI
23
y
Note:
2
For y = b sin ax, y = b cos ax, a 0 we find that sin a x sin ax 2 sin ax
a
2
and cos a x cos ax 2 cos ax . The period of both of these
a
2
function is . The amplitude of both the curves y = b sin ax and y = b cos ax is |b|. The
a
graph is magnified if |b| 1and is contracted if |b| 1.
Miscellaneous Exercise:
4 3 sin A cos A
i) If sin A = , find the value of .
5 4 cos ecA 3 tan A
n tan A
ii) If tan B = , prove that tan (A B) = (1 n) tan A.
1 (1 n ) tan 2 A
iii) Prove that sec A sec A 2 sec 2 A .
4 4
1 1 xy
iv) If cos x + cos y = , sin x + sin y = , find the values of tan and tan (x + y).
8 4 2
v) If sinx + siny = 3 (cosy – cosx) show that sin3x + sin3y = 0
cos A B cos C D
vi) If 0 then prove that tanA.tanB. tanC tanD = – 1
cos A B cos C D
sin 1m
vii) If , prove that tan tan = m.
cos 1 m 4 4
3
viii) Prove that sin 200 sin 400 sin 800 = .
8
9 3 5
ix) The value of 2 cos cos cos cos is equal to
13 13 13 13
(A) 2 (B) 0
(C) 1 (D) 3
P-2022-CBSE-P1-MATHEMATICS-TRI
24
xi) The value of 2 2 2 2 cos 8 where 0 < < is equal to
8
(A) 2cos (B) cos
(C) 2sin (D) 2cos
5 3
xii) If tan and tan , then the value of cos( + ) is
2 2 2 4
364 627
(A) (B)
725 725
240
(C) (D) none of these
339
1 tan2 15
xiii) The value of is
1 tan 2 15
(A) 1 (B) 3
3
(C) (D) 2
2
3 5 7 9 11 13
xv) The value of sin sin sin sin sin sin sin is
14 14 14 14 14 14 14
1 1
(A) (B)
16 64
1
(C) (D) none of these
128
P-2022-CBSE-P1-MATHEMATICS-TRI
25
ANSWERS TO EXERCISES
Exercise 1.
iii) x=y
Exercise 2.
i) 1
Exercise 3.
iv) a
Exercise 4.
1
i) 2 2
2
Exercise 5.
iii) minimum = 25
maximum = 25
Miscellaneous Exercise:
63 3 24
i) iv) ,
20 4 7
ix) B x) D
xi) A xii) B
xiii) C xiv) A
xv) B
P-2022-CBSE-P1-MATHEMATICS-TRI
26
SOLVED PROBLEMS
Subjective:
Level 0
3 5 7 1
Problem 1. Prove that 1+cos 1+cos 1+cos 1+cos .
8 8 8 8 8
5
Solution: Writing cos cos sin
8 2 8 8
3 7
cos cos sin , cos cos cos
8 2 8 8 8 8 8
L.H.S. = 1 cos 1 cos 1 sin 1 sin
8 8 8 8
2
1 1 1 1
= 1 cos2 1 sin2 sin2 cos2 sin = . R.H.S.
8 8 8 8 4 4 4 2 8
A B m 1 BA
Problem 2. If cosA = m cosB, then prove that cot tan .
2 m 1 2
Problem 3. Let 0 < A, B < satisfying the equation 3sin2A + 2sin2B = 1 and
2
3sin2A – 2sin2B = 0. Prove that A 2B .
2
P-2022-CBSE-P1-MATHEMATICS-TRI
27
3
A + 2B = or .
2 2
Given that 0 < A < and 0 < B < 0 < A + 2B < + .
2 2 2
Hence A + 2B = .
2
3 5 7 9 11 13 1
Problem 4. Pr ove that sin .sin .sin .sin .sin .sin sin .
14 14 14 14 14 14 14 64
Level I
P-2022-CBSE-P1-MATHEMATICS-TRI
28
Problem 6. If a cos2 + b sin2 = c has and as its solutions, then prove that
2b c a
tan + tan = , tantan = .
c a c a
Problem 7. For all in [0, /2], show that cos(sin ) > sin (cos ).
1 1
Solution: We have cos sin 2 cos sin
2 2
2 cos cos sin sin 2 cos
4 4 4
cos + sin 2 cos < - sin
2 2
sin(cos) < sin sin sin(cos) < cos(sin)
2
cos(sin) > sin(cos).
Alternative:
For 0 x /2, x sin x. ... (1)
Replace x by cos
cos sin(cos).
Take cosine of both sides of (1) and write for x
cos cos(sin). Hence sin(cos) < cos(sin).
Problem 8. Find the smallest positive number p for which the equation
cos (p sin x) = sin (p cos x) has a solution x [0, 2].
P-2022-CBSE-P1-MATHEMATICS-TRI
29
p . Hence the smallest value of p =
2 2 2 2
7
at x , .
4 4
Problem 9. Let cosA + cosB + cosC = 3/2 in a triangle ABC. Show that the triangle is
equilateral.
Solution: In a triangle A + B + C =
A B A B 3
cosA + cosB + cosC = 2 cos cos cosC
2 2 2
C A B C 3
2 cos cos 1 2sin2
2 2 2 2 2
C C A -B
4 sin2 – 4sin cos +1 = 0. …..(1)
2 2 2
Now sin(C/2) is real
A B
16 cos2 16 0
2
A B A B 2 A B
cos2 – 1 0 cos2 1 . But cos 1
2 2 2
A -B
cos2 =1
2
A = B. …(2)
Similarly it can be shown that B = C, C = A. Hence the triangle is equilateral.
Problem 10. Prove that 1 + cot cot for 0 < < . Find when equality sign holds.
2
2 tan cot 2
1
Solution: Since tan = 2
cot = 2 .
2
1 tan 2 cot
2 2
cot 2 1
2
Now 1 + cot – cot = 1 cot
2 2
2 cot
2
2
θ
2 cot cot 2 1 2 cot 2 cot 1
2 2 2 = 2
= 0 for 0 < <
θ
2 cot 2cot
2 2
1 + cot cot .
2
Equality holds when cot 1 0 = .
2 2
Alternative Solution:
P-2022-CBSE-P1-MATHEMATICS-TRI
30
c os cos / 2 cos 2 cos2 / 2
1 + cot cot= 1 1
2 sin sin / 2 sin 2 sin / 2 cos / 2
cos 1 cos
=1+ = 1 cosec 0.
sin sin
Level II
Hence 1 a2 sin2 1 c 2 sin2 a2 c 2 sin4
a2 c 2 cosec 2
2
Problem 12. If = , prove that tan tan 2 + tan 2 tan 4 + tan 4 tan = -7.
7
2
Solution: Given = cos ( + 2+ 4) = 1
7
cos ( + 2) cos 4 -sin ( + 2) sin 4 = 1
cos cos 2 cos 4 -sin sin 2 cos 4 -sin cos 2 sin 4 -cos sin 2
sin 4 = 1
1 –tan tan 2 -tan tan 4 -tan 2 tan 4 = sec sec 2 sec 4 ….(1)
Again; 7 = 2 sin 8 = sin (2 + )
8 sin cos cos 2 cos 4 = sin
sec sec 2 sec 4 = 8
Putting in (1); we get
tan tan 2 + tan tan 4 + tan 2 tan 4 = 7
Problem 13. If A, B, C are the angles of a triangle, then find the minimum value of
B C CA AB
cos cos cos
2 2 2 .
B C CA AB
cos cos cos
2 2 2
P-2022-CBSE-P1-MATHEMATICS-TRI
31
sin A sinB
Hence total sum is sinB sin A
sin A sinB
Now 2 (AM GM)
sinB sin A
given expression 6 and = 6 when A = B = C.
6 6
Problem 14. Consider the expression 3 sin x
3 cos x
. Find the minimum value.
6 6
Solution: 3sin x and 3cos x are positive numbers .
And A.M. G. M.
6 6
3sin x
3cos x
6
x cos6 x
3sin .... (1)
2
3
1 sin2 2x
= 2 3 4
1
6 6 1/8
3sin x
3cos x
2 3 4 = 2.3
Alternate:
6 6
Clearly the equality (1) holds for 3sin x
= 3cos x
6
1 1
sin6 x = cos6 x = .
2 8
tan tan
Problem 15. If , prove that sin2 + sin2 + sin2 = 0.
tan tan
P-2022-CBSE-P1-MATHEMATICS-TRI
32
Objective:
Level 0
True / False
Problem 1. If in ABC sin2A + sin2B + sin2C = 2, then the triangle is always right angled.
Above statement is true or false.
Solution: True
sin2A + sin2B + sin2C = 2
2 2 2
sin A + sin B 1 + sin C 1 = 0
sin2A cos2B cos2C = 0
2cosAcosBcosC = 0
either A = 90° or B = 90° or C = 90°.
Problem 2. If the sides of a right angled triangle are in G.P then the cosines of the acute
5 1 5 1
angles of the triangle are , .
2 2
Solution: False
a, b, c are in G.P b2 = ac A
and c2 = a2 + b2
c2 = a2 + ac
c
2
a a b
1 0
c c
a 1 5 B
=cos B = a C
c 2
5 1
cos B = (B is acute)
2
5 1
cos A = sin B = 1 cos2 B =
2
3
Problem 3. If < 2 < , then 2 2 2 cos 4 is – 2sin
2
Solution: False
2 2(1 cos 4) 2 2 | cos 2 |
= 2(1 cos 2)
3
= 2 | sin | = 2sin as .
2 4
Problem 4. The number of integral values of a for which the equation cos2x + a sinx = 2a – 7
possesses solutions is 5
Solution: True
The given equation may be written as
P-2022-CBSE-P1-MATHEMATICS-TRI
33
2
2 sin x – a sinx + 2a – 8 = 0,
a a 8 a4
so that sinx = sinx = .
4 2
a4
This is possible only if 1 1 or 2 a 6.
2
Solution: False
cosx varies from –1 to 1 for all real x.
Thus cos(cosx) varies from cos1 to cos0.
minimum value of cos(cosx) is cos1.
Solution: 1
cos2x (1 + cos2x) = cos2x + cos4x = cos2x + sin2x = 1.
x x
Problem 7. The maximum value of 4sin2 x + 3cos2x + sin cos is ____
2 2
Solution: 4 2
Maximum value of 4sin2x + 3cos2x i.e. sin2x + 3 is 4 and that of
x x
sin + cos = 2 , both attained at x = /2.
2 2
Hence the given function has maximum value 4 2 .
2ac
Solution:
a c2
2
P-2022-CBSE-P1-MATHEMATICS-TRI
34
Level I
Problem 9. If in a triangle ABC, C =90°, then the maximum value of sinA sinB is
1
(A) (B) 1
2
(C) 2 (D) None of these
Solution: A
1 1
sinA sinB = 2 sin A sinB = cos(A B) cos(A B)
2 2
1 1 1
= cos(A B) cos 90 = cos(A B)
2 2 2
1
Maximum value of sinA sinB = .
2
Solution: A
Maximum value of 2sinx + 4cosx = 2 5 . Hence the maximum value of
2sinx + 4cosx +3 is 2 5 3 .
Problem 11. If sin = 3sin( + 2), then the value of tan ( + ) + 2tan is
(A) 3 (B) 2
(C) 1 (D) 0
Solution: D
Given that sin = 3sin ( + 2) sin ( + ) = 3sin ( + + )
sin ( +)cos –cos( + ) sin =3sin ( + ) cos + 3cos ( + ) sin
–2sin ( + ) cos = 4cos ( + ) sin
sin( ) 2 sin
tan(+) + 2tan = 0.
cos( ) cos
Problem 12. If sin sin cos cos + 1 = 0, then the value of cot tan is
(A) –1 (B) 0
(C) 1 (D) none of these
Solution: A
Given sin sin – cos cos + 1 = 0
cos( + ) = 1 sin( + ) = 0
sin cos + cos sin = 0 cot tan = -1.
Problem 13. If cos 28 + sin28 = k3, then cos17 is equal to
k3 k3
(A) (B)
2 2
k3
(C) (D) none of these
2
P-2022-CBSE-P1-MATHEMATICS-TRI
35
Solution: A
cos17 = cos (45 – 28) = cos45 cos28 + sin45 sin28
cos 280 sin 280 k 3
= .
2 2
Level II
Problem 14. If sin, sin and cos are in G.P, then roots of the equation x2 + 2x cot + 1 = 0
are always
(A) equal (B) real
(C) imaginary (D) greater than 1
Solution: B
sin, sin, cos are in G.P.
sin2 = sin cos
cos2 = 1 – sin2 0
Now, the discriminant of the given equation is
4cot2 – 4 = 4 cos2 cosec2 0
Roots are always real.
2 ( n 1)
Problem 15. If S cos 2 cos 2 cos 2 , then S equals
n n n
n 1
(A) ( n 1) (B) ( n 1)
2 2
1 n
(C) ( n 2 ) (D)
2 2
Solution: C
2
S cos2 cos2 cos2 (n 1)
n n n
1 2 4 6
1 cos 1 cos 1 cos 1 cos 2(n 1)
2 n n n n
n1
1 2k 1 1
= n 1
2 cos
k 1
= n 1 1 n 2 .
n 2 2
Problem 16. If in a triangle ABC, sin2A + sin2B + sin2C = 2, then the triangle is always
(A) isosceles triangle (B) right angled
(C) acute angled (D) obtuse angled
Solution: B
sin2A + sin2 B + sin2C = 2
2 2 2
sin A – cos B + sin C = 1
or sin2C – (cos2B – sin2A) = 1
or - cos(B + A) cos(B – A) = cos2C
2
cosC cos(B – A) = cos C
cosC = 0 i.e. C = /2 or B – A = C B = /2 or A = /2.
P-2022-CBSE-P1-MATHEMATICS-TRI
36
Problem 17. Let k = 1, then 2 sin2k + 4 sin4k + 6 sin6k + … + 180 sin180k is equal to
(A) 90cosk (B) 90tan89
(C) 90 tank (D) 90cot89
Solution: B
Let y = 2 sin2 + 4 sin4 + … + 178 sin178
y = 178 sin178 + 176 sin176 + … + 2 sin2
2y = 180(sin2 + sin4 + … + sin178)
sin89
y = 90. sin 90 = 90tan89.
sin1
Problem 19. If , , , are the smallest positive angles in ascending order of magnitude
which have their sines equal to the positive quantity k, then the value of 4 sin
3 sin 2 sin sin is equal to
2 2 2 2
(A) 2 1 k (B) 2 1 k
(C) 2 k (D) none of these
Solution: B
Given < < < also sin = sin = sin = sin = k
and , , , are smallest positive angles
= , = 2 + , = 3
as sin = sin and >
sin = sin and >
sin = sin and > .
Putting these values in the given expansion, we have given expression
= 2 sin cos 2 1 sin 2 1 k .
2 2
Problem 20. If 3sin + 5cos = 5, then the value of 5sin – 3cos is equal to
(A) 5 (B) 3
(C) 4 (D) none of these
Solution: B
P-2022-CBSE-P1-MATHEMATICS-TRI
37
2
3sin = 5(1 – cos) = 5 2sin /2
tan/2 = 3/5
2
2 tan 1 tan 2
2
5sin – 3cos = 5 3
2 2
1 tan 1 tan
2 2
3 3 1 9
2
5 25
= 5 3
9 9
1 1
25 25
n
Problem 21. Let n be an odd integer. If sin n = b r sinr , for every value of , then
r 0
Solution: A
Writing = 0 on both sides, we get b0 = 0.
n
sinn r 1
Now,
sin
b sin
r 1
r .
Making 0, we get n = b1
Solution: B, C
We have
sinC cosC sin 2B C cos 2B C 2 2
sin 2B C sinC cos C cos 2B C 2 2
sin 180 A cosB sin 180 A sinB 2
sin A cosB sinB 2
1 1
sin A cosB sinB 1
2 2
sin A sin B 1, it is possible only when
4
sin A 1& sin B 1
4
A 90 & B 45 then C 45
P-2022-CBSE-P1-MATHEMATICS-TRI
38
Problem 23. If sin22.50 then
2 2 2 2
(A) (B)
2 2
1
(C) 2 (D) 4 2 2 2
2
Solution: C, D
2 2
sin22.50
2
2 2 1
2 , 2
4 2
Solution: A, B, C, D
sin / 2 2cos 2 / 2
tan . 1 sec .
2 cos / 2 cos
2 sin / 2 cos / 2
sin
= tan
cos cos
Repeating this process again and again, we get
fn tan 2n (1)
This (A) f2 tan 22 tan 1
16 16 4
(B) f3 tan 23. tan 1 etc.
32 32 4
Assertion / Reasoning
(A) Statement–1 is True, Statement–2 is true; Statement–2 is a correct explanation for
Statement–1
(B) Statement–1 is True, Statement -2 is true; Statement–2 is NOT a correct
explanation for Statement–1
(C) Statement –1 is True, Statement –2 is False
(D) Statement –1 is False, Statement –2 is True
Problem 25. Statement – 1: If A 0, B 0 and A B , then the maximum value of
3
tan A.tanB is 3.
P-2022-CBSE-P1-MATHEMATICS-TRI
39
tan A tanB
Statement – 2: tan A B , None of the angles A, B and (A+B)
1 tan A.tanB
should be an odd multiple of .
2
Solution: D
Using maximisation principle for symmetrical functions the value of tan A.tanB
will be maximum when A B .
1
So; For A B , the maximum value of tan A.tanB will be .
6 3
Problem 26. Statement – 1: The value of 5cos 3cos 3 lies in [ 4 ,10]
3
Statement – 2: The value of a cos x b sin x C lies in [C a2 b2 ,
C a2 b2 ]
Solution: A
5 cos 3 cos 3
3
3 3 3
5 cos cos sin 3
2 2
13 3 3
cos sin 3
2 2
The value lies between
2 2 2 2
13 3 3 13 3 3
3 and 3
2 2 2 2
i.e 4 and 10
Both statements I and II are true and statement II is a correct explanation of
statement I.
Comprehension – I
If A + B +C=1800, then
(i) sin2A sin2B sin2C = 4 sin A sin B sin C
A B C
(ii) cos2A cos2B cos2C 4cos cos cos
2 2 2
A B C
(iii) cos2A cos2B cos2C 4cos cos cos
2 2 2
A B C
(iv) cos A cosB cosC = 1 4 sin sin sin
2 2 2
(v) tan A tanB tanC tan A tanB tanC
(vi) cotBcot C cot Ccot A cot A cot B 1
A B C A B C
(vii) cot cot cot cot cot cot
2 2 2 2 2 2
P-2022-CBSE-P1-MATHEMATICS-TRI
40
Problem 27. If tan A + tan B + tanC= tan A. tan B. tan C, then
(A) A,B C, must be angles of a triangle
(B) The sum f any two of A, B ,C of equal to the third.
(C) A + B + C must be an integral multiple of
(D) None of these.
Solution: A
tan A tanB tanC tan A tanB tanC
tan(A B C)
1 tan A tanB tanB tanC tanC tan A
A B C n
Problem 28. In a triangle ABC, whose angles are acute and +ve such that A+B+C = and
A B C
cot cot cot K,then
2 2 2
(A) K 3 (B) K 3 3
(C) K 3 3 (D) none of these.
Solution: B
A B C S S3
tan 1
2 2 2 1 S2
S S2
tan 1 S2 1
2 1 S2
tan 2
A B B C C A
tan tan tan tan tan tan 1
2 2 2 2 2 2
C A B A B C
cot cot cot cot cot cot K
2 2 2 2 2 2
A B C
cot cot cot k
2 2 2
But we know that A G
A B C
cot cot cot 1/3
2 2 2 cot A cot B cot C
3 2 2 2
1/3 2/3 2
K 3K K 3 K 27
k3 3
Problem 29. If A, B, C are acute positive angles such that A + B + C = and cot A cot B cot
C = K, then
1 1
(A) K (B) K
3 3 3 3
1 1
(C) K (D) K
9 3
Solution: A
We Know that if A+ B +C=
tan A tanB tanC tan A tanB tanC...(1)
Since A.M. G.M,
P-2022-CBSE-P1-MATHEMATICS-TRI
41
tan A tanB tanC
(tan A tanB tanC)1/3
3
tan A tanB tanC 3(tan A tanB tanC)1/3
(tan A tanB tanC) 3(tan A tanB tanC)1/3
(tan A tanB tanC)3 3(tan A tanB tanC)1/3 (by(1))
3
tan A tanBC 27(tan A tanB tanC)
(tan A tanB tanC)2 27
tan A tanB tanC 3 3
1 1
cot A cotB cot C K
3 3 3 3
Comprehension – II
x, y, z are respectively the sines and p, q, r are respectively cosines of the angles , , which
2
are in A.P. with common difference .
3
Solution: C
2 2 4 4
yz + zx + xy = sin sin sin sin sub sin
3 3 3 3
1 2 2 2 6 4 4
= cos cos 2 cos cos 2 cos cos 2
2 3 3 3 3 3 3
1 3 3
= 2cos 2 cos cos 2 .
2 2 3 4
P-2022-CBSE-P1-MATHEMATICS-TRI
42
Matrix Match Type
Solution: C
(P) sin cos cos sin sin sin
2
1 1 1 1
cos sin cos sin 2 cos sin
2 2 2 2 2
1
cos
4 2 2
(Q) tan cos cot sin tan sin
2
cos sin
2
1
cos sin
2
1 1
sin sin2
4 2 2 4 8
(R) sin2 cos2 1
3
sin2 cos2 1
sin cos 3 sin2 .cos2 . 1 1
6 6
3 2
sin6 cos6
sin 2 1
4
(S) cos520 cos 680 cos1720
2cos1120 cos600 cos 680 cos1120 cos680 2cos900.cos 220 0
P-2022-CBSE-P1-MATHEMATICS-TRI
43
Problem 33. If A + B + C = , then match the following (equality must hold)
Column – I Column – II
(A) cos A + cos B + cos C (p) 3/2
A B C (q) – 3
(B) sin sin sin
2 2 2
1 3
(C) cos A cosB cosBcosC cosCcos A (r)
2 8
(D) cos A cos B cos C (s) – 1
Numerical Based
12 3 3
Problem 34. If sin 0 and cos then
13 2 5 2
65 sin 60 equals;
Solution: 4
sin sin .cos cos sin
12 3 5 4 56
13 5 13 5 65
65 sin 56
12 3
Problem 35. If and lie between 0 and such that cos and sin
2 13 5
then the value of 65 sin 2 56 is
Solution: 0
sin2 sin
5 4 12 3 56
13 5 13 5 65
65 sin 2 56 0
P-2022-CBSE-P1-MATHEMATICS-TRI
44
+
Problem 36. If A + B = 90° where A, B R , let maximum value of sinA + sinB is equal to t,
then t2 is ______
Solution: 2
sin A sinB A B
sin sin .
2 2 4
2 cos y 1 x y 2
Problem 37. If cosx = where x, y (0, ) let tan cot is equal to t, then t is
2 cos y 2 2
equal to _______
Solution: 3
1 tan2 (y / 2)
2 2 1
1 tan2 (x / 2) 1 tan y / 2
1 tan2 (x / 2) 1 tan2 y / 2
2
1 tan2 y / 2
y x
3 tan2 tan2
2 2
x y
tan cot 3 .
2 2
Problem 38. If sinx + siny cos cosx x R, then siny + cos is equal to ___________
Solution: 1
sinx + siny cos cosx x R.
Let x =
2
sin y 1 (sin y = 1)
1 + sin x cos cos x
cos cos x sin x 1
cos2 1 1
cos = 0.
Decimal Type
Problem 39. Determine the smallest positive value of x (in degree) for which
tan(x + 100) = tan(x + 50) tanx tan(x − 50).
Solution: 30.00
tan(x + 100) = tan(x + 50) tanx tan(x − 50).
tan x 1000
tan x 500 tan x
tan x 500
=
sin x 1000 cos x 500 sin x 50 sin x
0
sin 2x 50 sin150
0
cos50 cos 2x 50
0 0 0
sin 2x 50 sin150
0
cos 2x 50 cos50
0 0 0
P-2022-CBSE-P1-MATHEMATICS-TRI
45
Use componendo and dividendo
We get
sin 2x 500
cos500
sin150 0
cos 2x 500
cos50 + 2 sin(2x + 50 ) cos(2x + 500) = 0
0 0
2 1
Problem 40. If a = and cosa cos2a cos3a … cos999a = , then log2k = ___________
1999 k
Solution: 999.00
1
= cosa cos2a cos3a … cos999a
k
let p = sina sin2a … sin999a
2999 p
= (2sina cosa)(2sin2a cos2a)…(2cos999a sin999a)
k
2999 p
= sin2a sin4a … sin1998a
k
= (sin2a sin4a…sin998a)(–sin(2 – 1000a))(–sin(2 – 1002a)) … (–sin(2 –
1998a))
(sin2a sin4a … sin1998a)(sin999a sin997a … sina)
=p
2999 = k.
P-2022-CBSE-P1-MATHEMATICS-TRI
46
CHAPTER PRACTICE PROBLEM
Subjective:
Level I
1. Find the maximum and minimum value of sin2 x + 4cos 2x, for x R.
3 5 7 3
2. Prove that sin4 sin4 sin4 sin4 .
16 16 16 16 2
9
r
3. Find the value of cos2 18 .
r 1
Level II
1 cos 4
4. Prove that 0 for all .
2 sin 3
Objective:
x y 2
1. sin2 = , where x, y R, gives real if and only if
4xy
(A) x + y = 0 (B) x = y
(C) |x| = |y| 0 (D) none of these
A B A B
2. Let a = cosA + cosB – cos(A + B) and b = 4 sin . sin . cos . Then a – b is equal
2 2 2
to
(A) 1 (B) 0
(C) – 1 (D) none of these
5. If lies in fourth quadrant, then 4cos4 sin2 2 4 cos2 is equal to
2 2
(A) 1 (B) 2
(C) –2 (D) 0
P-2022-CBSE-P1-MATHEMATICS-TRI
47
6. If ( + + + ) = then cos cos -sin sin =
(A) 4 (B) 2
(C) 0 (D) none of these
7. If x + y = 2 then minimum value of sec x + sec y is, x, y 0,
2
(A) 2 cos (B) cos 2
(C) 2 sec (D) none of these
tan700 tan200
8. =
4 tan500
(A) 1 (B) 1/2
(C) –1 (D) –1/2
n n
cos A cosB sin A sinB
3. (n even or odd) =
sin A sinB cos A cosB
A B A B
(A) 2 tann (B) 2cot n
2 2
(C) 0 (D) None of these
P-2022-CBSE-P1-MATHEMATICS-TRI
48
4. If sin 1/ 5 and sin 3 / 5 ,then lies in the interval
(A) [0, / 4] (B) [ / 2,3 / 4]
(C) [3 / 4, ] (D) [ ,5 / 4]
Numerical Based
2. If A B , then (1 tan A)(1 tanB)
4
1 3
3. =
sin10 cos10
9 3 5
6. The expression 2cos .cos cos cos is equal to
13 13 13 13
P-2022-CBSE-P1-MATHEMATICS-TRI
49
ASSIGNMENT PROBLEMS
Subjective:
Level 0
17
3. Prove that – 4 cos2x + 3sinx for all real x.
8
2b
4. If tan x (a c), y a cos2 x 2b sin x cos x c sin2 x ,
ac
z a sin2 x 2b sin x cos x c cos2 x , prove that y – z = a – c.
3 5
5. If cos = - and sin , and lies in the third quadrant and lies in the fourth
5 13
quadrant, find tan( + ), cot( – ).
5 9 13 1
7. Prove that 1 sin 1 sin 1 sin 1 sin .
8 8 8 8 8
0 0
log10 tan30 log10 tan 890
8. Find the value of elog10 tan1 log10 tan 2 .
1
9. Find the minimum value of the expression .
cos2 x sin2 x
4 4
3 5 7 3
11. Prove that sin4 sin4 sin4 sin4 .
16 16 16 16 2
3
12. Prove that sin 20 sin 40 sin 80 = .
8
P-2022-CBSE-P1-MATHEMATICS-TRI
50
m n
13. If m tan ( 30°) = n tan ( + 120°), show that cos2 = .
2 m n
1
14. Prove that 2 sinA cos3A 2sin3A cosA = sin 4A.
2
sin sin 2
15. Prove that tan .
1 cos cos 2
3 5 7
18. Pr ove that sin4 sin4 sin4 sin4 3/ 2.
8 8 8 8
P-2022-CBSE-P1-MATHEMATICS-TRI
51
Level I
3 5 7
2. Find the value of cos cos cos cos .
8 8 8 8
7 x
4. If sinx + cosx = where x 0, , then find the value of tan .
2 4 2
tan tan
7. If , prove that sin2 + sin2 + sin2 = 0.
tan tan
9. Prove that the expression cos2 + cos2( + ) 2 cos cos cos ( + ) is independent
of .
12. If cos ( + ) = 4/5, sin ( ) = 5/13 and , lie between 0 and , find tan 2.
4
P-2022-CBSE-P1-MATHEMATICS-TRI
52
Level II
1. The angles of a quadrilateral are in G.P. with common ratio r > 1. If the largest angle is 8
times the smallest angle, find the angles of the quadrilateral in degrees.
1 cos 3x
2. Show that cos ec2x cos ec4x ,
2 sin x sin 2x sin 4x
cos 3x cos 5x cos 7x
and hence prove that ... n terms
sin 2x sin 4x sin 4x sin 6x sin 6x sin 8x
1
cos ec2x cos ec(2n 2)x .
2 sin x
3. If sin(z + y - x), sin(x + z - y), sin(y + x - z) be in A.P., prove that tanx, tany, tanz are also
in A.P.
1 7
5. If sinx cosy = and 3 tanx = 4 tany then show that sin(x + y) = .
4 16
3
6. If sin( cos) = cos( sin), then show that sin2 = .
4
acos+b ab
7. If cos= ,prove that tan tan .
a+bcos 2 ab 2
3
8. If A + B + C = , prove that cosA + cosB + cosC and hence deduce that
2
A B C 1
sin sin sin .
2 2 2 8
tan 3A sin 3A 2k 1
9. If k , prove that and hence deduce that either k > 3 or k < .
tan A sin A k 1 3
10. If a sin2 + b cos2 = m, b sin2 + a cos2 = n and a tan = b tan, prove that
1 1 1 1
.
n m a b
P-2022-CBSE-P1-MATHEMATICS-TRI
53
Objective:
Level I
2. If tanx. tany = a and x + y = /6, then tanx and tany satisfy the equation
(A) x2 3 (1 – a)x + a = 0 (B) 3 x2 – (1 – a)x + a 3 = 0
2 2
(C) x + 3 (1 + a)x - a = 0 (D) 3 x + (1 + a)x – a 3 = 0
2
1 sin cos
5. is equal to
1 sin cos
1 sin
(A) cot2 (B)
2 1 sin
(C) tan2/2 (D) none of these
6. k is rational if it is equal to
(A) sin 150 sin 300 (B) sin 150 cos 150
(C) sin 150 cos 750 (D) none of these
1 3 3 2 4
8. sin sin sin3 is equal to
sin 3 3 3
(A) 4/3 (B) 3/4
(C) 3/4 (D) none of these
1 sin 4A 1
9. If y = , then one of the value of y is
1 sin 4A 1
(A) tanA (B) cotA
(C) tan(/4 A) (D) cot(/4 + A)
P-2022-CBSE-P1-MATHEMATICS-TRI
54
10. The number of solutions of the equation sin (ex) = 5x + 5–x is
(A) 0 (B) 1
(C) 2 (D) none of these
3 4
15. If 0 < < , cos( + ) = and cos( – ) = , then sin2 is equals
4 5 5
(A) 1 (B) 0
(C) 2 (D) none of these
x2 y2
16. The equation sin2 = is possible if
2 xy
(A) x = y (B) x = -y
(C) 2x = y (D) none of these
1 1
18. If tan = , tan = , then + is equal to
2 3
(A) 0 (B) /2
(C) /4 (D)
tan690 tan660
20. =
1 tan690 tan660
(A) 1 (B) –1
(C) 0 (D) none of these
P-2022-CBSE-P1-MATHEMATICS-TRI
55
Level II
2. The maximum value of 1 + sin + 2 cos , R, equals
4 4
(A) 3 (B) 5
(C) 4 (D) none of these
5 7
3. The numerical value of sin . sin . sin is equal to
18 18 18
1
(A) 1 (B)
8
1
(C) (D) none of these
4
4. If tan. tan . tan = -1, ( 0 < < /2), then value of 3 sin - 4 cos3 =
3 3
(A) 1 (B) -1
(C) 1/ 2 (D) -1/ 2
5. If and are two distinct roots of the equation a tan x + b sec x = c, then tan ( + ) is
equal to
a2 c 2 a2 c 2
(A) 2 2
(B) 2
a c a c2
2ac 2ac
(C) 2 2
(D) 2
a c a c2
y
9. If cos (x – y), cos x and cos (x + y) are in H.P., then cos x sec is equal to
2
(A) 1 (B) 3
(C) 2 (D) none of these
P-2022-CBSE-P1-MATHEMATICS-TRI
56
2cos-1
10. If cos= , 0 ,0 , then tan cot is equal to
2-cos 2 2
(A) 1 (B) 2
(C) 3 (D) none of these
cos sin p q
11. If ,then is equal to
p q sec 2 cosec2
(A) p (B) q
(C) qp (D) none of these
12. For 0 1, 2, …, n /2 and cot 1 cot 2 … cot n = 1, the maximum value of
(cos 1) (cos 2) (cos 3) … (cos n) is
(A) 1/2n (B) 1/2n/2
(C) 21/n (D) none of these
15. If sinB is the geometric mean between sinA and cosA, then cos2B is equal to
(A) 2sin2 A (B) 2cos2 A
4 4
2 2
(C) 2cos A (D) 2sin A
4 4
2 4
16. Given a, b, c are non-zero. If asin = b sin c sin , then
3 3
(A) ab + bc + ca = 1 (B) ab + bc + ca = 0
1 1 1 1 1 1
(C) ab bc ca (D) 1
a b c a b c
1
17. If tan x , then sec tan is equal to
4x
1
(A) 2x (B)
2x
1
(C) 2x (D)
2x
P-2022-CBSE-P1-MATHEMATICS-TRI
57
18. If tan (cosec sin ) , then tan2 can be equal to
2 2
(A) 2 5 (B) 2 5
(C) 9 4 5 2 5
(D) 9 4 5 2 5
10
2
19. If R [0, ] for 1 k 10, then the maximum value of (1 sin R )(1 cos2 R ) is
R1
10 10
3 9
(A) (B)
2 4
20 5
3 9
(C) (D)
2 4
20. If x = Xcos Ysin, y = Xsin + Ycos and x2 + 4xy + y2 = AX2 + BY2 0 , then
2
(A) (B)
6 4
(C) A = B (D) B = 1
21. The quadratic equation whose roots are sec2 and cosec2 can be
(A) x2 2x + 2 = 0 (B) x2 5x + 5 = 0
2
(C) x 7x + 7 = 0 (D) x2 9x + 9 = 0
22. If , , are acute angle such that cos = tan , cos = tan and cos = tan , then
(A) sin = sin (B) sin = sin
5 1
(C) sin = (D) none of these
2
Assertion / Reason:
Questions (12–13) contain STATEMENT-1 (Assertion) and STATEMENT-2 (Reason).
(A) Statement -1 is True, Statement -2 is True; Statement-2 is a correct explanation for
Statement-1
(B) Statement -1 is True, Statement -2 is True; Statement-2 is NOT a correct explanation
for Statement-1
(C) Statement -1 is True, Statement -2 is False
(D) Statement -1 is False, Statement -2 is True
24. STATEMENT -1: If A, B, C are the angles of a triangle such that angle A is obtuse then
tan B tan C > 1.
because
tanB tan C
STATEMENT -2: In any triangle tan A = .
tanB tan C 1
P-2022-CBSE-P1-MATHEMATICS-TRI
58
25. STATEMENT -1: The minimum value of the expression sin + sin + sin , where , ,
are real numbers such that + + = is negative.
because
STATEMENT -2: , , are angles of a triangle.
Comprehension:
Read the following passage and answer the questions that follow:
n n
If Pn = sin + cos , where n is whole number and R, then
P7 P5
27. The value of is
P5 P3
P7 P5
(A) (B)
P5 P3
P3 P3
(C) (D)
P1 P5
Read the following passage and answer the questions that follow
ABC is a triangle, the incircle touches the sides BC, CA and AB at D, E and F respectively. BD,
CE and AF are consecutive natural numbers. I is the incentre of the triangles. The radius of the
incircle is 4 units.
A B C
31. cot cot cot is equal to
2 2 2
(A) 7/4 (B) 3/4
(C) 21/4 (D) ½
P-2022-CBSE-P1-MATHEMATICS-TRI
59
Match Matrix:
If log2 27 a and
Cube root of the reciprocal of
3a (S)
(IV) log6 16 k then k is a (iv) 1 2 4
3a cos .cos .cos
9 9 9
positive integer number.
(I). Which of the following options is the only CORRECT combination?
(A) (II) (iv) (P) (B) (II) (iv) (Q)
(C) (II) (iii) (P) (D) (II) (ii) (Q)
(II). Which of the following options is the only CORRECT combination?
(A) (III) (i) (Q) (B) (III) (iii) (P)
(C) (III) (i) (P) (D) (III) (i) (S)
(III). Which of the following options is the only INCORRECT combination?
(A) (IV) (iii) (R) (B) (IV) (iii) (P)
(C) (I) (ii) (S) (D) (II) (ii) (P)
P-2022-CBSE-P1-MATHEMATICS-TRI
60
Numerical Based
34. Let 0 < A, B < satisfying the equalities 3sin2A + 2sin2B = 1 and 3sin2A – 2sin2B = 0.
2
Then A + 2B = , where k is equal to _____________.
k
35. If A lies in the second quadrant and 3tanA + 4 = 0, then the value of
10
(2cotA – 5cosA + sinA) is equal to ____________
23
Decimal Type
a
1 cosa tan2
37. If 2 k cosa where k, w and p have no common factor other than
a w pcosa
sin2
2
1, then the value k2 + w2 + p2 is equal to
38. If y = (sin x + cosec x)2 + (cos x + sec x)2 + (tan x + cot x)2
p
where x R – {x ; x = n, (2n + 1)/2 n I} and minimum value of y = p then is
3
where [.] denotes the greatest function
1
39. If , , , be first 4 positive solutions of sin x = with < < < , then value of
4
sin 2 sin 3 sin 4 sin is
2 2 2 2
P-2022-CBSE-P1-MATHEMATICS-TRI
61
ANSWERS TO CHAPTER PRACTICE PROBLEMS
Subjective:
Level I
3. 4
5. 3
Level II
Objective:
1. C 2. A 3. A 4. A
5. B 6. C 7. C 8. B
9. A 10. D
1. D 2. A, C 3. B, C 4. A, C
Numerical Based
1. 0 2. 2 3. 4 4. 1
5. 0 6. 0 7. 4 8. 0
P-2022-CBSE-P1-MATHEMATICS-TRI
62
ANSWERS TO ASSIGNMENT PROBLEMS
Subjective:
Level 0
3
1. 8. 1
2
33 16 1
5. , 9.
56 63 2
Level I
1 7 1
2. 4.
8 3
1
1
2 56
8. 2 12.
33
Level II
3. (i)
2ab
, (ii)
b 2
a2 b
2
a2 2
a2 b2 a2 b2
Objective:
Level I
1. B 2. B 3. A 4. A
5. C 6. B 7. B 8. C
9. B 10. A 11. D 12. B
13. C 14. B 15. A 16. A
17. D 18. C 19. A 20. B
Level II
1. A 2. C 3. B 4. C
5. D 6. D 7. A 8. B
9. C 10. C 11. A 12. B
P-2022-CBSE-P1-MATHEMATICS-TRI
63
13. C 14. B 15. A, C 16. B, C
17. A, D 18. B, C 19. B, C 20. B, C, D
21. B, C, D 22. A, B, C 23. A, C 24. D
25. C 26. D 27. C 28. C
29. C 30. D 31. C
32. (A) (r) (B) (s) (C) (q) (D) (p)
33. (I). A (II). A (III). D
34. 2 35. 1 36. 4 37. 6.00
38. 4.00 39. 2.24
P-2022-CBSE-P1-MATHEMATICS-TRI